Day-558 | Daily MCQs | UPSC Prelims | ENVIRONMENT AND ECOLOGY

Day-558

Time limit: 0

Quiz-summary

0 of 5 questions completed

Questions:

  1. 1
  2. 2
  3. 3
  4. 4
  5. 5

Information

DAILY MCQ

You have already completed the quiz before. Hence you can not start it again.

Quiz is loading...

You must sign in or sign up to start the quiz.

You have to finish following quiz, to start this quiz:

Results

0 of 5 questions answered correctly

Your time:

Time has elapsed

You have reached 0 of 0 points, (0)

Categories

  1. Not categorized 0%
  1. 1
  2. 2
  3. 3
  4. 4
  5. 5
  1. Answered
  2. Review
  1. Question 1 of 5
    1. Question

    1. The ‘Central Asian Mammals Initiative’, recently seen in the news, is started by which one of the following organisations?

    Correct

    Answer: C
    Explanation:
    The Central Asian region harbours the largest intact and still interconnected grasslands worldwide. As such, it is of global importance for many migratory and nomadic mammals which rely on the vast steppe, desert and mountain ecosystems that enable the essential long-distance movements which ensure their survival. Mass migrants in this region include CMS-listed species such as the Saiga antelope, Mongolian gazelle and Khulan.
    The UN Convention on the Conservation of Migratory Species of Wild Animals (CMS), also known as the Bonn Convention, has strengthened its engagement in Central Asia to conserve migratory mammals, their habitats and the vital role they play in preserving intact ecosystems. The Central Asian Mammals Initiative (CAMI) (Resolution 11.24 (Rev.COP13)) and its associated Programme of Work were adopted by CMS Parties at the COP11 and revised at COP13 in Gandhinagar, India (February 2020).
    With an initiative to strengthen the conservation of Central Asian migratory mammals, CMS aims to provide a common framework to coordinate conservation activities in the region and to coherently address major threats to migratory species.
    Additional information about CMS
    • The UN Convention on the Conservation of Migratory Species of Wild Animals (CMS) was signed in 1979 and has been into force since 1983. It is an intergovernmental treaty under the UNEP- popularly known as Bonn Convention.
    • As the only global convention specializing in the conservation of (terrestrial, avian and marine) migratory species, their habitats and migration routes, CMS complements and co-operates with a number of other international organizations, NGOs and partners in the media as well as in the corporate sector.

    Incorrect

    Answer: C
    Explanation:
    The Central Asian region harbours the largest intact and still interconnected grasslands worldwide. As such, it is of global importance for many migratory and nomadic mammals which rely on the vast steppe, desert and mountain ecosystems that enable the essential long-distance movements which ensure their survival. Mass migrants in this region include CMS-listed species such as the Saiga antelope, Mongolian gazelle and Khulan.
    The UN Convention on the Conservation of Migratory Species of Wild Animals (CMS), also known as the Bonn Convention, has strengthened its engagement in Central Asia to conserve migratory mammals, their habitats and the vital role they play in preserving intact ecosystems. The Central Asian Mammals Initiative (CAMI) (Resolution 11.24 (Rev.COP13)) and its associated Programme of Work were adopted by CMS Parties at the COP11 and revised at COP13 in Gandhinagar, India (February 2020).
    With an initiative to strengthen the conservation of Central Asian migratory mammals, CMS aims to provide a common framework to coordinate conservation activities in the region and to coherently address major threats to migratory species.
    Additional information about CMS
    • The UN Convention on the Conservation of Migratory Species of Wild Animals (CMS) was signed in 1979 and has been into force since 1983. It is an intergovernmental treaty under the UNEP- popularly known as Bonn Convention.
    • As the only global convention specializing in the conservation of (terrestrial, avian and marine) migratory species, their habitats and migration routes, CMS complements and co-operates with a number of other international organizations, NGOs and partners in the media as well as in the corporate sector.

  2. Question 2 of 5
    2. Question

    2. Consider the following:
    1. Flapshell turtle
    2. Blackshell turtle
    3. Indian tent turtle
    How many of the above species of turtle are herbivores?

    Correct

    Answer: A
    Explanation:
    Option 3 is correct.
    Indian Tent Turtle: The species is native to India, Nepal and Bangladesh with three subspecies recorded from the region viz., P. t. tentoria, P. t. circumdata and P. t. flaviventer.
    ● P. t. tentoria occurs in peninsular India and is recorded from Orissa, Maharashtra, Andhra Pradesh, Assam and Madhya Pradesh.
    ● P. t. circumdata occurs in the western tributaries of Ganga and the rivers of Gujarat. It is found in Rajasthan, Madhya Pradesh, Uttar Pradesh and Gujarat.
    ● P. t. flaviventer occurs in the northern tributaries of Ganga and is recorded from Uttar Pradesh, Bihar, West Bengal and Assam.
    Its habitats include still water pools on river side and slow running water near the river banks. These are active swimmers and are mainly herbivorous.
    Due to the attractive appearance of the species, they are illegally traded in the pet market.

    Conservation Status:
    ● IUCN : Least concern
    ● Wildlife Protection Act, 1972 : Schedule I
    ● CITES : Appendix II
    Options 1 and 2 are incorrect: Both flapshell and black turtle are omnivores. They feed on aquatic plants and insects, frogs, shrimp, snails, fruits, flowers, grasses and seeds.
    Flapshell turtle
    • Flapshell turtle lives in shallow, stagnant waters in lakes, rivers, ponds and streams in India, Pakistan, Sri Lanka, Nepal, Bangladesh and Myanmar.
    • It is a freshwater species of turtle that is adapted to survive in drought conditions. It often burrows and moves from one water hole to another to avoid desiccation. It also undergoes aestivation to survive dry conditions.
    • It prefers swampy areas with soil and exposure to sunlight as common nesting sites. It is generally solitary and active during the day.
    • It is listed as Schedule I species in Wildlife (Protection) Act, 1972, thus it has been accorded the highest level of protection in India.
    Blackshell turtle
    • Black turtles (freshwater turtle species) are commonly found in the Indomalayan biogeographical realm in India, Nepal, Sri lanka, Myanmar and Bangladesh.
    • It is listed under Schedule II of Wildlife (Protection) Act, 1972.

    Incorrect

    Answer: A
    Explanation:
    Option 3 is correct.
    Indian Tent Turtle: The species is native to India, Nepal and Bangladesh with three subspecies recorded from the region viz., P. t. tentoria, P. t. circumdata and P. t. flaviventer.
    ● P. t. tentoria occurs in peninsular India and is recorded from Orissa, Maharashtra, Andhra Pradesh, Assam and Madhya Pradesh.
    ● P. t. circumdata occurs in the western tributaries of Ganga and the rivers of Gujarat. It is found in Rajasthan, Madhya Pradesh, Uttar Pradesh and Gujarat.
    ● P. t. flaviventer occurs in the northern tributaries of Ganga and is recorded from Uttar Pradesh, Bihar, West Bengal and Assam.
    Its habitats include still water pools on river side and slow running water near the river banks. These are active swimmers and are mainly herbivorous.
    Due to the attractive appearance of the species, they are illegally traded in the pet market.

    Conservation Status:
    ● IUCN : Least concern
    ● Wildlife Protection Act, 1972 : Schedule I
    ● CITES : Appendix II
    Options 1 and 2 are incorrect: Both flapshell and black turtle are omnivores. They feed on aquatic plants and insects, frogs, shrimp, snails, fruits, flowers, grasses and seeds.
    Flapshell turtle
    • Flapshell turtle lives in shallow, stagnant waters in lakes, rivers, ponds and streams in India, Pakistan, Sri Lanka, Nepal, Bangladesh and Myanmar.
    • It is a freshwater species of turtle that is adapted to survive in drought conditions. It often burrows and moves from one water hole to another to avoid desiccation. It also undergoes aestivation to survive dry conditions.
    • It prefers swampy areas with soil and exposure to sunlight as common nesting sites. It is generally solitary and active during the day.
    • It is listed as Schedule I species in Wildlife (Protection) Act, 1972, thus it has been accorded the highest level of protection in India.
    Blackshell turtle
    • Black turtles (freshwater turtle species) are commonly found in the Indomalayan biogeographical realm in India, Nepal, Sri lanka, Myanmar and Bangladesh.
    • It is listed under Schedule II of Wildlife (Protection) Act, 1972.

  3. Question 3 of 5
    3. Question

    3. The ‘Bridgetown Initiative’, often seen in the news, is concerned with:

    Correct

    Answer: A
    Explanation:
    The Bridgetown Initiative, led by Barbados, is a proposal to reform the world of development finance, particularly how rich countries help poor countries cope with and adapt to climate change.
    Barbados is one of the Caribbean’s most vulnerable nations to climate change. Hurricanes are more frequent and intense and the island is also at risk from rising sea levels, storm surges, coastal erosion, droughts and flooding. Now the government of Barbados, led by Mia Mottley, the island’s first woman prime minister, is championing a radical initiative that could transform how lending is made to developing nations in a climate crisis.
    The key demands of the Bridgetown Initiative are:
    1. LIQUIDITY SUPPORT
    ● U.N. member states should fast-track the transfer of $100 billion in so-called ‘Special Drawing Rights’, a monetary reserve currency, to programmes that support climate resilience and subsidise lending to low-income countries.
    ● The International Monetary Fund should also immediately suspend surcharges – additional interest payments imposed on heavily indebted borrowing countries – for two to three years.
    ● It should also restore “enhanced access limits” established during the COVID pandemic for two emergency financial support instruments, the Rapid Credit Facility (RCF) and Rapid Financing Instruments.
    2. DEBT SUSTAINABILITY
    ● G20 creditor countries should redesign their Common Framework for restructuring the debt of poor countries in default, notably by speeding up debt relief talks and allowing middle-income countries to access it.
    ● The IMF should encourage the restructuring of unsustainable debt in a way that is consistent across countries, and change the way it analyses the debt to incentivise investments that create future savings, such as those for climate adaptation.
    ● Public and private creditors should include disaster clauses in lending deals to allow countries to divert debt payments to disaster relief; and refinance high-interest and short-term debt with credit guarantees and longer maturities.
    ● U.N. member states should agree to raise $100 billion a year for a fund to help pay for the climate-related loss and damage suffered by developing countries.
    3. PRIVATE CAPITAL
    ● The IMF and multilateral development banks should offer $100 billion a year in currency risk guarantees to help drive private sector investment in projects that would help developing countries make the transition to a low-carbon economy.
    4. DEVELOPMENT LENDING
    ● The G20 and other shareholders of the World Bank, IMF and development institutions should fully implement the 2022 recommendations of a panel of experts aimed at boosting lending by the multilateral development banks.
    ● Increase the leveraging of the World Bank’s International Development Association, which provides concessional finance; fully fund its emergency support facility to $6 billion by end-2023; and scale up the IDA’s funding to $279 billion.
    ● Raise the access limits to concessional finance through the Poverty Reduction and Growth Trust and the Resilience & Sustainability Trust.
    ● Assess funding eligibility in light of a country’s vulnerability and provide low-cost, 50-year loans to help them invest in areas including climate resilience, water security, pandemic preparedness and access to renewable energy.
    ● Simplify and harmonise the way countries can apply to access loans across the world, and provide more support in the process. The international financial institutions should also finance development plans that help protect shared resources.

    Incorrect

    Answer: A
    Explanation:
    The Bridgetown Initiative, led by Barbados, is a proposal to reform the world of development finance, particularly how rich countries help poor countries cope with and adapt to climate change.
    Barbados is one of the Caribbean’s most vulnerable nations to climate change. Hurricanes are more frequent and intense and the island is also at risk from rising sea levels, storm surges, coastal erosion, droughts and flooding. Now the government of Barbados, led by Mia Mottley, the island’s first woman prime minister, is championing a radical initiative that could transform how lending is made to developing nations in a climate crisis.
    The key demands of the Bridgetown Initiative are:
    1. LIQUIDITY SUPPORT
    ● U.N. member states should fast-track the transfer of $100 billion in so-called ‘Special Drawing Rights’, a monetary reserve currency, to programmes that support climate resilience and subsidise lending to low-income countries.
    ● The International Monetary Fund should also immediately suspend surcharges – additional interest payments imposed on heavily indebted borrowing countries – for two to three years.
    ● It should also restore “enhanced access limits” established during the COVID pandemic for two emergency financial support instruments, the Rapid Credit Facility (RCF) and Rapid Financing Instruments.
    2. DEBT SUSTAINABILITY
    ● G20 creditor countries should redesign their Common Framework for restructuring the debt of poor countries in default, notably by speeding up debt relief talks and allowing middle-income countries to access it.
    ● The IMF should encourage the restructuring of unsustainable debt in a way that is consistent across countries, and change the way it analyses the debt to incentivise investments that create future savings, such as those for climate adaptation.
    ● Public and private creditors should include disaster clauses in lending deals to allow countries to divert debt payments to disaster relief; and refinance high-interest and short-term debt with credit guarantees and longer maturities.
    ● U.N. member states should agree to raise $100 billion a year for a fund to help pay for the climate-related loss and damage suffered by developing countries.
    3. PRIVATE CAPITAL
    ● The IMF and multilateral development banks should offer $100 billion a year in currency risk guarantees to help drive private sector investment in projects that would help developing countries make the transition to a low-carbon economy.
    4. DEVELOPMENT LENDING
    ● The G20 and other shareholders of the World Bank, IMF and development institutions should fully implement the 2022 recommendations of a panel of experts aimed at boosting lending by the multilateral development banks.
    ● Increase the leveraging of the World Bank’s International Development Association, which provides concessional finance; fully fund its emergency support facility to $6 billion by end-2023; and scale up the IDA’s funding to $279 billion.
    ● Raise the access limits to concessional finance through the Poverty Reduction and Growth Trust and the Resilience & Sustainability Trust.
    ● Assess funding eligibility in light of a country’s vulnerability and provide low-cost, 50-year loans to help them invest in areas including climate resilience, water security, pandemic preparedness and access to renewable energy.
    ● Simplify and harmonise the way countries can apply to access loans across the world, and provide more support in the process. The international financial institutions should also finance development plans that help protect shared resources.

  4. Question 4 of 5
    4. Question

    4. The buttonwood trees have drawn much attention in recent times in states like Goa. Which one of the following best explains the reason behind it?

    Correct

    Answer: D
    Explanation:
    Conocarpus, also known as buttonwood tree, is an invasive species like Prosopis juliflora and Lantana camara which raises much apprehension among environmentalists since the tree is used for urban greening in various parts of the country. It is renowned for its use in public spaces for its ornamental looks but is known to have negative effects on the local biodiversity.
    One of the major concerns is that Conocarpus trees have deep roots that can damage telecommunication lines, drainage systems, and freshwater systems.
    Secondly, the trees of this species flower in winter and spread pollen in nearby areas. It is learnt that this is causing diseases like cold, cough, asthma, allergy etc.
    With glossy, dark-green leaves, Conocarpus is an evergreen tree species. The animals do not find this fast-growing species as palatable enough.
    Additional information about this tree:
    ● It is a mangrove tree species that can grow well in acidic and alkaline soil conditions.
    ● It has high water-absorption capacity.
    ● There are two species of Conocarpus (buttonwood) trees, with several varieties and hybrids — Conocarpus erectus, which is more widely used in India, is native to South America, and C. lancifolius is native to East Africa.
    ● It is an evergreen tree species which is ideal for seashore plantation since it is highly tolerant of salt and drought conditions.
    ● The strong wood of buttonwood was used for firewood, cabinet-making (furniture) and charcoal-making.
    ● It propagates by seeds or stem cuttings.
    ● Serious pollen allergies and respiratory problems have been reported in the vicinity of their plantations, which has led to civic agencies taking action to ban them. The problems had already been noted in Kuwait and other parts of the Middle East, as well as in Karachi and Faizabad in Pakistan, when they started being used in greening projects in India.

    Incorrect

    Answer: D
    Explanation:
    Conocarpus, also known as buttonwood tree, is an invasive species like Prosopis juliflora and Lantana camara which raises much apprehension among environmentalists since the tree is used for urban greening in various parts of the country. It is renowned for its use in public spaces for its ornamental looks but is known to have negative effects on the local biodiversity.
    One of the major concerns is that Conocarpus trees have deep roots that can damage telecommunication lines, drainage systems, and freshwater systems.
    Secondly, the trees of this species flower in winter and spread pollen in nearby areas. It is learnt that this is causing diseases like cold, cough, asthma, allergy etc.
    With glossy, dark-green leaves, Conocarpus is an evergreen tree species. The animals do not find this fast-growing species as palatable enough.
    Additional information about this tree:
    ● It is a mangrove tree species that can grow well in acidic and alkaline soil conditions.
    ● It has high water-absorption capacity.
    ● There are two species of Conocarpus (buttonwood) trees, with several varieties and hybrids — Conocarpus erectus, which is more widely used in India, is native to South America, and C. lancifolius is native to East Africa.
    ● It is an evergreen tree species which is ideal for seashore plantation since it is highly tolerant of salt and drought conditions.
    ● The strong wood of buttonwood was used for firewood, cabinet-making (furniture) and charcoal-making.
    ● It propagates by seeds or stem cuttings.
    ● Serious pollen allergies and respiratory problems have been reported in the vicinity of their plantations, which has led to civic agencies taking action to ban them. The problems had already been noted in Kuwait and other parts of the Middle East, as well as in Karachi and Faizabad in Pakistan, when they started being used in greening projects in India.

  5. Question 5 of 5
    5. Question

    5. Consider the following:
    1. Squids
    2. Shrimps
    3. Saltwater crocodiles
    4. Porpoises
    5. Seals
    How many of the above are the examples of nekton species?

    Correct

    Answer: D
    Explanation:
    Options 1, 2, 3, 4 and 5 are correct.
    Animals that swim or move freely in the ocean are nektons. Nekton comes in all shapes and sizes. They live in shallow and deep ocean waters. Most nekton eat zooplankton, other nektons or they scavenge for waste.
    There are three types of nektons:
    The largest group of nektons are chordates and have bones or cartilage. This group includes:
    ● bony fish
    ● whales
    ● sharks
    ● turtles
    ● snakes
    ● eels
    ● porpoises
    ● dolphins
    ● seals
    Molluscan nekton are animals like octopus and squid.
    Arthropod nekton are animals like shrimp. Most arthropods are benthos or bottom dwelling organisms.

    Incorrect

    Answer: D
    Explanation:
    Options 1, 2, 3, 4 and 5 are correct.
    Animals that swim or move freely in the ocean are nektons. Nekton comes in all shapes and sizes. They live in shallow and deep ocean waters. Most nekton eat zooplankton, other nektons or they scavenge for waste.
    There are three types of nektons:
    The largest group of nektons are chordates and have bones or cartilage. This group includes:
    ● bony fish
    ● whales
    ● sharks
    ● turtles
    ● snakes
    ● eels
    ● porpoises
    ● dolphins
    ● seals
    Molluscan nekton are animals like octopus and squid.
    Arthropod nekton are animals like shrimp. Most arthropods are benthos or bottom dwelling organisms.

window.wpAdvQuizInitList = window.wpAdvQuizInitList || []; window.wpAdvQuizInitList.push({ id: '#wpAdvQuiz_597', init: { quizId: 597, mode: 0, globalPoints: 10, timelimit: 0, resultsGrade: [0], bo: 0, qpp: 0, catPoints: [10], formPos: 0, lbn: "Finish quiz", json: {"2779":{"type":"single","id":2779,"catId":0,"points":2,"correct":[0,0,1,0]},"2780":{"type":"single","id":2780,"catId":0,"points":2,"correct":[1,0,0,0]},"2781":{"type":"single","id":2781,"catId":0,"points":2,"correct":[1,0,0,0]},"2782":{"type":"single","id":2782,"catId":0,"points":2,"correct":[0,0,0,1]},"2783":{"type":"single","id":2783,"catId":0,"points":2,"correct":[0,0,0,1]}} } });




Day-557 | Daily MCQs | UPSC Prelims | CURRENT DEVELOPMENTS

Day-557

Time limit: 0

Quiz-summary

0 of 5 questions completed

Questions:

  1. 1
  2. 2
  3. 3
  4. 4
  5. 5

Information

DAILY MCQ

You have already completed the quiz before. Hence you can not start it again.

Quiz is loading...

You must sign in or sign up to start the quiz.

You have to finish following quiz, to start this quiz:

Results

0 of 5 questions answered correctly

Your time:

Time has elapsed

You have reached 0 of 0 points, (0)

Categories

  1. Not categorized 0%
  1. 1
  2. 2
  3. 3
  4. 4
  5. 5
  1. Answered
  2. Review
  1. Question 1 of 5
    1. Question

    1. With reference to the Kampala Declaration on climate change, consider the following statements:
    1. It is a pioneering framework adopted by 48 African nations to manage climate-induced migration.
    2. It was co-hosted by the Governments of Kenya and Uganda with support from the Office of the High Commissioner for Human Rights (OHCHR) and the United Nations Framework Convention on Climate Change (UNFCCC).
    Which of the statements given above is/are correct?

    Correct

    Answer: A
    Context: In August 2023, a total of 48 African countries have now agreed to adopt the Kampala Ministerial Declaration on Migration, Environment and Climate Change (KDMECC) to address the nexus of human mobility and climate change in the continent.
    Explanation:
    Statement 1 is correct: It is a pioneering framework adopted by 48 African nations to manage climate-induced migration.
    Statement 2 is incorrect: It was co-hosted by the Governments of Kenya and Uganda with support from the International Organization for Migration (IOM) and the United Nations Framework Convention on Climate Change (UNFCCC).
    Additional information:
    ● KDMECC was originally signed and agreed upon by 15 African states in Kampala, Uganda in July 2022.
    ● The Declaration is the first comprehensive, action-oriented framework led by Member States to address climate-induced mobility in a practical and effective manner.
    The KDMECC aims to address the challenges associated with climate-induced migration through several key objectives:
    ● Comprehensive Framework: Providing member states with a comprehensive framework to develop and implement policies and strategies addressing climate-induced migration.
    ● Leveraging Migration for Development: Encouraging the integration of migration into sustainable development strategies, ensuring it benefits both migrants and host communities.
    ● Inclusivity: Prioritizing the voices of vulnerable groups, including youth, women, and those in disadvantaged situations, to ensure their needs and concerns are central to the response.

    Incorrect

    Answer: A
    Context: In August 2023, a total of 48 African countries have now agreed to adopt the Kampala Ministerial Declaration on Migration, Environment and Climate Change (KDMECC) to address the nexus of human mobility and climate change in the continent.
    Explanation:
    Statement 1 is correct: It is a pioneering framework adopted by 48 African nations to manage climate-induced migration.
    Statement 2 is incorrect: It was co-hosted by the Governments of Kenya and Uganda with support from the International Organization for Migration (IOM) and the United Nations Framework Convention on Climate Change (UNFCCC).
    Additional information:
    ● KDMECC was originally signed and agreed upon by 15 African states in Kampala, Uganda in July 2022.
    ● The Declaration is the first comprehensive, action-oriented framework led by Member States to address climate-induced mobility in a practical and effective manner.
    The KDMECC aims to address the challenges associated with climate-induced migration through several key objectives:
    ● Comprehensive Framework: Providing member states with a comprehensive framework to develop and implement policies and strategies addressing climate-induced migration.
    ● Leveraging Migration for Development: Encouraging the integration of migration into sustainable development strategies, ensuring it benefits both migrants and host communities.
    ● Inclusivity: Prioritizing the voices of vulnerable groups, including youth, women, and those in disadvantaged situations, to ensure their needs and concerns are central to the response.

  2. Question 2 of 5
    2. Question

    2. With reference to the term ‘Medicane’, seen in news, consider the following statements:
    1. Medicanes are temperate cyclones originating in mediterranean region.
    2. Like tropical cyclones, Medicanes form in hot conditions at the end of summer.
    3. They are also formed over the Black Sea.
    How many of the statements given above are correct?

    Correct

    Answer: B
    Context: The flash flood that has killed thousands of people in Libya in September 2023 followed a “medicane”, a rare but destructive weather phenomenon that scientists believe will intensify in a warming world.
    Explanation:
    Statement 1 is incorrect: Medicanes are tropical cyclones originating in the Mediterranean region.
    Mediterranean cyclone or hurricane which is so-called Medicane are not rare phenomenon. Such large storms happen in the Mediterranean region every few years.
    Statement 2 is correct: Like tropical cyclones, medicanes form in hot conditions at the end of summer.
    Statement 3 is correct: Medicanes used to occur only in the Mediterranean Sea, but in the last two decades they have been formed over the Black Sea too.
    Additional information:
    ● Mediterranean tropical-like cyclones are commonly known as ‘Medicanes’ while a rapidly rotating storm system with a low-pressure centre, a closed low-level atmospheric circulation with strong winds and a spiral arrangement of thunderstorms are called ‘Hurricanes’.
    ● Medicanes are not formally classified as cyclones and are not monitored by any agency.
    ● National Oceanic and Atmospheric Administration (NOAA) and Hellenic National Meteorological Service are the governing agencies for Medicanes.
    ● The cyclones in the Mediterranean Sea do not form easily because it is dry but due to SST (sea surface temperature), it has caused floods in Italy, Spain and France.

    Incorrect

    Answer: B
    Context: The flash flood that has killed thousands of people in Libya in September 2023 followed a “medicane”, a rare but destructive weather phenomenon that scientists believe will intensify in a warming world.
    Explanation:
    Statement 1 is incorrect: Medicanes are tropical cyclones originating in the Mediterranean region.
    Mediterranean cyclone or hurricane which is so-called Medicane are not rare phenomenon. Such large storms happen in the Mediterranean region every few years.
    Statement 2 is correct: Like tropical cyclones, medicanes form in hot conditions at the end of summer.
    Statement 3 is correct: Medicanes used to occur only in the Mediterranean Sea, but in the last two decades they have been formed over the Black Sea too.
    Additional information:
    ● Mediterranean tropical-like cyclones are commonly known as ‘Medicanes’ while a rapidly rotating storm system with a low-pressure centre, a closed low-level atmospheric circulation with strong winds and a spiral arrangement of thunderstorms are called ‘Hurricanes’.
    ● Medicanes are not formally classified as cyclones and are not monitored by any agency.
    ● National Oceanic and Atmospheric Administration (NOAA) and Hellenic National Meteorological Service are the governing agencies for Medicanes.
    ● The cyclones in the Mediterranean Sea do not form easily because it is dry but due to SST (sea surface temperature), it has caused floods in Italy, Spain and France.

  3. Question 3 of 5
    3. Question

    3. Often seen in news, the term ‘Atlantic Declaration’ refers to:

    Correct

    Answer: C
    Context: The United Kingdom and the United States (US) President signed “The Atlantic Declaration: A Framework for a Twenty-First Century US-UK Economic Partnership” during the latter’s visit to Washington in June 2023.
    Explanation:
    ● Declared as the first of its kind, the declaration and its action plans are expected to promote a “new type of innovative partnership” to cover all areas of cooperation including technology, economy and trade.
    ● While deepening the trade and investment relations, the declaration is also expected to strengthen the UK-US cooperation in sectors like defence, health, space, and science.
    ● The new declaration highlights that the nature of national security is changing with economics, and it is becoming more intertwined.
    ● The US and the UK are facing new international challenges including from “authoritarian states” such as China and Russia, along with disruptive technologies, non-state actions and transnational issues like climate change.
    ● To deal with the emerging challenges, both countries have worked together to strengthen their resilience in trade and technology; and cooperated to expand the scope of their defence, security, and intelligence relations.

    Incorrect

    Answer: C
    Context: The United Kingdom and the United States (US) President signed “The Atlantic Declaration: A Framework for a Twenty-First Century US-UK Economic Partnership” during the latter’s visit to Washington in June 2023.
    Explanation:
    ● Declared as the first of its kind, the declaration and its action plans are expected to promote a “new type of innovative partnership” to cover all areas of cooperation including technology, economy and trade.
    ● While deepening the trade and investment relations, the declaration is also expected to strengthen the UK-US cooperation in sectors like defence, health, space, and science.
    ● The new declaration highlights that the nature of national security is changing with economics, and it is becoming more intertwined.
    ● The US and the UK are facing new international challenges including from “authoritarian states” such as China and Russia, along with disruptive technologies, non-state actions and transnational issues like climate change.
    ● To deal with the emerging challenges, both countries have worked together to strengthen their resilience in trade and technology; and cooperated to expand the scope of their defence, security, and intelligence relations.

  4. Question 4 of 5
    4. Question

    4. Consider the following statements regarding the International Organization of Supreme Audit Institutions (INTOSAI):
    1. It has a special consultative status with the Economic and Social Council (ECOSOC) of the United Nations.
    2. Its governing board meets annually to provide strategic leadership, stewardship, and continuity of INTOSAI activities.
    3. The CAG of India is a member of the Governing Board of the INTOSAI.
    How many of the statements given above are correct?

    Correct

    Answer: C
    Context: CAG of India inaugurated the International Organization of Supreme Audit Institutions’s (INTOSAI) 15th Knowledge Sharing Committee meeting at Abu Dhabi.
    Explanation:
    Statement 1 is correct: It has special consultative status with the Economic and Social Council (ECOSOC) of the United Nations.
    Statement 2 is correct: The INTOSAI Governing Board meets annually to provide strategic leadership, stewardship, and continuity of INTOSAI activities.
    Statement 3 is correct: The CAG of India is a member of the Governing Board of the INTOSAI.
    Additional information:
    ● The International Organization of Supreme Audit Institutions (INTOSAI) is an intergovernmental organization whose members are supreme audit institutions.
    ● Nearly every supreme audit institution in the world is a member of INTOSAI. Depending on the type of system used in their home country, the members of INTOSAI may be variously titled the Chief Financial Controller, the Office of the Comptroller General, the Office of the Auditor General, the Court of Accounts, or the Board of Audit.
    ● INTOSAI holds a triennial conference entitled the International Congress of Supreme Audit Institutions (INCOSAI).
    ● It publishes the quarterly International Journal of Government Auditing and publishes guidelines and international standards on auditing.

    Incorrect

    Answer: C
    Context: CAG of India inaugurated the International Organization of Supreme Audit Institutions’s (INTOSAI) 15th Knowledge Sharing Committee meeting at Abu Dhabi.
    Explanation:
    Statement 1 is correct: It has special consultative status with the Economic and Social Council (ECOSOC) of the United Nations.
    Statement 2 is correct: The INTOSAI Governing Board meets annually to provide strategic leadership, stewardship, and continuity of INTOSAI activities.
    Statement 3 is correct: The CAG of India is a member of the Governing Board of the INTOSAI.
    Additional information:
    ● The International Organization of Supreme Audit Institutions (INTOSAI) is an intergovernmental organization whose members are supreme audit institutions.
    ● Nearly every supreme audit institution in the world is a member of INTOSAI. Depending on the type of system used in their home country, the members of INTOSAI may be variously titled the Chief Financial Controller, the Office of the Comptroller General, the Office of the Auditor General, the Court of Accounts, or the Board of Audit.
    ● INTOSAI holds a triennial conference entitled the International Congress of Supreme Audit Institutions (INCOSAI).
    ● It publishes the quarterly International Journal of Government Auditing and publishes guidelines and international standards on auditing.

  5. Question 5 of 5
    5. Question

    5. Consider the following statements:
    1. India has notified a greater number of critical minerals than the USA to drive its clean energy push.
    2. The Geological Survey of India has been given the responsibility to periodically update the list of critical minerals for India.
    Which of the statements given above is/are correct?

    Correct

    Answer: D
    Context: The Indian government recently listed 30 minerals, including nickel, titanium, vanadium and tungsten as critical to drive its clean energy push.
    Explanation:
    Statement 1 is incorrect: India has notified 30 critical minerals. The US has declared 50 minerals critical in light of their role in national security or economic development. Japan has identified a set of 31 minerals as critical for their economy. The UK considers 18 minerals critical, EU (34) and Canada (31).
    Statement 2 is incorrect: Creation of Centre of Excellence for Critical Minerals (CECM) in the Ministry of Mines is recommended by the Expert Committee of Ministry of Mines. CECM has the responsibility to periodically update the list of critical minerals for India and notify the critical mineral strategy from time to time.
    Statement 3 is correct:
    Additional information:
    ● Expert Committe under Ministry of Mines has identified a set of 30 critical minerals for India.
    ● These are Antimony, Beryllium, Bismuth, Cobalt, Copper, Gallium, Germanium, Graphite, Hafnium, Indium, Lithium, Molybdenum, Niobium, Nickel, PGE, Phosphorous, Potash, REE, Rhenium, Silicon, Strontium, Tantalum, Tellurium, Tin, Titanium, Tungsten, Vanadium, Zirconium, Selenium and Cadmium.
    ● Top Producers: Chile, Indonesia, Congo, China, Australia and South Africa.

    Incorrect

    Answer: D
    Context: The Indian government recently listed 30 minerals, including nickel, titanium, vanadium and tungsten as critical to drive its clean energy push.
    Explanation:
    Statement 1 is incorrect: India has notified 30 critical minerals. The US has declared 50 minerals critical in light of their role in national security or economic development. Japan has identified a set of 31 minerals as critical for their economy. The UK considers 18 minerals critical, EU (34) and Canada (31).
    Statement 2 is incorrect: Creation of Centre of Excellence for Critical Minerals (CECM) in the Ministry of Mines is recommended by the Expert Committee of Ministry of Mines. CECM has the responsibility to periodically update the list of critical minerals for India and notify the critical mineral strategy from time to time.
    Statement 3 is correct:
    Additional information:
    ● Expert Committe under Ministry of Mines has identified a set of 30 critical minerals for India.
    ● These are Antimony, Beryllium, Bismuth, Cobalt, Copper, Gallium, Germanium, Graphite, Hafnium, Indium, Lithium, Molybdenum, Niobium, Nickel, PGE, Phosphorous, Potash, REE, Rhenium, Silicon, Strontium, Tantalum, Tellurium, Tin, Titanium, Tungsten, Vanadium, Zirconium, Selenium and Cadmium.
    ● Top Producers: Chile, Indonesia, Congo, China, Australia and South Africa.

window.wpAdvQuizInitList = window.wpAdvQuizInitList || []; window.wpAdvQuizInitList.push({ id: '#wpAdvQuiz_596', init: { quizId: 596, mode: 0, globalPoints: 10, timelimit: 0, resultsGrade: [0], bo: 0, qpp: 0, catPoints: [10], formPos: 0, lbn: "Finish quiz", json: {"2774":{"type":"single","id":2774,"catId":0,"points":2,"correct":[1,0,0,0]},"2775":{"type":"single","id":2775,"catId":0,"points":2,"correct":[0,1,0,0]},"2776":{"type":"single","id":2776,"catId":0,"points":2,"correct":[0,0,1,0]},"2777":{"type":"single","id":2777,"catId":0,"points":2,"correct":[0,0,1,0]},"2778":{"type":"single","id":2778,"catId":0,"points":2,"correct":[0,0,0,1]}} } });




Day-556 | Daily MCQs | UPSC Prelims | ECONOMICS

Day-556

Time limit: 0

Quiz-summary

0 of 5 questions completed

Questions:

  1. 1
  2. 2
  3. 3
  4. 4
  5. 5

Information

DAILY MCQ

You have already completed the quiz before. Hence you can not start it again.

Quiz is loading...

You must sign in or sign up to start the quiz.

You have to finish following quiz, to start this quiz:

Results

0 of 5 questions answered correctly

Your time:

Time has elapsed

You have reached 0 of 0 points, (0)

Categories

  1. Not categorized 0%
  1. 1
  2. 2
  3. 3
  4. 4
  5. 5
  1. Answered
  2. Review
  1. Question 1 of 5
    1. Question

    1. In the context of economics, which of the following statements correctly describes the ‘Lewis Model’?

    Correct

    Answer: A
    Explanation:
    • In 1954, economist William Arthur Lewis put forth the “Economic Development with Unlimited Supplies of Labor”. For this work Lewis won the Nobel Prize in Economics in 1979.
    • The crux of the model suggested that surplus labor in agriculture could be redirected to the manufacturing sector by offering wages just high enough to attract workers away from the farm.
    • This shift, in theory, would stimulate industrial growth, enhance productivity, and lead to economic development.
    Challenges in Implementation of Lewis Model in India:
    • Obstacles of Low Wages: Low wages and inadequate social security in urban manufacturing facilities fails to entice rural agricultural laborers to relocate, given the high costs of urban living, posing a hurdle to the implementation of the Lewis model.
    • Technological Shift in Manufacturing: Manufacturing industries are increasingly capital-intensive, relying on labor-displacing technologies like robotics and artificial intelligence.
    • This transition restricts the absorptive capacity of labor-intensive sectors to accommodate surplus agricultural workers.
    • Disguised Unemployment: India faces a scenario of disguised unemployment in the agricultural sector, where a surplus of workers is engaged in activities that do not significantly contribute to increased productivity or income.
    • This surplus labor situation complicates the transition of workers to other sectors.
    • Skill Mismatch: There exists a mismatch between the skills demanded by the industries and the skills possessed by the workforce.
    • The education system might not adequately prepare individuals for the demands of the modern job market, resulting in a skill gap that impedes labor absorption in industries.
    • Overemphasis on White-Collar Jobs: Societal perceptions often prioritize white-collar jobs over vocational or technical skills.
    • This bias against blue-collar work can limit the workforce available for skilled trade positions and technical jobs, affecting industrial growth.

    Incorrect

    Answer: A
    Explanation:
    • In 1954, economist William Arthur Lewis put forth the “Economic Development with Unlimited Supplies of Labor”. For this work Lewis won the Nobel Prize in Economics in 1979.
    • The crux of the model suggested that surplus labor in agriculture could be redirected to the manufacturing sector by offering wages just high enough to attract workers away from the farm.
    • This shift, in theory, would stimulate industrial growth, enhance productivity, and lead to economic development.
    Challenges in Implementation of Lewis Model in India:
    • Obstacles of Low Wages: Low wages and inadequate social security in urban manufacturing facilities fails to entice rural agricultural laborers to relocate, given the high costs of urban living, posing a hurdle to the implementation of the Lewis model.
    • Technological Shift in Manufacturing: Manufacturing industries are increasingly capital-intensive, relying on labor-displacing technologies like robotics and artificial intelligence.
    • This transition restricts the absorptive capacity of labor-intensive sectors to accommodate surplus agricultural workers.
    • Disguised Unemployment: India faces a scenario of disguised unemployment in the agricultural sector, where a surplus of workers is engaged in activities that do not significantly contribute to increased productivity or income.
    • This surplus labor situation complicates the transition of workers to other sectors.
    • Skill Mismatch: There exists a mismatch between the skills demanded by the industries and the skills possessed by the workforce.
    • The education system might not adequately prepare individuals for the demands of the modern job market, resulting in a skill gap that impedes labor absorption in industries.
    • Overemphasis on White-Collar Jobs: Societal perceptions often prioritize white-collar jobs over vocational or technical skills.
    • This bias against blue-collar work can limit the workforce available for skilled trade positions and technical jobs, affecting industrial growth.

  2. Question 2 of 5
    2. Question

    2. Consider the following statements:
    1. Mutual funds invest money in equities only.
    2. Mutual funds can reduce the volatility in return on investment.
    3. Infrastructure Investment Trust (InvIT) and Real estate Investment Trust (ReITs) are similar to mutual funds.
    How many of the statements given above are correct?

    Correct

    Answer: B
    Explanation:
    Statement 1 is incorrect: Mutual funds collect money from investors and invest the money, on their behalf, in different securities (debt, equity or both).
    Statement 2 is correct: Equity mutual funds pool money from multiple investors to create a diversified portfolio of stocks. Due to risk diversification, the volatility in returns generated from mutual funds investments are generally lower than that of direct equity investing.
    Statement 3 is correct: Infrastructure investment Trust (InvIt) and Real estate Investment Trust(ReITs) are similar to mutual funds. This is because both REITs and InvITs pool money from a large number of investors and invest money on physical assets to generate income which is distributed as dividend.
    Additional information:
    ● An equity mutual fund is a type of mutual fund that primarily invests in stocks or equities. It allows individual investors to pool their money together and invest in a diversified portfolio of stocks selected and managed by professional fund managers.
    ● The objective of equity mutual funds is to provide investors with the opportunity to participate in the potential returns and dividends generated by a wide range of publicly traded companies.
    ● Equity funds are actively managed by experienced fund managers who conduct research and analysis to make investment decisions. These professionals aim to select stocks that they believe will generate positive returns for investors.

    Incorrect

    Answer: B
    Explanation:
    Statement 1 is incorrect: Mutual funds collect money from investors and invest the money, on their behalf, in different securities (debt, equity or both).
    Statement 2 is correct: Equity mutual funds pool money from multiple investors to create a diversified portfolio of stocks. Due to risk diversification, the volatility in returns generated from mutual funds investments are generally lower than that of direct equity investing.
    Statement 3 is correct: Infrastructure investment Trust (InvIt) and Real estate Investment Trust(ReITs) are similar to mutual funds. This is because both REITs and InvITs pool money from a large number of investors and invest money on physical assets to generate income which is distributed as dividend.
    Additional information:
    ● An equity mutual fund is a type of mutual fund that primarily invests in stocks or equities. It allows individual investors to pool their money together and invest in a diversified portfolio of stocks selected and managed by professional fund managers.
    ● The objective of equity mutual funds is to provide investors with the opportunity to participate in the potential returns and dividends generated by a wide range of publicly traded companies.
    ● Equity funds are actively managed by experienced fund managers who conduct research and analysis to make investment decisions. These professionals aim to select stocks that they believe will generate positive returns for investors.

  3. Question 3 of 5
    3. Question

    3. With reference to various curves in economics, consider the following pairs:
    Economics curves – Relationships between variables
    1. Laffer Curve – Inflation rate and Unemployment rate
    2. Phillips Curve – Tax revenue and Tax rate
    3. Lorenz Curve – National Income and Population Distribution
    How many of the above pairs are correctly matched?

    Correct

    Answer: A
    Explanation
    • Pair 1 is incorrect: Laffer curve explains the relationship between tax revenue and tax rate. It states that at lower as well as higher rate of tax, the tax revenue is low but tax revenue is high at the optimal rate of tax. According to the Laffer curve, if tax rates are increased above a certain level, then tax revenues can actually fall because higher tax rates discourage people from working, also there is high tax evasion.
    • Pair 2 is incorrect: The Phillips curve is an economic theory that inflation and unemployment have a stable and inverse relationship. Developed by William Phillips, it claims that with economic growth comes inflation, which in turn should lead to more jobs and less unemployment.
    • Pair 3 is correct: Lorenz Curve is a graphical distribution of wealth. It shows the proportion of income earned by any given percentage of the population.

    Incorrect

    Answer: A
    Explanation
    • Pair 1 is incorrect: Laffer curve explains the relationship between tax revenue and tax rate. It states that at lower as well as higher rate of tax, the tax revenue is low but tax revenue is high at the optimal rate of tax. According to the Laffer curve, if tax rates are increased above a certain level, then tax revenues can actually fall because higher tax rates discourage people from working, also there is high tax evasion.
    • Pair 2 is incorrect: The Phillips curve is an economic theory that inflation and unemployment have a stable and inverse relationship. Developed by William Phillips, it claims that with economic growth comes inflation, which in turn should lead to more jobs and less unemployment.
    • Pair 3 is correct: Lorenz Curve is a graphical distribution of wealth. It shows the proportion of income earned by any given percentage of the population.

  4. Question 4 of 5
    4. Question

    4. Consider the following statements regarding Gross Domestic Product (GDP)and Gross Value Added (GVA):
    1. GDP gives the picture of an economy from the demand perspective while GVA reflects the state of economic activity from the supply side.
    2. GDP includes the value of intermediate goods whereas GVA does not.
    Which of the statements given above is/are correct?

    Correct

    Answer: A
    Explanation:
    • Statement 1 is correct: GVA is the value added to the product to enhance the various aspects of the product whereas GDP is the total amount of products produced in the country. While GDP gives the picture from the consumers’ side or demand perspective, GVA gives a picture of the state of economic activity from the producers’ side or supply side. Both measures need not match because of the difference in treatment of net taxes.
    • Statement 2 is incorrect: Both GDP as well as GVA do not include the value of intermediate goods. GDP is calculated by adding Net Taxes (taxes earned by the government – subsidies paid by the government) to the GVA. GDP fails to gauge the real economic scenario because a sharp increase in the output can be due to higher tax collections which could be on account of better compliance or coverage, rather than the real output situation. As a result, GVA is a more precise measure of the value added by each sector of the economy.
    Additional information:
    ● GDP is the sum of private consumption, gross investment in the economy, government investment, government spending and net foreign trade (the difference between exports and imports).
    ● GDP = private consumption + gross investment + government investment + government spending + (exports-imports)
    ● GDP measures the value of a country’s final goods and services (those purchased by the final user) generated in a specific time period (say, a quarter or a year). It includes all of the output produced within a country’s borders.
    GDP = Private consumption + gross investment + government investment + government spending + (exports – imports)
    ● Private Consumption Expenditure refers to the value of all goods and services purchased for consumption by households.
    ● Government Consumption Expenditure refers to the value of all goods and services purchased for consumption by the government.
    ● Gross Investment refers to the total value of all capital investments made in the economy.
    The GVA of a sector is defined as the value of output minus the value of its intermediary inputs. This “value added” is distributed among the primary production factors, labour and capital. By examining GVA growth, one may determine which sectors of the economy are doing well and which are struggling.
    Calculation of GVA
    ● GVA is the sum of a country’s GDP and net of subsidies and taxes in the economy at the macro level, according to national accounting.
    ● Gross Value Added = GDP + product subsidies – product taxes
    ● Previously, India measured GVA at ‘factor cost’ until a new methodology was implemented, in which GVA at ‘basic prices’ became the primary measure of economic output.
    ● GVA at basic prices will include production taxes and exclude production subsidies.
    ● GVA at factor cost included no taxes and excluded no subsidies

    Incorrect

    Answer: A
    Explanation:
    • Statement 1 is correct: GVA is the value added to the product to enhance the various aspects of the product whereas GDP is the total amount of products produced in the country. While GDP gives the picture from the consumers’ side or demand perspective, GVA gives a picture of the state of economic activity from the producers’ side or supply side. Both measures need not match because of the difference in treatment of net taxes.
    • Statement 2 is incorrect: Both GDP as well as GVA do not include the value of intermediate goods. GDP is calculated by adding Net Taxes (taxes earned by the government – subsidies paid by the government) to the GVA. GDP fails to gauge the real economic scenario because a sharp increase in the output can be due to higher tax collections which could be on account of better compliance or coverage, rather than the real output situation. As a result, GVA is a more precise measure of the value added by each sector of the economy.
    Additional information:
    ● GDP is the sum of private consumption, gross investment in the economy, government investment, government spending and net foreign trade (the difference between exports and imports).
    ● GDP = private consumption + gross investment + government investment + government spending + (exports-imports)
    ● GDP measures the value of a country’s final goods and services (those purchased by the final user) generated in a specific time period (say, a quarter or a year). It includes all of the output produced within a country’s borders.
    GDP = Private consumption + gross investment + government investment + government spending + (exports – imports)
    ● Private Consumption Expenditure refers to the value of all goods and services purchased for consumption by households.
    ● Government Consumption Expenditure refers to the value of all goods and services purchased for consumption by the government.
    ● Gross Investment refers to the total value of all capital investments made in the economy.
    The GVA of a sector is defined as the value of output minus the value of its intermediary inputs. This “value added” is distributed among the primary production factors, labour and capital. By examining GVA growth, one may determine which sectors of the economy are doing well and which are struggling.
    Calculation of GVA
    ● GVA is the sum of a country’s GDP and net of subsidies and taxes in the economy at the macro level, according to national accounting.
    ● Gross Value Added = GDP + product subsidies – product taxes
    ● Previously, India measured GVA at ‘factor cost’ until a new methodology was implemented, in which GVA at ‘basic prices’ became the primary measure of economic output.
    ● GVA at basic prices will include production taxes and exclude production subsidies.
    ● GVA at factor cost included no taxes and excluded no subsidies

  5. Question 5 of 5
    5. Question

    5. Consider the following statements regarding Nominal Effective Exchange Rate (NEER) and Real Effective Exchange Rate (REER):
    1. A Nominal Effective Exchange Rate (NEER) adjusted for inflation equals its Real Effective Exchange Rate (REER).
    2. An increase in a nation’s REER is an indication that its exports are becoming more expensive and its imports are becoming cheaper.
    Which of the statements given above is/are correct?

    Correct

    Answer: C
    Explanation:
    Statement 1 is correct: A Nominal Effective Exchange Rate (NEER) adjusted for inflation equals its Real Effective Exchange Rate (REER). The important aspect of NEER is that it is not adjusted for inflation. Thus, it does not say anything about the real strength of a currency; it is just a relative value. REER is considered a more accurate measure to gauge a currency’s strength. The Real Effective Exchange Rate (REER) is the NEER adjusted by inflation differentials between the domestic currency and that of the trading partners. Thus, it says about the real strength of a domestic currency against a basket of currencies, unlike NEER.
    Statement 2 is correct: An increase in a nation’s REER is an indication that its exports are becoming more expensive and its imports are becoming cheaper.
    Additional information:
    ● The Real Effective Exchange Rate (REER) is the weighted average of a country’s currency in relation to an index or basket of other major currencies.
    ● The weights are determined by comparing the relative trade balance of a country’s currency against that of each country in the index.
    ● The Nominal Effective Exchange Rate (NEER) is an index of the weighted average of bilateral exchange rates of home currency with respect to a basket of currencies of trading partners. It is also known as the trade-weighted currency index.
    ● An increase in NEER indicates that the domestic currency has appreciated against the basket of currencies whereas a decrease indicates a relative depreciation.
    NEER and REER in the Indian context
    ● The Reserve Bank compiles and disseminates indices of NEER and REER of the Indian rupee.
    ● The latest series was constructed with the base year 2015-16. The new indices of NEER and REER replaced the old series from 2004-05 onwards.
    ● The coverage of the NEER and REER basket has been expanded from 36 to 40 currencies. This was in view of the growing importance of emerging markets and developing economies in India’s foreign trade and to better reflect shifts in external competitiveness.
    ● The selection of currencies for NEER and REER in the new series is based on two criteria:
    ● Trading partners with extremely high and volatile inflation are excluded. This is because their currencies tend to experience rapid nominal declines, undermining the stability of the NEER and REER indices. This obscures their usefulness in the assessment of external competitiveness.
    ● Data on inflation and exchange rates of trading partners should be available on a regular basis.

    Incorrect

    Answer: C
    Explanation:
    Statement 1 is correct: A Nominal Effective Exchange Rate (NEER) adjusted for inflation equals its Real Effective Exchange Rate (REER). The important aspect of NEER is that it is not adjusted for inflation. Thus, it does not say anything about the real strength of a currency; it is just a relative value. REER is considered a more accurate measure to gauge a currency’s strength. The Real Effective Exchange Rate (REER) is the NEER adjusted by inflation differentials between the domestic currency and that of the trading partners. Thus, it says about the real strength of a domestic currency against a basket of currencies, unlike NEER.
    Statement 2 is correct: An increase in a nation’s REER is an indication that its exports are becoming more expensive and its imports are becoming cheaper.
    Additional information:
    ● The Real Effective Exchange Rate (REER) is the weighted average of a country’s currency in relation to an index or basket of other major currencies.
    ● The weights are determined by comparing the relative trade balance of a country’s currency against that of each country in the index.
    ● The Nominal Effective Exchange Rate (NEER) is an index of the weighted average of bilateral exchange rates of home currency with respect to a basket of currencies of trading partners. It is also known as the trade-weighted currency index.
    ● An increase in NEER indicates that the domestic currency has appreciated against the basket of currencies whereas a decrease indicates a relative depreciation.
    NEER and REER in the Indian context
    ● The Reserve Bank compiles and disseminates indices of NEER and REER of the Indian rupee.
    ● The latest series was constructed with the base year 2015-16. The new indices of NEER and REER replaced the old series from 2004-05 onwards.
    ● The coverage of the NEER and REER basket has been expanded from 36 to 40 currencies. This was in view of the growing importance of emerging markets and developing economies in India’s foreign trade and to better reflect shifts in external competitiveness.
    ● The selection of currencies for NEER and REER in the new series is based on two criteria:
    ● Trading partners with extremely high and volatile inflation are excluded. This is because their currencies tend to experience rapid nominal declines, undermining the stability of the NEER and REER indices. This obscures their usefulness in the assessment of external competitiveness.
    ● Data on inflation and exchange rates of trading partners should be available on a regular basis.

window.wpAdvQuizInitList = window.wpAdvQuizInitList || []; window.wpAdvQuizInitList.push({ id: '#wpAdvQuiz_595', init: { quizId: 595, mode: 0, globalPoints: 10, timelimit: 0, resultsGrade: [0], bo: 0, qpp: 0, catPoints: [10], formPos: 0, lbn: "Finish quiz", json: {"2769":{"type":"single","id":2769,"catId":0,"points":2,"correct":[1,0,0,0]},"2770":{"type":"single","id":2770,"catId":0,"points":2,"correct":[0,1,0,0]},"2771":{"type":"single","id":2771,"catId":0,"points":2,"correct":[1,0,0,0]},"2772":{"type":"single","id":2772,"catId":0,"points":2,"correct":[1,0,0,0]},"2773":{"type":"single","id":2773,"catId":0,"points":2,"correct":[0,0,1,0]}} } });




Day-555 | Daily MCQs | UPSC Prelims | SCIENCE AND TECHNOLOGY

Day-555

Time limit: 0

Quiz-summary

0 of 5 questions completed

Questions:

  1. 1
  2. 2
  3. 3
  4. 4
  5. 5

Information

DAILY MCQ

You have already completed the quiz before. Hence you can not start it again.

Quiz is loading...

You must sign in or sign up to start the quiz.

You have to finish following quiz, to start this quiz:

Results

0 of 5 questions answered correctly

Your time:

Time has elapsed

You have reached 0 of 0 points, (0)

Categories

  1. Not categorized 0%
  1. 1
  2. 2
  3. 3
  4. 4
  5. 5
  1. Answered
  2. Review
  1. Question 1 of 5
    1. Question

    1. With reference to the birth control pills, consider the following mechanisms:
    1. Stopping or reducing ovulation
    2. Thinning of the cervical mucus
    3. Thickening of uterus lining
    Birth control pills can target how many of the above mechanisms to prevent pregnancy in females?

    Correct

    Answer: A
    Explanation
    The birth control pill is a type of oral contraception that uses hormones to prevent pregnancy. The hormones in birth control pills prevent pregnancy by blocking conception when sperm fertilizes an egg. They also cause changes in the uterus so that it can’t support a pregnancy. Oral contraceptive pills are either combined estrogen-progesterone (also called combined oral contraceptive pill- COC) or progesterone-only pill (POP).
    ● Mechanism 1 is correct: One of the mechanisms targeted by oral contraceptives is to stop ovulation. Ovulation is the release of eggs from the ovaries of females. Thus, there is no egg for fertilization which prevents pregnancy.
    ● Mechanism 2 is incorrect: Thickening of cervical mucus is another mechanism. This creates a barrier that prevents sperm from entering the uterus and eventually reaching the egg.
    ● Mechanism 3 is incorrect: Thinning of uterus lining is also achieved through oral contraceptive pills. This prevents fertilized eggs from getting attached and growing into a fetus.

    Incorrect

    Answer: A
    Explanation
    The birth control pill is a type of oral contraception that uses hormones to prevent pregnancy. The hormones in birth control pills prevent pregnancy by blocking conception when sperm fertilizes an egg. They also cause changes in the uterus so that it can’t support a pregnancy. Oral contraceptive pills are either combined estrogen-progesterone (also called combined oral contraceptive pill- COC) or progesterone-only pill (POP).
    ● Mechanism 1 is correct: One of the mechanisms targeted by oral contraceptives is to stop ovulation. Ovulation is the release of eggs from the ovaries of females. Thus, there is no egg for fertilization which prevents pregnancy.
    ● Mechanism 2 is incorrect: Thickening of cervical mucus is another mechanism. This creates a barrier that prevents sperm from entering the uterus and eventually reaching the egg.
    ● Mechanism 3 is incorrect: Thinning of uterus lining is also achieved through oral contraceptive pills. This prevents fertilized eggs from getting attached and growing into a fetus.

  2. Question 2 of 5
    2. Question

    2. With reference to the Worldcoin project, consider the following statements:
    1. It aims to create one of the world’s largest cryptocurrency networks for digital finance.
    2. Fingerprint scan will be used for authentication and signup purposes on the network.
    3. It will be open to everyone regardless of his/her country.
    How many of the above statements are correct?

    Correct

    Answer: B
    Explanation
    A project called Worldcoin has been launched by OpenAI, an Artificial Intelligence company. The project claims to be building the world’s largest identity and financial public network.
    ● Statement 1 is correct: The project aims to introduce Worldcoin token as a new cryptocurrency that offers a new and unique method for identification to reduce the risk of fraud. Worldcoin token (WLD) can be used as a medium of purchases and transfers globally using digital assets and traditional currencies.
    ● Statement 2 is incorrect: Iris scan (eye) is essential to sign up to the network. It will be done through a ball-like object called an ‘orb’. Once the orb’s iris scan verifies the person is a real human, it creates a World ID for them. The biometric data would help differentiate between humans and Artificial Intelligence systems and prevent duplication of IDs from the same person. It can then be used as an ID in a variety of everyday applications – such as a cryptocurrency wallet – without revealing the user’s identity.
    ● Statement 3 is correct: The Worldcoin protocol is intended to be the world’s largest identity and financial public network, open to everyone regardless of their country, background, or economic status.

    Incorrect

    Answer: B
    Explanation
    A project called Worldcoin has been launched by OpenAI, an Artificial Intelligence company. The project claims to be building the world’s largest identity and financial public network.
    ● Statement 1 is correct: The project aims to introduce Worldcoin token as a new cryptocurrency that offers a new and unique method for identification to reduce the risk of fraud. Worldcoin token (WLD) can be used as a medium of purchases and transfers globally using digital assets and traditional currencies.
    ● Statement 2 is incorrect: Iris scan (eye) is essential to sign up to the network. It will be done through a ball-like object called an ‘orb’. Once the orb’s iris scan verifies the person is a real human, it creates a World ID for them. The biometric data would help differentiate between humans and Artificial Intelligence systems and prevent duplication of IDs from the same person. It can then be used as an ID in a variety of everyday applications – such as a cryptocurrency wallet – without revealing the user’s identity.
    ● Statement 3 is correct: The Worldcoin protocol is intended to be the world’s largest identity and financial public network, open to everyone regardless of their country, background, or economic status.

  3. Question 3 of 5
    3. Question

    3. With reference to the ‘Formation water’, consider the following statements:
    1. It appears during the mining of coal from underground.
    2. It has a very high salt content.
    3. It cannot be treated by any means.
    How many of the above statements are correct?

    Correct

    Answer: A
    Explanation
    ● Statement 1 is incorrect: Formation water is water that appears during the drilling process for oil and gas extraction. It is very dense and viscous.
    ● Statement 2 is correct: The Formation water consists of oily components, brine solutions, and solvents that are used during various phases in the oil industry. It has a high salt content which includes sodium, potassium, calcium, and magnesium salts.
    ● Statement 3 is incorrect: There are ways and means to treat formation water. It depends on the usage of the treated product water.

    Treatment of the formation water
    The process can be summed up in four stages:
    1. Separation: removing crude, oil and other contaminants in the water that can clog filters downstream.
    2. Suspended Solid Removal: eliminating small size particles remaining.
    3. Dissolved solid removal: using low- and high-pressure membranes.
    4. Conditioning: depending on the quality of the water it may be a post treatment station for disinfection or activated carbon filtration.
    More about formation water
    ● If formation water is released without treatment, it can lead to the degradation of water quality, posing a significant threat to aquatic life.
    ● The toxins in formation water can also bio-magnify into the human food chain, if fish and other aquatic products, harvested from the affected area, are consumed.

    Incorrect

    Answer: A
    Explanation
    ● Statement 1 is incorrect: Formation water is water that appears during the drilling process for oil and gas extraction. It is very dense and viscous.
    ● Statement 2 is correct: The Formation water consists of oily components, brine solutions, and solvents that are used during various phases in the oil industry. It has a high salt content which includes sodium, potassium, calcium, and magnesium salts.
    ● Statement 3 is incorrect: There are ways and means to treat formation water. It depends on the usage of the treated product water.

    Treatment of the formation water
    The process can be summed up in four stages:
    1. Separation: removing crude, oil and other contaminants in the water that can clog filters downstream.
    2. Suspended Solid Removal: eliminating small size particles remaining.
    3. Dissolved solid removal: using low- and high-pressure membranes.
    4. Conditioning: depending on the quality of the water it may be a post treatment station for disinfection or activated carbon filtration.
    More about formation water
    ● If formation water is released without treatment, it can lead to the degradation of water quality, posing a significant threat to aquatic life.
    ● The toxins in formation water can also bio-magnify into the human food chain, if fish and other aquatic products, harvested from the affected area, are consumed.

  4. Question 4 of 5
    4. Question

    4. The ‘Turing Test’ is related to which of the following scientific developments?

    Correct

    Answer: B
    Explanation
    ● The Turing Test is used in artificial intelligence (AI) for determining whether a computer is capable of thinking like a human being. The test is named after Alan Turing, the founder of the Turing Test and an English computer scientist, cryptanalyst, mathematician, and theoretical biologist.
    ● The original Turing Test requires three terminals, each of which is physically separated from the other two. One terminal is operated by a computer, while the other two are operated by humans.
    ● During the test, one of the human being functions as the questioner, while the second human and the computer function as respondents. The questioner interrogates the respondents within a specific subject area, using a specified format and context. After a preset length of time or number of questions, the questioner is then asked to decide which respondent was human and which was a computer.

    Incorrect

    Answer: B
    Explanation
    ● The Turing Test is used in artificial intelligence (AI) for determining whether a computer is capable of thinking like a human being. The test is named after Alan Turing, the founder of the Turing Test and an English computer scientist, cryptanalyst, mathematician, and theoretical biologist.
    ● The original Turing Test requires three terminals, each of which is physically separated from the other two. One terminal is operated by a computer, while the other two are operated by humans.
    ● During the test, one of the human being functions as the questioner, while the second human and the computer function as respondents. The questioner interrogates the respondents within a specific subject area, using a specified format and context. After a preset length of time or number of questions, the questioner is then asked to decide which respondent was human and which was a computer.

  5. Question 5 of 5
    5. Question

    5. Meissner Effect is related to which of the following technologies?

    Correct

    Answer: B
    Explanation
    ● Meissner effect is the expulsion of a magnetic field from the interior of a material that is in the process of becoming a superconductor.

    ● A super conductor loses its resistance to the flow of electrical currents when cooled below a certain temperature called the critical temperature. It is close to absolute zero or zero kelvin.
    ● The Meissner Effect is used in magnetic levitation, which means a body is suspended with no support except a magnetic field. Modern high-speed bullet trains use the phenomenon of magnetic levitation.

    Incorrect

    Answer: B
    Explanation
    ● Meissner effect is the expulsion of a magnetic field from the interior of a material that is in the process of becoming a superconductor.

    ● A super conductor loses its resistance to the flow of electrical currents when cooled below a certain temperature called the critical temperature. It is close to absolute zero or zero kelvin.
    ● The Meissner Effect is used in magnetic levitation, which means a body is suspended with no support except a magnetic field. Modern high-speed bullet trains use the phenomenon of magnetic levitation.

window.wpAdvQuizInitList = window.wpAdvQuizInitList || []; window.wpAdvQuizInitList.push({ id: '#wpAdvQuiz_594', init: { quizId: 594, mode: 0, globalPoints: 10, timelimit: 0, resultsGrade: [0], bo: 0, qpp: 0, catPoints: [10], formPos: 0, lbn: "Finish quiz", json: {"2764":{"type":"single","id":2764,"catId":0,"points":2,"correct":[1,0,0,0]},"2765":{"type":"single","id":2765,"catId":0,"points":2,"correct":[0,1,0,0]},"2766":{"type":"single","id":2766,"catId":0,"points":2,"correct":[1,0,0,0]},"2767":{"type":"single","id":2767,"catId":0,"points":2,"correct":[0,1,0,0]},"2768":{"type":"single","id":2768,"catId":0,"points":2,"correct":[0,1,0,0]}} } });




Day-554 | Daily MCQs | UPSC Prelims | POLITY

Day-554

Time limit: 0

Quiz-summary

0 of 5 questions completed

Questions:

  1. 1
  2. 2
  3. 3
  4. 4
  5. 5

Information

DAILY MCQ

You have already completed the quiz before. Hence you can not start it again.

Quiz is loading...

You must sign in or sign up to start the quiz.

You have to finish following quiz, to start this quiz:

Results

0 of 5 questions answered correctly

Your time:

Time has elapsed

You have reached 0 of 0 points, (0)

Categories

  1. Not categorized 0%
  1. 1
  2. 2
  3. 3
  4. 4
  5. 5
  1. Answered
  2. Review
  1. Question 1 of 5
    1. Question

    1. Consider the following:
    1. Consolidated Fund of India
    2. Contingency Fund of India
    3. Public Account of India
    The Union Government Finance Accounts depicts the receipts and payments from how many of the funds mentioned above?

    Correct

    Answer: C
    Explanation:
    Statement 1 is correct: The Annual Accounts of the Union Government provide a comprehensive overview of the government’s financial transactions for a given fiscal year. These audit of these accounts are presented to the Parliament by the President of India while the report is prepared by the Comptroller and Auditor General (CAG) of India, typically within six months of the conclusion of the fiscal year. The Annual Accounts serve as a crucial tool for ensuring transparency and accountability in the government’s financial management.
    Components of the Annual Accounts: It consists of two main components:
    1. Finance Accounts: The Finance Accounts provide a detailed record of all receipts and payments made by the government during the fiscal year. These accounts include information on revenue collection, expenditure incurred; from the Consolidated Fund of India, Contingency Fund and Public Account; and the overall financial position of the government.(Hence Option C is correct)
    2. Appropriation Accounts: The Appropriation Accounts compare the actual expenditure incurred by the government with the amounts authorized by the Parliament through the annual budget. These accounts highlight any variances between the budgeted and actual expenditure and provide explanations for these variations.

    Incorrect

    Answer: C
    Explanation:
    Statement 1 is correct: The Annual Accounts of the Union Government provide a comprehensive overview of the government’s financial transactions for a given fiscal year. These audit of these accounts are presented to the Parliament by the President of India while the report is prepared by the Comptroller and Auditor General (CAG) of India, typically within six months of the conclusion of the fiscal year. The Annual Accounts serve as a crucial tool for ensuring transparency and accountability in the government’s financial management.
    Components of the Annual Accounts: It consists of two main components:
    1. Finance Accounts: The Finance Accounts provide a detailed record of all receipts and payments made by the government during the fiscal year. These accounts include information on revenue collection, expenditure incurred; from the Consolidated Fund of India, Contingency Fund and Public Account; and the overall financial position of the government.(Hence Option C is correct)
    2. Appropriation Accounts: The Appropriation Accounts compare the actual expenditure incurred by the government with the amounts authorized by the Parliament through the annual budget. These accounts highlight any variances between the budgeted and actual expenditure and provide explanations for these variations.

  2. Question 2 of 5
    2. Question

    2. Consider the following:
    1. Equality before the law
    2. Transparency and predictability
    3. Civic duty
    4. Independence of the judiciary
    How many of the given above are considered as the important features of “Rule of Law” in the Indian context?

    Correct

    Answer: C
    Explanation:
    Option 1 is correct: The rule of law implies that everyone, regardless of their status or position, is subject to the law and must abide by it. This means that no one is above the law and that everyone is treated equally under the law.The principle of equality before the law is essential for ensuring fairness and justice in society. It prevents discrimination and ensures that everyone has equal access to the legal system. When the law is applied equally, it creates a sense of trust and legitimacy among the people, fostering a more stable and harmonious society.
    Option 2 is correct: The rule of law requires that the law be clear, transparent, and predictable. This means that the law should be written in a way that is easy to understand and that it should be applied consistently and impartially.When the law is clear and predictable, it reduces uncertainty and promotes compliance. It also allows individuals to hold the government accountable for its actions and to challenge decisions that they believe violate their rights.
    Option 3 is incorrect: Civic duties are the responsibilities that citizens have to their communities and their country. These duties are often enshrined in law, but they can also be based on tradition, custom, or moral values. In India, there are a number of civic duties that are considered to be important for all citizens. These duties include:
    ● Obeying the law: This is the most basic civic duty, and it is essential for maintaining order and stability in society. Citizens should obey all laws, even those they disagree with, and they should report any violations to the authorities.
    ● Voting: Voting is another fundamental civic duty. It allows citizens to participate in the democratic process and to choose their representatives. Citizens should register to vote and participate in all elections.
    ● Paying taxes: Taxes are an important source of revenue for the government, and they are used to fund a variety of public services, such as education, healthcare, and infrastructure. Citizens should pay their taxes on time and in full.
    ● Protecting the environment: The environment is a vital resource that we must protect for future generations. Citizens should take steps to conserve resources, reduce pollution, and recycle.
    In addition to these general civic duties, there are also a number of specific civic duties that are enshrined in the Indian Constitution. These duties include:
    ● Abide by the Constitution and respect its ideals and institutions, the National Flag and the National Anthem: This duty requires citizens to uphold the values and principles of the Constitution and to respect the symbols of national unity.
    ● Cherish and follow the noble ideals which inspired our national struggle for freedom: This duty requires citizens to appreciate the sacrifices made by those who fought for India’s independence and to strive to live up to their ideals.
    ● Uphold and protect the sovereignty, unity, and integrity of India: This duty requires citizens to defend India’s borders, to promote unity among its people, and to protect its national interests.
    Option 4 is correct: The rule of law requires that the judiciary be independent of the executive and legislative branches of government. This means that the judiciary should be able to make decisions without fear of political interference or intimidation.An independent judiciary is essential for upholding the rule of law and protecting individual rights. When the judiciary is free from political influence, it can ensure that the law is applied fairly and impartially. It can also protect individuals from arbitrary or unjust actions by the government.
    Additional Information:
    Rule of Law vs. Due Process of Law
    While the rule of law and due process of law are related concepts, they are not the same. The rule of law is a broader principle that encompasses the entire legal system, including the laws themselves, the institutions that enforce them, and the values that underpin them.
    Due process of law, on the other hand, is a specific legal concept that refers to the procedures that must be followed when the government takes action against an individual.
    In essence, the rule of law is the foundation upon which due process of law is built. The rule of law ensures that the legal system is fair, just, and impartial, while due process of law ensures that individuals are treated fairly and have their rights protected when the government takes action against them.

    Incorrect

    Answer: C
    Explanation:
    Option 1 is correct: The rule of law implies that everyone, regardless of their status or position, is subject to the law and must abide by it. This means that no one is above the law and that everyone is treated equally under the law.The principle of equality before the law is essential for ensuring fairness and justice in society. It prevents discrimination and ensures that everyone has equal access to the legal system. When the law is applied equally, it creates a sense of trust and legitimacy among the people, fostering a more stable and harmonious society.
    Option 2 is correct: The rule of law requires that the law be clear, transparent, and predictable. This means that the law should be written in a way that is easy to understand and that it should be applied consistently and impartially.When the law is clear and predictable, it reduces uncertainty and promotes compliance. It also allows individuals to hold the government accountable for its actions and to challenge decisions that they believe violate their rights.
    Option 3 is incorrect: Civic duties are the responsibilities that citizens have to their communities and their country. These duties are often enshrined in law, but they can also be based on tradition, custom, or moral values. In India, there are a number of civic duties that are considered to be important for all citizens. These duties include:
    ● Obeying the law: This is the most basic civic duty, and it is essential for maintaining order and stability in society. Citizens should obey all laws, even those they disagree with, and they should report any violations to the authorities.
    ● Voting: Voting is another fundamental civic duty. It allows citizens to participate in the democratic process and to choose their representatives. Citizens should register to vote and participate in all elections.
    ● Paying taxes: Taxes are an important source of revenue for the government, and they are used to fund a variety of public services, such as education, healthcare, and infrastructure. Citizens should pay their taxes on time and in full.
    ● Protecting the environment: The environment is a vital resource that we must protect for future generations. Citizens should take steps to conserve resources, reduce pollution, and recycle.
    In addition to these general civic duties, there are also a number of specific civic duties that are enshrined in the Indian Constitution. These duties include:
    ● Abide by the Constitution and respect its ideals and institutions, the National Flag and the National Anthem: This duty requires citizens to uphold the values and principles of the Constitution and to respect the symbols of national unity.
    ● Cherish and follow the noble ideals which inspired our national struggle for freedom: This duty requires citizens to appreciate the sacrifices made by those who fought for India’s independence and to strive to live up to their ideals.
    ● Uphold and protect the sovereignty, unity, and integrity of India: This duty requires citizens to defend India’s borders, to promote unity among its people, and to protect its national interests.
    Option 4 is correct: The rule of law requires that the judiciary be independent of the executive and legislative branches of government. This means that the judiciary should be able to make decisions without fear of political interference or intimidation.An independent judiciary is essential for upholding the rule of law and protecting individual rights. When the judiciary is free from political influence, it can ensure that the law is applied fairly and impartially. It can also protect individuals from arbitrary or unjust actions by the government.
    Additional Information:
    Rule of Law vs. Due Process of Law
    While the rule of law and due process of law are related concepts, they are not the same. The rule of law is a broader principle that encompasses the entire legal system, including the laws themselves, the institutions that enforce them, and the values that underpin them.
    Due process of law, on the other hand, is a specific legal concept that refers to the procedures that must be followed when the government takes action against an individual.
    In essence, the rule of law is the foundation upon which due process of law is built. The rule of law ensures that the legal system is fair, just, and impartial, while due process of law ensures that individuals are treated fairly and have their rights protected when the government takes action against them.

  3. Question 3 of 5
    3. Question

    3. Consider the following statements about the Mayoral System in India:
    1. Mayoral system in India is not explicitly mentioned in the Constitution of India.
    2. The Mayor is chosen through indirect election by the councilors from among themselves in all the states.
    3. It aims to decentralize the power from the state government to the municipal level
    How many of the above given statements are correct?

    Correct

    Answer: B
    Explanation:
    Statement 1 is correct:The Constitution of India does not explicitly mandate the mayoral system, but it allows states to adopt it through their own legislation. Several Indian states have implemented the mayoral system, including Andhra Pradesh, Chhattisgarh, Jharkhand, Madhya Pradesh, Odisha, Tamil Nadu, and Telangana.The mayor is merely a ceremonial authority, and executive decisions are carried out by the municipal commissioner appointed by the state government.
    Statement 2 is incorrect:The Mayor in the Municipal Corporation is usually chosen through indirect election by the councilors from among themselves. However, few states including Uttarakhand, Chhattisgarh, Jharkhand, Madhya Pradesh, Uttar Pradesh, and Tamil Nadu – have mayors who are elected directly. This inconsistency is because of the lack of clarity about the Mayoral system in the Constitution of India. Also the 74th amendment did not prescribe the manner of election, tenure, or powers of the Mayors/Chairpersons of Urban Local Bodies.
    Statement 3 is correct: The mayoral system aims to decentralize power from the state government to the municipal level, empowering local bodies to make decisions that directly impact their citizens.The mayoral system seeks to bring governance closer to the people, giving them a direct say in matters that affect their daily lives. This decentralization of power enhances local accountability and encourages greater citizen participation in the decision-making process.

    Roles, Functions, and Powers of the Mayor
    The specific roles, functions, and powers of the mayor vary from state to state. However, in general, the mayor is responsible for:
    ● Providing leadership and direction to the municipal administration
    ● Presiding over the meetings of the municipal corporation
    ● Overseeing the implementation of municipal policies and programs
    ● Representing the municipality in official engagements
    ● Ensuring the financial well-being of the municipality
    ● Promoting the interests of the city and its citizens
    The exact extent of the mayor’s powers depends on the specific provisions of the state legislation governing the mayoral system.

    Incorrect

    Answer: B
    Explanation:
    Statement 1 is correct:The Constitution of India does not explicitly mandate the mayoral system, but it allows states to adopt it through their own legislation. Several Indian states have implemented the mayoral system, including Andhra Pradesh, Chhattisgarh, Jharkhand, Madhya Pradesh, Odisha, Tamil Nadu, and Telangana.The mayor is merely a ceremonial authority, and executive decisions are carried out by the municipal commissioner appointed by the state government.
    Statement 2 is incorrect:The Mayor in the Municipal Corporation is usually chosen through indirect election by the councilors from among themselves. However, few states including Uttarakhand, Chhattisgarh, Jharkhand, Madhya Pradesh, Uttar Pradesh, and Tamil Nadu – have mayors who are elected directly. This inconsistency is because of the lack of clarity about the Mayoral system in the Constitution of India. Also the 74th amendment did not prescribe the manner of election, tenure, or powers of the Mayors/Chairpersons of Urban Local Bodies.
    Statement 3 is correct: The mayoral system aims to decentralize power from the state government to the municipal level, empowering local bodies to make decisions that directly impact their citizens.The mayoral system seeks to bring governance closer to the people, giving them a direct say in matters that affect their daily lives. This decentralization of power enhances local accountability and encourages greater citizen participation in the decision-making process.

    Roles, Functions, and Powers of the Mayor
    The specific roles, functions, and powers of the mayor vary from state to state. However, in general, the mayor is responsible for:
    ● Providing leadership and direction to the municipal administration
    ● Presiding over the meetings of the municipal corporation
    ● Overseeing the implementation of municipal policies and programs
    ● Representing the municipality in official engagements
    ● Ensuring the financial well-being of the municipality
    ● Promoting the interests of the city and its citizens
    The exact extent of the mayor’s powers depends on the specific provisions of the state legislation governing the mayoral system.

  4. Question 4 of 5
    4. Question

    4. “Appointment of Governors shall be made on the recommendation of a committee comprising the Prime Minister, the Home Minister, the Lok Sabha Speaker and the concerned Chief Minister of the State”.
    Which of the following commissions made the recommendation given above?

    Correct

    Answer: C
    Explanation: The correct answer is MM Punchhi Commission. Among other recommendations, the recommendations with respect to the appointment and removal of the Governor of the State include:
    ● There should be a say of the state’s Chief Minister while making the Governor’s appointment.
    ● A committee should be formed that is entrusted with the task of appointment of governors. This committee may comprise the Prime Minister, the Home Minister, the Lok Sabha’s speaker and the concerned Chief Minister of the State.
    ● The term of appointment should be five years.
    ● Governor could only be removed via a resolution by the State Legislature.
    Rajamannar Committee (1971) stressed that the Governor of the state should not consider himself as an agent of the centre but play his role as the constitutional head of the State.
    Sarkaria Commission:
    ● The Constitution itself should lay down the process for consulting the chief minister when appointing a state governor.
    ● As long as the council of ministers has a majority in the assembly, the governor cannot dismiss it.
    ● Unless there are some exceptionally compelling circumstances, a governor’s five-year governorship should not be disrupted.

    Incorrect

    Answer: C
    Explanation: The correct answer is MM Punchhi Commission. Among other recommendations, the recommendations with respect to the appointment and removal of the Governor of the State include:
    ● There should be a say of the state’s Chief Minister while making the Governor’s appointment.
    ● A committee should be formed that is entrusted with the task of appointment of governors. This committee may comprise the Prime Minister, the Home Minister, the Lok Sabha’s speaker and the concerned Chief Minister of the State.
    ● The term of appointment should be five years.
    ● Governor could only be removed via a resolution by the State Legislature.
    Rajamannar Committee (1971) stressed that the Governor of the state should not consider himself as an agent of the centre but play his role as the constitutional head of the State.
    Sarkaria Commission:
    ● The Constitution itself should lay down the process for consulting the chief minister when appointing a state governor.
    ● As long as the council of ministers has a majority in the assembly, the governor cannot dismiss it.
    ● Unless there are some exceptionally compelling circumstances, a governor’s five-year governorship should not be disrupted.

  5. Question 5 of 5
    5. Question

    5. The Simon Commission was appointed by the British government to report on the working of which of the following?

    Correct

    Answer: D
    Explanation:
    Government of India Act, 1919 provided for the appointment of the statutory commission to inquire into and report on its working after ten years of coming into force.
    The Government of India Act 1919 was the codified version of the Montague-Chelmsford reforms – named after Edwin Charles Montague and Lord Chelmsford, who held positions of the Secretary of State and Viceroy of British India respectively. The Act was sold to the Indians as ‘a step in the progressive realisation of responsible government in India as an integral part of the empire’.
    The persistent demand for further reforms led the British government to appoint Simon Commission in 1927 and report on the working of the Indian Constitution as established under Government of India Act, 1919.

    Incorrect

    Answer: D
    Explanation:
    Government of India Act, 1919 provided for the appointment of the statutory commission to inquire into and report on its working after ten years of coming into force.
    The Government of India Act 1919 was the codified version of the Montague-Chelmsford reforms – named after Edwin Charles Montague and Lord Chelmsford, who held positions of the Secretary of State and Viceroy of British India respectively. The Act was sold to the Indians as ‘a step in the progressive realisation of responsible government in India as an integral part of the empire’.
    The persistent demand for further reforms led the British government to appoint Simon Commission in 1927 and report on the working of the Indian Constitution as established under Government of India Act, 1919.

window.wpAdvQuizInitList = window.wpAdvQuizInitList || []; window.wpAdvQuizInitList.push({ id: '#wpAdvQuiz_593', init: { quizId: 593, mode: 0, globalPoints: 10, timelimit: 0, resultsGrade: [0], bo: 0, qpp: 0, catPoints: [10], formPos: 0, lbn: "Finish quiz", json: {"2759":{"type":"single","id":2759,"catId":0,"points":2,"correct":[0,0,1,0]},"2760":{"type":"single","id":2760,"catId":0,"points":2,"correct":[0,0,1,0]},"2761":{"type":"single","id":2761,"catId":0,"points":2,"correct":[0,1,0,0]},"2762":{"type":"single","id":2762,"catId":0,"points":2,"correct":[0,0,1,0]},"2763":{"type":"single","id":2763,"catId":0,"points":2,"correct":[0,0,0,1]}} } });




Day-553 | Daily MCQs | UPSC Prelims | GEOGRAPHY

Day-553

Time limit: 0

Quiz-summary

0 of 5 questions completed

Questions:

  1. 1
  2. 2
  3. 3
  4. 4
  5. 5

Information

DAILY MCQ

You have already completed the quiz before. Hence you can not start it again.

Quiz is loading...

You must sign in or sign up to start the quiz.

You have to finish following quiz, to start this quiz:

Results

0 of 5 questions answered correctly

Your time:

Time has elapsed

You have reached 0 of 0 points, (0)

Categories

  1. Not categorized 0%
  1. 1
  2. 2
  3. 3
  4. 4
  5. 5
  1. Answered
  2. Review
  1. Question 1 of 5
    1. Question

    1. With reference to temperature inversion, consider the following statements:
    1. In temperature inversion, temperature decreases with increase in elevation.
    2. Temperature inversion at surface level promotes instability in lower layers of atmosphere.
    Which of the statements given above is/are correct?

    Correct

    Answer: D
    Explanation:
    Statement 1 is incorrect:
    ● Normal lapse rate (NLR): When temperature decreases with increase in elevation, it is called NLR. But sometimes, the situation is reversed and the normal lapse rate is inverted. It is called Inversion of temperature i.e. increase in temperature with increase in elevation.
    ● Inversion is usually of short duration but quite common nonetheless.
    Normal conditions required:
    ● A long winter night with clear skies and still air is ideal situation for inversion.
    ● The heat of the day is radiated off during the night, and by early morning hours, the earth is cooler than the air above.
    ● Example – over polar areas, temperature inversion is normal throughout the year.
    Statement 2 is incorrect:
    ● Surface inversion promotes stability in the lower layers of the atmosphere.
    ● Smoke and dust particles get collected beneath the inversion layer and spread horizontally to fill the lower strata of the atmosphere.
    ● Dense fogs in mornings are common occurrences especially during winter season.
    ● This inversion commonly lasts for few hours until the sun comes up and beings to warm the earth.
    Additional Information
    Temperature inversion on hilly regions:
    ● The inversion takes place in hills and mountains due to air drainage.
    ● Cold air at the hills and mountains, produced during night, flows under the influence of gravity.
    ● Being heavy and dense, the cold air acts almost like water and moves down the slope to pile up deeply in pockets and valley bottoms with warm air above. This is called air drainage.
    ● It protects plants from frost damages.

    Incorrect

    Answer: D
    Explanation:
    Statement 1 is incorrect:
    ● Normal lapse rate (NLR): When temperature decreases with increase in elevation, it is called NLR. But sometimes, the situation is reversed and the normal lapse rate is inverted. It is called Inversion of temperature i.e. increase in temperature with increase in elevation.
    ● Inversion is usually of short duration but quite common nonetheless.
    Normal conditions required:
    ● A long winter night with clear skies and still air is ideal situation for inversion.
    ● The heat of the day is radiated off during the night, and by early morning hours, the earth is cooler than the air above.
    ● Example – over polar areas, temperature inversion is normal throughout the year.
    Statement 2 is incorrect:
    ● Surface inversion promotes stability in the lower layers of the atmosphere.
    ● Smoke and dust particles get collected beneath the inversion layer and spread horizontally to fill the lower strata of the atmosphere.
    ● Dense fogs in mornings are common occurrences especially during winter season.
    ● This inversion commonly lasts for few hours until the sun comes up and beings to warm the earth.
    Additional Information
    Temperature inversion on hilly regions:
    ● The inversion takes place in hills and mountains due to air drainage.
    ● Cold air at the hills and mountains, produced during night, flows under the influence of gravity.
    ● Being heavy and dense, the cold air acts almost like water and moves down the slope to pile up deeply in pockets and valley bottoms with warm air above. This is called air drainage.
    ● It protects plants from frost damages.

  2. Question 2 of 5
    2. Question

    2. Consider the following statements:
    Statement I: The North Sea records higher salinity despite being located at higher latitude.
    Statement II: Ocean currents are major determinants of salinity of oceans/seas.
    Which one of the following is correct in respect of the above statements?

    Correct

    Answer: A
    Explanation
    Both the Statements are correct and Statement II is correct explanation of Statement I:
    ● The North Sea, in spite of its location in higher latitudes, records higher salinity due to more saline water brought by the North Atlantic Drift. While, Baltic Sea records low salinity due to influx of river waters in large quantity.
    Horizontal Distribution of Salinity:
    • The horizontal distribution of salinity in oceans and seas is influence by several factors.
    These include:
    ● The proximity to major river mouths, where freshwater input can lower salinity levels. Example – Salinity is very low in Black Sea due to enormous fresh water influx by rivers.
    ● Evaporation rates, which can increase salinity. Example – The Mediterranean Sea records higher salinity due to high evaporation.
    ● Ocean currents also play a role in distributing salinity horizontally, as they transport water masses with different salinity levels across different regions.
    Other factors may be:
    ● Precipitation
    ● Influx of river water
    ● Atmospheric pressure and wind direction
    ● Global warming

    Incorrect

    Answer: A
    Explanation
    Both the Statements are correct and Statement II is correct explanation of Statement I:
    ● The North Sea, in spite of its location in higher latitudes, records higher salinity due to more saline water brought by the North Atlantic Drift. While, Baltic Sea records low salinity due to influx of river waters in large quantity.
    Horizontal Distribution of Salinity:
    • The horizontal distribution of salinity in oceans and seas is influence by several factors.
    These include:
    ● The proximity to major river mouths, where freshwater input can lower salinity levels. Example – Salinity is very low in Black Sea due to enormous fresh water influx by rivers.
    ● Evaporation rates, which can increase salinity. Example – The Mediterranean Sea records higher salinity due to high evaporation.
    ● Ocean currents also play a role in distributing salinity horizontally, as they transport water masses with different salinity levels across different regions.
    Other factors may be:
    ● Precipitation
    ● Influx of river water
    ● Atmospheric pressure and wind direction
    ● Global warming

  3. Question 3 of 5
    3. Question

    3. Consider the following pairs:
    Cropping Seasons – Crops
    1. Kharif Season – Gram
    2. Rabi Season – Cotton
    3. Zaid Season – Cucumber
    How many of the pairs given above are correctly matched?

    Correct

    Answer: A
    Explanation:
    Pair 1 is not correctly matched:
    ● There are three distinct crop seasons in the northern and interior parts of the country, namely Kharif, Rabi and Zaid.
    ● The Kharif season largely coincides with Southwest Monsoon under which the cultivation of tropical crops, such as rice, cotton, jute, jowar, bajra, and tur is possible.
    Pair 2 is not correctly matched:
    ● The rabi season begins with the onset of winter in October-November and ends in March-April.
    ● The low-temperature conditions during this season facilitate the cultivation of temperate and subtropical crops such as wheat, gram, and mustard.
    Pair 3 is correctly matched:
    ● Zaid is a short duration summer cropping season beginning after harvesting of rabi crops.
    ● The cultivation of watermelons, cucumbers, vegetables and fodder crops during this season is done on irrigated lands.
    However, this type of distinction in the cropping season does not exist in the southern parts of the country. Here, the temperature is high enough to grow tropical crops during any period in the year provided the soil moisture is available. Therefore, in this region same crops can be grown thrice in an agricultural year provided there is sufficient soil moisture.

    Incorrect

    Answer: A
    Explanation:
    Pair 1 is not correctly matched:
    ● There are three distinct crop seasons in the northern and interior parts of the country, namely Kharif, Rabi and Zaid.
    ● The Kharif season largely coincides with Southwest Monsoon under which the cultivation of tropical crops, such as rice, cotton, jute, jowar, bajra, and tur is possible.
    Pair 2 is not correctly matched:
    ● The rabi season begins with the onset of winter in October-November and ends in March-April.
    ● The low-temperature conditions during this season facilitate the cultivation of temperate and subtropical crops such as wheat, gram, and mustard.
    Pair 3 is correctly matched:
    ● Zaid is a short duration summer cropping season beginning after harvesting of rabi crops.
    ● The cultivation of watermelons, cucumbers, vegetables and fodder crops during this season is done on irrigated lands.
    However, this type of distinction in the cropping season does not exist in the southern parts of the country. Here, the temperature is high enough to grow tropical crops during any period in the year provided the soil moisture is available. Therefore, in this region same crops can be grown thrice in an agricultural year provided there is sufficient soil moisture.

  4. Question 4 of 5
    4. Question

    4. According to the Koeppen’s scheme of classification of Indian climatic regions, ‘As’ type of climatic region refers to:

    Correct

    Answer: D
    Explanation:
    ● Koeppen’s scheme of classification of Indian climatic regions divided India into 8 major climatic regions.
    ● Coromandel coast of Tamil Nadu has been categorised as a ‘Monsoon with dry summer’ climatic region i.e. ‘As’ type of climate.
    ● Major climatic region according to Koeppen’s climatic classification in India are as given in the table below:

    Additional Information
    ● A climatic region has a homogeneous climatic condition which is the result of a combination of factors. Temperature and rainfall are two important elements which are considered to be decisive in all the schemes of climatic classification. The classification of climate, however, is a complex exercise. There are different schemes of classification of climate.
    ● Major climatic types of India are based on Koeppen’s scheme.
    Koeppen’s scheme
    ● Koeppen based his scheme of Climatic classification on monthly values of temperature and precipitation.
    ● He identified five major climatic types, namely:
    I. Tropical climates, where mean monthly temperature throughout the year is over 18°C.
    II. Dry climates, where precipitation is very low in comparison to temperature, and hence, dry. If dryness is less, it is semi-arid (S); if it is more, the climate is arid (W).
    III. Warm temperate climates, where mean temperature of the coldest month is between 18°C and minus 3°C.
    IV. Cool temperate climates, where mean temperature of the warmest month is over 10°C, and mean temperature of the coldest month is under minus 3°C.
    V. Ice climates, where mean temperature of the warmest month is under 10°C.
    ● Koeppen used letter symbols to denote climatic types as given above.
    ● Each type is further sub-divided into sub-types on the basis of seasonal variations in the distributional pattern of rainfall and temperature.
    ● He used S for semi-arid and W for arid and the following small letters to define sub-types:
    ⮚ f (sufficient precipitation)
    ⮚ m (rainforest despite a dry monsoon season)
    ⮚ w (dryseason in winter)
    ⮚ h (dry and hot)
    ⮚ c (less than four months with mean temperature over 10°C)
    ⮚ g (Gangetic plain).
    ● Accordingly, India can be divided into nine climatic regions (including steppe climate – Bshw).
    ● India’s climatic regions are shown in the following map.

    Incorrect

    Answer: D
    Explanation:
    ● Koeppen’s scheme of classification of Indian climatic regions divided India into 8 major climatic regions.
    ● Coromandel coast of Tamil Nadu has been categorised as a ‘Monsoon with dry summer’ climatic region i.e. ‘As’ type of climate.
    ● Major climatic region according to Koeppen’s climatic classification in India are as given in the table below:

    Additional Information
    ● A climatic region has a homogeneous climatic condition which is the result of a combination of factors. Temperature and rainfall are two important elements which are considered to be decisive in all the schemes of climatic classification. The classification of climate, however, is a complex exercise. There are different schemes of classification of climate.
    ● Major climatic types of India are based on Koeppen’s scheme.
    Koeppen’s scheme
    ● Koeppen based his scheme of Climatic classification on monthly values of temperature and precipitation.
    ● He identified five major climatic types, namely:
    I. Tropical climates, where mean monthly temperature throughout the year is over 18°C.
    II. Dry climates, where precipitation is very low in comparison to temperature, and hence, dry. If dryness is less, it is semi-arid (S); if it is more, the climate is arid (W).
    III. Warm temperate climates, where mean temperature of the coldest month is between 18°C and minus 3°C.
    IV. Cool temperate climates, where mean temperature of the warmest month is over 10°C, and mean temperature of the coldest month is under minus 3°C.
    V. Ice climates, where mean temperature of the warmest month is under 10°C.
    ● Koeppen used letter symbols to denote climatic types as given above.
    ● Each type is further sub-divided into sub-types on the basis of seasonal variations in the distributional pattern of rainfall and temperature.
    ● He used S for semi-arid and W for arid and the following small letters to define sub-types:
    ⮚ f (sufficient precipitation)
    ⮚ m (rainforest despite a dry monsoon season)
    ⮚ w (dryseason in winter)
    ⮚ h (dry and hot)
    ⮚ c (less than four months with mean temperature over 10°C)
    ⮚ g (Gangetic plain).
    ● Accordingly, India can be divided into nine climatic regions (including steppe climate – Bshw).
    ● India’s climatic regions are shown in the following map.

  5. Question 5 of 5
    5. Question

    5. With reference to the various types of droughts, consider the following statements:
    1. Meteorological drought is a situation when the productivity of a natural ecosystem fails due to shortage of water and as a consequence of ecological distress; damages are induced in the ecosystem.
    2. Ecological drought is a situation when there is a prolonged period of inadequate rainfall marked with mal-distribution of the same over time and space.
    3. Agricultural drought, also known as soil moisture drought, is characterised by low soil moisture that is necessary to support the crops, thereby resulting in crop failures.
    4. Hydrological drought is a situation when the availability of water in different storages and reservoirs like aquifers, lakes, reservoirs, etc. falls below what the precipitation can replenish.
    How many of the statements given above are correct?

    Correct

    Answer: B
    Explanation:
    Statement 1 is incorrect:
    ● Meteorological drought: It is a situation when there is a prolonged period of inadequate rainfall marked with mal-distribution of the same over time and space.
    Statement 2 is incorrect:
    ● Ecological drought: When the productivity of a natural ecosystem fails due to shortage of water and as a consequence of ecological distress, damages are induced in the ecosystem.
    Statement 3 is correct:
    ● Agricultural drought: It is also known as soil moisture drought, characterised by low soil moisture that is necessary to support the crops, thereby resulting in crop failures. Moreover, if an area has more than 30 percent of its gross cropped area under irrigation, the area is excluded from the drought-prone category.
    Statement 4 is correct:
    ● Hydrological drought: It results when the availability of water in different storages and reservoirs like aquifers, lakes, reservoirs, etc. falls below what the precipitation can replenish.

    Incorrect

    Answer: B
    Explanation:
    Statement 1 is incorrect:
    ● Meteorological drought: It is a situation when there is a prolonged period of inadequate rainfall marked with mal-distribution of the same over time and space.
    Statement 2 is incorrect:
    ● Ecological drought: When the productivity of a natural ecosystem fails due to shortage of water and as a consequence of ecological distress, damages are induced in the ecosystem.
    Statement 3 is correct:
    ● Agricultural drought: It is also known as soil moisture drought, characterised by low soil moisture that is necessary to support the crops, thereby resulting in crop failures. Moreover, if an area has more than 30 percent of its gross cropped area under irrigation, the area is excluded from the drought-prone category.
    Statement 4 is correct:
    ● Hydrological drought: It results when the availability of water in different storages and reservoirs like aquifers, lakes, reservoirs, etc. falls below what the precipitation can replenish.

window.wpAdvQuizInitList = window.wpAdvQuizInitList || []; window.wpAdvQuizInitList.push({ id: '#wpAdvQuiz_592', init: { quizId: 592, mode: 0, globalPoints: 10, timelimit: 0, resultsGrade: [0], bo: 0, qpp: 0, catPoints: [10], formPos: 0, lbn: "Finish quiz", json: {"2754":{"type":"single","id":2754,"catId":0,"points":2,"correct":[0,0,0,1]},"2755":{"type":"single","id":2755,"catId":0,"points":2,"correct":[1,0,0,0]},"2756":{"type":"single","id":2756,"catId":0,"points":2,"correct":[1,0,0,0]},"2757":{"type":"single","id":2757,"catId":0,"points":2,"correct":[0,0,0,1]},"2758":{"type":"single","id":2758,"catId":0,"points":2,"correct":[0,1,0,0]}} } });




Day-552 | Daily MCQs | UPSC Prelims | HISTORY

14-12-2023

Time limit: 0

Quiz-summary

0 of 5 questions completed

Questions:

  1. 1
  2. 2
  3. 3
  4. 4
  5. 5

Information

DAILY MCQS

You have already completed the quiz before. Hence you can not start it again.

Quiz is loading...

You must sign in or sign up to start the quiz.

You have to finish following quiz, to start this quiz:

Results

0 of 5 questions answered correctly

Your time:

Time has elapsed

You have reached 0 of 0 points, (0)

Categories

  1. GS Prelims 0%
  1. 1
  2. 2
  3. 3
  4. 4
  5. 5
  1. Answered
  2. Review
  1. Question 1 of 5
    1. Question

    1. With reference to the history of education in India, consider the following statements:

    1. William Jones who was appointed as judge of the Supreme Court took the initiative to set up Calcutta Madrasa.
    2. The English Education Act of 1835, introduced English as the medium of instruction at all levels of learning.

    Which of the statements given above is/are correct?

    Correct

    Answer: D
    Explanation:

    The early idea which helped in shaping the policies of education during colonial rule was Orientalism. It focused on studying and understanding the scholarly knowledge of the language and culture of Asia.

    Some of the early personalities who supported this idea were:

    • William Jones who had an appointment as a junior judge at the Supreme Court that the Company had set up at Calcutta. He along with some of his followers like Henry Thomas Colebrooke and Nathaniel Halhed, set up the Asiatic Society of Bengal, and started a journal called Asiatick Researches. So, statement 1 is incorrect.
    • Warren Hastings took the initiative to set up the Calcutta Madrasa in 1781 to promote the study of Arabic, Persian and Islamic law and believed that the ancient customs of the country and Oriental learning ought to be the basis of British rule in India.
    • From the early nineteenth century many British officials began to criticise the Orientalist vision of learning. The British effort ought to be to teach what was useful and practical. So Indians should be made familiar with the scientific and technical advances that the West had made, rather than with the poetry and sacred literature of the Orient.
    • The most influential ideologues of these were James Mill and Thomas Babington Macaulay. With great energy and passion, Macaulay emphasised the need to teach the English language.
    • Following Macaulay’s minute, the English Education Act of 1835 was introduced. The decision was to make English the medium of instruction for higher education, and to stop the promotion of Oriental institutions like the Calcutta Madrasa and Benaras Sanskrit College. So, statement 2 is incorrect.
    Incorrect

    Answer: D
    Explanation:

    The early idea which helped in shaping the policies of education during colonial rule was Orientalism. It focused on studying and understanding the scholarly knowledge of the language and culture of Asia.

    Some of the early personalities who supported this idea were:

    • William Jones who had an appointment as a junior judge at the Supreme Court that the Company had set up at Calcutta. He along with some of his followers like Henry Thomas Colebrooke and Nathaniel Halhed, set up the Asiatic Society of Bengal, and started a journal called Asiatick Researches. So, statement 1 is incorrect.
    • Warren Hastings took the initiative to set up the Calcutta Madrasa in 1781 to promote the study of Arabic, Persian and Islamic law and believed that the ancient customs of the country and Oriental learning ought to be the basis of British rule in India.
    • From the early nineteenth century many British officials began to criticise the Orientalist vision of learning. The British effort ought to be to teach what was useful and practical. So Indians should be made familiar with the scientific and technical advances that the West had made, rather than with the poetry and sacred literature of the Orient.
    • The most influential ideologues of these were James Mill and Thomas Babington Macaulay. With great energy and passion, Macaulay emphasised the need to teach the English language.
    • Following Macaulay’s minute, the English Education Act of 1835 was introduced. The decision was to make English the medium of instruction for higher education, and to stop the promotion of Oriental institutions like the Calcutta Madrasa and Benaras Sanskrit College. So, statement 2 is incorrect.
  2. Question 2 of 5
    2. Question

    2. With reference to the cultural history of India, the term ‘charak-puja’, refers to:

    Correct

    Answer: B

    Explanation:

    • The hook-swinging ritual (known as charak puja in Bengal) was a form of devotion to the female deity Mari-amma. The devotee would be beaten on the back by a priest until the flesh there was numbed, at which point metal hooks would be passed through the flesh. He would then be raised on the gibbet and swung around, taking care not to show any pain and even to shout and laugh.
    • In the vernaculars of the country “hook-swinging” is variously known as charak-puja, pota-puja, khidi-mari, bhokta-puja, chata-parab, soodaloo, and silloo.

    Incorrect

    Answer: B

    Explanation:

    • The hook-swinging ritual (known as charak puja in Bengal) was a form of devotion to the female deity Mari-amma. The devotee would be beaten on the back by a priest until the flesh there was numbed, at which point metal hooks would be passed through the flesh. He would then be raised on the gibbet and swung around, taking care not to show any pain and even to shout and laugh.
    • In the vernaculars of the country “hook-swinging” is variously known as charak-puja, pota-puja, khidi-mari, bhokta-puja, chata-parab, soodaloo, and silloo.

  3. Question 3 of 5
    3. Question

    3. Consider the following statements about Pandita Ramabai:

    1. Her book titled Stripurushtulna is the first known autobiography written by an Indian woman.
    2. She set up a Mission in Khedgaon near Pune in 1898, where widows and poor women were encouraged to become literate and independent.
    3. In one of the remarkable stories written by her, she talks about a place called ‘ladyland’ where women had freedom to study and work.

    How many statements given above are correct?

    Correct

    Answer: A

    Explanation:

    • Statement 1 is incorrect: Stripurushtulna (A Comparison between Women and Men) is authored by Tarabai Shinde from Poona criticising the social differences between men and women.
    • Amar Jiban written by Rashsundari Devi (1800–1890), who was born in West Bengal, is the first known autobiography written by an Indian woman.
    • Statement 2 is correct: Pandita Ramabai (1858–1922) was given the title ‘Pandita’ because she could read and write Sanskrit, a remarkable achievement as women then were not allowed such knowledge. She went on to set up a Mission in Khedgaon near Pune in 1898, where widows and poor women were encouraged not only to become literate but to be independent.
    • Statement 3 is incorrect: Ladyland is a part of a remarkable story written by Rokeya Sakhawat Hossain titled Sultana’s Dream in 1905 to practise her English skills. This story imagined a woman called Sultana who reaches a place called Ladyland. Ladyland is a place where women had the freedom to study, work, and create inventions like controlling rain from the clouds and flying air cars.
    Incorrect

    Answer: A

    Explanation:

    • Statement 1 is incorrect: Stripurushtulna (A Comparison between Women and Men) is authored by Tarabai Shinde from Poona criticising the social differences between men and women.
    • Amar Jiban written by Rashsundari Devi (1800–1890), who was born in West Bengal, is the first known autobiography written by an Indian woman.
    • Statement 2 is correct: Pandita Ramabai (1858–1922) was given the title ‘Pandita’ because she could read and write Sanskrit, a remarkable achievement as women then were not allowed such knowledge. She went on to set up a Mission in Khedgaon near Pune in 1898, where widows and poor women were encouraged not only to become literate but to be independent.
    • Statement 3 is incorrect: Ladyland is a part of a remarkable story written by Rokeya Sakhawat Hossain titled Sultana’s Dream in 1905 to practise her English skills. This story imagined a woman called Sultana who reaches a place called Ladyland. Ladyland is a place where women had the freedom to study, work, and create inventions like controlling rain from the clouds and flying air cars.
  4. Question 4 of 5
    4. Question

    4. Consider the following paragraph:

    He forged a link with the Muslim League, and decided to launch a civil disobedience movement to destroy the Holwell monument that stood in Calcutta as a reminder of a Black hole tragedy which most people believed did never happen and was invented only to tar the memory of Siraj-ud-daula, the last independent ruler of Bengal. But before it could start, he was arrested by the British on 3 July 1940 under the Defence of India Act.

    Which of the following personalities is being referred to in the above paragraph?

    Correct

    Answer: C

    Explanation:

    • When the 2nd World War broke out in Europe, Subhas Chandra Bose was of the view that Indians must utilise this opportunity to take advantage of the empire’s weakest moment to gain Independence.
    • In Bengal, he forged a link with the Muslim League, and decided to launch a civil disobedience movement to destroy the Holwell monument that stood in Calcutta as a reminder of a Black hole tragedy which most people believed did never happen and was invented only to tar the memory of Siraj-ud-daula, the last independent ruler of Bengal. It was a campaign that had an obvious appeal to the Muslims and thus could further strengthen the Hindu-Muslim pact in Bengal. But before it could start, he was arrested by the British on 3 July 1940 under the Defence of India Act.
    • The Holwell Monument was erected by G. Holwell during the short tenure of his Calcutta Governorship in 1760 to commemorate those deceased in the Black Hole tragedy. The monument was a symbol representing the alleged savagery of the last Nawab of Bengal, Siraj-ud-daula, and the bravery of the British soldiers who sacrificed their lives. It was argued, ‘the monument must go because it is not merely an unwarranted stain on the memory of the Nawab but has stood in the heart of Calcutta for the last 150 years or more as the symbol of our slavery and humiliation.’
    Incorrect

    Answer: C

    Explanation:

    • When the 2nd World War broke out in Europe, Subhas Chandra Bose was of the view that Indians must utilise this opportunity to take advantage of the empire’s weakest moment to gain Independence.
    • In Bengal, he forged a link with the Muslim League, and decided to launch a civil disobedience movement to destroy the Holwell monument that stood in Calcutta as a reminder of a Black hole tragedy which most people believed did never happen and was invented only to tar the memory of Siraj-ud-daula, the last independent ruler of Bengal. It was a campaign that had an obvious appeal to the Muslims and thus could further strengthen the Hindu-Muslim pact in Bengal. But before it could start, he was arrested by the British on 3 July 1940 under the Defence of India Act.
    • The Holwell Monument was erected by G. Holwell during the short tenure of his Calcutta Governorship in 1760 to commemorate those deceased in the Black Hole tragedy. The monument was a symbol representing the alleged savagery of the last Nawab of Bengal, Siraj-ud-daula, and the bravery of the British soldiers who sacrificed their lives. It was argued, ‘the monument must go because it is not merely an unwarranted stain on the memory of the Nawab but has stood in the heart of Calcutta for the last 150 years or more as the symbol of our slavery and humiliation.’
  5. Question 5 of 5
    5. Question

    5. With reference to the history of India, the term ‘dalams’ refers to:

    Correct

    Answer: D

    Explanation:

    • The term ‘dalams’ refers to revolutionary units. Mostly women played an active role in these organisations. They joined on their own, acted as couriers of secret messages, arranged shelter and few of them took up guns and became participating members of the dalams. One of the prominent examples of this was the Tebhaga movement which began in Bengal in 1946 under communist-led kisan sabhas with the sharecroppers’ demand for two-thirds share of the produce.
    Incorrect

    Answer: D

    Explanation:

    • The term ‘dalams’ refers to revolutionary units. Mostly women played an active role in these organisations. They joined on their own, acted as couriers of secret messages, arranged shelter and few of them took up guns and became participating members of the dalams. One of the prominent examples of this was the Tebhaga movement which began in Bengal in 1946 under communist-led kisan sabhas with the sharecroppers’ demand for two-thirds share of the produce.
window.wpAdvQuizInitList = window.wpAdvQuizInitList || []; window.wpAdvQuizInitList.push({ id: '#wpAdvQuiz_591', init: { quizId: 591, mode: 0, globalPoints: 10, timelimit: 0, resultsGrade: [0], bo: 0, qpp: 0, catPoints: {"2":10}, formPos: 0, lbn: "Finish quiz", json: {"2749":{"type":"single","id":2749,"catId":2,"points":2,"correct":[0,0,0,1]},"2750":{"type":"single","id":2750,"catId":2,"points":2,"correct":[0,1,0,0]},"2751":{"type":"single","id":2751,"catId":2,"points":2,"correct":[1,0,0,0]},"2752":{"type":"single","id":2752,"catId":2,"points":2,"correct":[0,0,1,0]},"2753":{"type":"single","id":2753,"catId":2,"points":2,"correct":[0,0,0,1]}} } });



Day-551 | Daily MCQs | UPSC Prelims | GENERAL SCIENCE AND TECHNOLOGICAL DEVELOPMENTS

Day-551

Time limit: 0

Quiz-summary

0 of 5 questions completed

Questions:

  1. 1
  2. 2
  3. 3
  4. 4
  5. 5

Information

DAILY MCQ

You have already completed the quiz before. Hence you can not start it again.

Quiz is loading...

You must sign in or sign up to start the quiz.

You have to finish following quiz, to start this quiz:

Results

0 of 5 questions answered correctly

Your time:

Time has elapsed

You have reached 0 of 0 points, (0)

Categories

  1. Not categorized 0%
  1. 1
  2. 2
  3. 3
  4. 4
  5. 5
  1. Answered
  2. Review
  1. Question 1 of 5
    1. Question

    1. With reference to the birth control pills, consider the following mechanisms:
    1. Stopping or reducing ovulation
    2. Thinning of the cervical mucus
    3. Thickening of uterus lining
    Birth control pills can target how many of the above mechanisms to prevent pregnancy in females?

    Correct

    Answer: A
    Explanation
    The birth control pill is a type of oral contraception that uses hormones to prevent pregnancy. The hormones in birth control pills prevent pregnancy by blocking conception when sperm fertilizes an egg. They also cause changes in the uterus so that it can’t support a pregnancy. Oral contraceptive pills are either combined estrogen-progesterone (also called combined oral contraceptive pill- COC) or progesterone-only pill (POP).
    ● Mechanism 1 is correct: One of the mechanisms targeted by oral contraceptives is to stop ovulation. Ovulation is the release of eggs from the ovaries of females. Thus, there is no egg for fertilization which prevents pregnancy.
    ● Mechanism 2 is incorrect: Thickening of cervical mucus is another mechanism. This creates a barrier that prevents sperm from entering the uterus and eventually reaching the egg.
    ● Mechanism 3 is incorrect: Thinning of uterus lining is also achieved through oral contraceptive pills. This prevents fertilized eggs from getting attached and growing into a fetus.

    Incorrect

    Answer: A
    Explanation
    The birth control pill is a type of oral contraception that uses hormones to prevent pregnancy. The hormones in birth control pills prevent pregnancy by blocking conception when sperm fertilizes an egg. They also cause changes in the uterus so that it can’t support a pregnancy. Oral contraceptive pills are either combined estrogen-progesterone (also called combined oral contraceptive pill- COC) or progesterone-only pill (POP).
    ● Mechanism 1 is correct: One of the mechanisms targeted by oral contraceptives is to stop ovulation. Ovulation is the release of eggs from the ovaries of females. Thus, there is no egg for fertilization which prevents pregnancy.
    ● Mechanism 2 is incorrect: Thickening of cervical mucus is another mechanism. This creates a barrier that prevents sperm from entering the uterus and eventually reaching the egg.
    ● Mechanism 3 is incorrect: Thinning of uterus lining is also achieved through oral contraceptive pills. This prevents fertilized eggs from getting attached and growing into a fetus.

  2. Question 2 of 5
    2. Question

    2. With reference to the Worldcoin project, consider the following statements:
    1. It aims to create one of the world’s largest cryptocurrency networks for digital finance.
    2. Fingerprint scan will be used for authentication and signup purposes on the network.
    3. It will be open to everyone regardless of his/her country.
    How many of the above statements are correct?

    Correct

    Answer: B
    Explanation
    A project called Worldcoin has been launched by OpenAI, an Artificial Intelligence company. The project claims to be building the world’s largest identity and financial public network.
    ● Statement 1 is correct: The project aims to introduce Worldcoin token as a new cryptocurrency that offers a new and unique method for identification to reduce the risk of fraud. Worldcoin token (WLD) can be used as a medium of purchases and transfers globally using digital assets and traditional currencies.
    ● Statement 2 is incorrect: Iris scan (eye) is essential to sign up to the network. It will be done through a ball-like object called an ‘orb’. Once the orb’s iris scan verifies the person is a real human, it creates a World ID for them. The biometric data would help differentiate between humans and Artificial Intelligence systems and prevent duplication of IDs from the same person. It can then be used as an ID in a variety of everyday applications – such as a cryptocurrency wallet – without revealing the user’s identity.
    ● Statement 3 is correct: The Worldcoin protocol is intended to be the world’s largest identity and financial public network, open to everyone regardless of their country, background, or economic status.

    Incorrect

    Answer: B
    Explanation
    A project called Worldcoin has been launched by OpenAI, an Artificial Intelligence company. The project claims to be building the world’s largest identity and financial public network.
    ● Statement 1 is correct: The project aims to introduce Worldcoin token as a new cryptocurrency that offers a new and unique method for identification to reduce the risk of fraud. Worldcoin token (WLD) can be used as a medium of purchases and transfers globally using digital assets and traditional currencies.
    ● Statement 2 is incorrect: Iris scan (eye) is essential to sign up to the network. It will be done through a ball-like object called an ‘orb’. Once the orb’s iris scan verifies the person is a real human, it creates a World ID for them. The biometric data would help differentiate between humans and Artificial Intelligence systems and prevent duplication of IDs from the same person. It can then be used as an ID in a variety of everyday applications – such as a cryptocurrency wallet – without revealing the user’s identity.
    ● Statement 3 is correct: The Worldcoin protocol is intended to be the world’s largest identity and financial public network, open to everyone regardless of their country, background, or economic status.

  3. Question 3 of 5
    3. Question

    3. With reference to the ‘Formation water’, consider the following statements:
    1. It appears during the mining of coal from underground.
    2. It has a very high salt content.
    3. It cannot be treated by any means.
    How many of the above statements are correct?

    Correct

    Answer: A
    Explanation
    ● Statement 1 is incorrect: Formation water is water that appears during the drilling process for oil and gas extraction. It is very dense and viscous.
    ● Statement 2 is correct: The Formation water consists of oily components, brine solutions, and solvents that are used during various phases in the oil industry. It has a high salt content which includes sodium, potassium, calcium, and magnesium salts.
    ● Statement 3 is incorrect: There are ways and means to treat formation water. It depends on the usage of the treated product water.
    Treatment of the formation water
    The process can be summed up in four stages:
    1. Separation: removing crude, oil and other contaminants in the water that can clog filters downstream.
    2. Suspended Solid Removal: eliminating small size particles remaining.
    3. Dissolved solid removal: using low- and high-pressure membranes.
    4. Conditioning: depending on the quality of the water it may be a post treatment station for disinfection or activated carbon filtration.
    More about formation water
    ● If formation water is released without treatment, it can lead to the degradation of water quality, posing a significant threat to aquatic life.
    ● The toxins in formation water can also bio-magnify into the human food chain, if fish and other aquatic products, harvested from the affected area, are consumed.

    Incorrect

    Answer: A
    Explanation
    ● Statement 1 is incorrect: Formation water is water that appears during the drilling process for oil and gas extraction. It is very dense and viscous.
    ● Statement 2 is correct: The Formation water consists of oily components, brine solutions, and solvents that are used during various phases in the oil industry. It has a high salt content which includes sodium, potassium, calcium, and magnesium salts.
    ● Statement 3 is incorrect: There are ways and means to treat formation water. It depends on the usage of the treated product water.
    Treatment of the formation water
    The process can be summed up in four stages:
    1. Separation: removing crude, oil and other contaminants in the water that can clog filters downstream.
    2. Suspended Solid Removal: eliminating small size particles remaining.
    3. Dissolved solid removal: using low- and high-pressure membranes.
    4. Conditioning: depending on the quality of the water it may be a post treatment station for disinfection or activated carbon filtration.
    More about formation water
    ● If formation water is released without treatment, it can lead to the degradation of water quality, posing a significant threat to aquatic life.
    ● The toxins in formation water can also bio-magnify into the human food chain, if fish and other aquatic products, harvested from the affected area, are consumed.

  4. Question 4 of 5
    4. Question

    4. The ‘Turing Test’ is related to which of the following scientific developments?

    Correct

    Answer: B
    Explanation
    ● The Turing Test is used in artificial intelligence (AI) for determining whether a computer is capable of thinking like a human being. The test is named after Alan Turing, the founder of the Turing Test and an English computer scientist, cryptanalyst, mathematician, and theoretical biologist.
    ● The original Turing Test requires three terminals, each of which is physically separated from the other two. One terminal is operated by a computer, while the other two are operated by humans.
    ● During the test, one of the human being functions as the questioner, while the second human and the computer function as respondents. The questioner interrogates the respondents within a specific subject area, using a specified format and context. After a preset length of time or number of questions, the questioner is then asked to decide which respondent was human and which was a computer.

    Incorrect

    Answer: B
    Explanation
    ● The Turing Test is used in artificial intelligence (AI) for determining whether a computer is capable of thinking like a human being. The test is named after Alan Turing, the founder of the Turing Test and an English computer scientist, cryptanalyst, mathematician, and theoretical biologist.
    ● The original Turing Test requires three terminals, each of which is physically separated from the other two. One terminal is operated by a computer, while the other two are operated by humans.
    ● During the test, one of the human being functions as the questioner, while the second human and the computer function as respondents. The questioner interrogates the respondents within a specific subject area, using a specified format and context. After a preset length of time or number of questions, the questioner is then asked to decide which respondent was human and which was a computer.

  5. Question 5 of 5
    5. Question

    5. Meissner Effect is related to which of the following technologies?

    Correct

    Answer: B
    Explanation
    ● Meissner effect is the expulsion of a magnetic field from the interior of a material that is in the process of becoming a superconductor.

    ● A super conductor loses its resistance to the flow of electrical currents when cooled below a certain temperature called the critical temperature. It is close to absolute zero or zero kelvin.
    ● The Meissner Effect is used in magnetic levitation, which means a body is suspended with no support except a magnetic field. Modern high-speed bullet trains use the phenomenon of magnetic levitation.

    Incorrect

    Answer: B
    Explanation
    ● Meissner effect is the expulsion of a magnetic field from the interior of a material that is in the process of becoming a superconductor.

    ● A super conductor loses its resistance to the flow of electrical currents when cooled below a certain temperature called the critical temperature. It is close to absolute zero or zero kelvin.
    ● The Meissner Effect is used in magnetic levitation, which means a body is suspended with no support except a magnetic field. Modern high-speed bullet trains use the phenomenon of magnetic levitation.

window.wpAdvQuizInitList = window.wpAdvQuizInitList || []; window.wpAdvQuizInitList.push({ id: '#wpAdvQuiz_590', init: { quizId: 590, mode: 0, globalPoints: 10, timelimit: 0, resultsGrade: [0], bo: 0, qpp: 0, catPoints: [10], formPos: 0, lbn: "Finish quiz", json: {"2744":{"type":"single","id":2744,"catId":0,"points":2,"correct":[1,0,0,0]},"2745":{"type":"single","id":2745,"catId":0,"points":2,"correct":[0,1,0,0]},"2746":{"type":"single","id":2746,"catId":0,"points":2,"correct":[1,0,0,0]},"2747":{"type":"single","id":2747,"catId":0,"points":2,"correct":[0,1,0,0]},"2748":{"type":"single","id":2748,"catId":0,"points":2,"correct":[0,1,0,0]}} } });




Day-550 | Daily MCQs | UPSC Prelims | GENERAL SCIENCE AND TECHNOLOGICAL DEVELOPMENTS

Day-550

Time limit: 0

Quiz-summary

0 of 5 questions completed

Questions:

  1. 1
  2. 2
  3. 3
  4. 4
  5. 5

Information

DAILY MCQ

You have already completed the quiz before. Hence you can not start it again.

Quiz is loading...

You must sign in or sign up to start the quiz.

You have to finish following quiz, to start this quiz:

Results

0 of 5 questions answered correctly

Your time:

Time has elapsed

You have reached 0 of 0 points, (0)

Categories

  1. Not categorized 0%
  1. 1
  2. 2
  3. 3
  4. 4
  5. 5
  1. Answered
  2. Review
  1. Question 1 of 5
    1. Question

    1. With reference to the extraction of oxygen, consider the following statements:
    1. Oxygen for medical purposes is commonly produced through distillation of air.
    2. Oxygen for use in space stations is produced through electrolysis of water.
    Which of the statements given above is/are correct?

    Correct

    Answer: C
    Explanation
    We know that 65% of the human body is oxygen. Oxygen is vital for respiration, the process that transfers energy from glucose to cells. In fact, every cell in our body requires oxygen. When we breathe air in, oxygen molecules enter the lungs and pass through the lung walls into our blood.
    ● Statement 1 is correct: There are various methods through which Liquid Medical Oxygen (LMO) can be produced. The most common production method is the separation of oxygen in Air Separation Units (ASUs). This is also known as Fractional Distillation method.
    Fractional Distillation method
    The various gases constituting atmospheric air are separated into different components after cooling them into a liquid state. Liquid oxygen is then extracted from it.
    Other Method – Pressure Swing Adsorption Technique
    This method leverages the property that under high pressure, gases tend to be attracted to solid surfaces. This process is known as adsorption. The higher the pressure, the more the adsorption of gas.
    ● Statement 2 is correct: Since there is no air in space, water is broken down using electricity in a process known as electrolysis of water. This generates 100% pure oxygen.

    Incorrect

    Answer: C
    Explanation
    We know that 65% of the human body is oxygen. Oxygen is vital for respiration, the process that transfers energy from glucose to cells. In fact, every cell in our body requires oxygen. When we breathe air in, oxygen molecules enter the lungs and pass through the lung walls into our blood.
    ● Statement 1 is correct: There are various methods through which Liquid Medical Oxygen (LMO) can be produced. The most common production method is the separation of oxygen in Air Separation Units (ASUs). This is also known as Fractional Distillation method.
    Fractional Distillation method
    The various gases constituting atmospheric air are separated into different components after cooling them into a liquid state. Liquid oxygen is then extracted from it.
    Other Method – Pressure Swing Adsorption Technique
    This method leverages the property that under high pressure, gases tend to be attracted to solid surfaces. This process is known as adsorption. The higher the pressure, the more the adsorption of gas.
    ● Statement 2 is correct: Since there is no air in space, water is broken down using electricity in a process known as electrolysis of water. This generates 100% pure oxygen.

  2. Question 2 of 5
    2. Question

    2. Consider the following:
    Statement I: A slow motion video camera has a higher frame rate.
    Statement II: Higher frame rate leads to smoother video quality.
    Which one of the following is correct in respect of the above statements?

    Correct

    Answer: A
    Explanation
    Frame rate is the number of individual video frames that your camera captures, per second. In video production, a video’s frame rate is expressed as frames per second (fps).
    Standard Frame Rates:
    24fps: Cinematic Standard
    30fps: Video Standard
    60fps: Slow-Motion Standard

    ● Statement 1 is correct: A slow motion camera has a frame rate of at least 60 fps. Ultra Slow-Motion Cameras, used in sports and movie production, have a very high frame rate of 1000 fps or even more.
    ● Statement 1 is correct and is the correct explanation of statement 1: Higher frame rate means more individual frames can be captured. This is used to smoothen the video as more subtle movements are better captured. This also reduces stuttering of videos and thus videos appear smoother at a higher frame rate. That is why a slow-motion camera has a higher frame rate.

    Incorrect

    Answer: A
    Explanation
    Frame rate is the number of individual video frames that your camera captures, per second. In video production, a video’s frame rate is expressed as frames per second (fps).
    Standard Frame Rates:
    24fps: Cinematic Standard
    30fps: Video Standard
    60fps: Slow-Motion Standard

    ● Statement 1 is correct: A slow motion camera has a frame rate of at least 60 fps. Ultra Slow-Motion Cameras, used in sports and movie production, have a very high frame rate of 1000 fps or even more.
    ● Statement 1 is correct and is the correct explanation of statement 1: Higher frame rate means more individual frames can be captured. This is used to smoothen the video as more subtle movements are better captured. This also reduces stuttering of videos and thus videos appear smoother at a higher frame rate. That is why a slow-motion camera has a higher frame rate.

  3. Question 3 of 5
    3. Question

    3. With reference to the Demon Particles, consider the following statements:
    1. They do not interact with the light and are transparent.
    2. They are chargeless as well as massless in nature.
    Which of the statements given above is/are correct?

    Correct

    Answer: C
    Explanation
    Recently researchers have discovered a unique particle, known as a “demon particle”, within a metal called Strontium Ruthenate. This discovery has the potential to develop superconductors capable of operating at room temperature. They are composite particles made up of a combination of electrons. Demon particles were first predicted by theoretical physicist David Pines in 1956.
    ● Statement 1 is correct: Demon Particles are transparent, so they are tough to be excited by photonic energy and do not interact with light.
    ● Statement 2 is correct: Demon particles are chargeless as well as massless particles. Due to these properties, they are not detected under the effects of electric or magnetic fields. Due to absence of interaction with light, electric or magnetic fields, they are termed Demon Particles.

    Incorrect

    Answer: C
    Explanation
    Recently researchers have discovered a unique particle, known as a “demon particle”, within a metal called Strontium Ruthenate. This discovery has the potential to develop superconductors capable of operating at room temperature. They are composite particles made up of a combination of electrons. Demon particles were first predicted by theoretical physicist David Pines in 1956.
    ● Statement 1 is correct: Demon Particles are transparent, so they are tough to be excited by photonic energy and do not interact with light.
    ● Statement 2 is correct: Demon particles are chargeless as well as massless particles. Due to these properties, they are not detected under the effects of electric or magnetic fields. Due to absence of interaction with light, electric or magnetic fields, they are termed Demon Particles.

  4. Question 4 of 5
    4. Question

    4. Wind Tunnel is related to which of the following activities?

    Correct

    Answer: A
    Explanation
    Wind tunnels are machines where an object is held stationary inside a tube, and air is blown around it to study the interaction between the object and the moving air. They are used to test the aerodynamic effects of aircraft, rockets, cars, and buildings. Most of the time, powerful fans move air through the tube. The object to be tested is fastened in the tunnel so that it will not move. The air moving around the still object shows what would happen if the object were moving through the air. How the air moves can be studied in different ways. Smoke or dye can be placed in the air and can be seen as it moves. Threads can be attached to the object to show how the air is moving. Special instruments are often used to measure the force of the air on the object.

    Incorrect

    Answer: A
    Explanation
    Wind tunnels are machines where an object is held stationary inside a tube, and air is blown around it to study the interaction between the object and the moving air. They are used to test the aerodynamic effects of aircraft, rockets, cars, and buildings. Most of the time, powerful fans move air through the tube. The object to be tested is fastened in the tunnel so that it will not move. The air moving around the still object shows what would happen if the object were moving through the air. How the air moves can be studied in different ways. Smoke or dye can be placed in the air and can be seen as it moves. Threads can be attached to the object to show how the air is moving. Special instruments are often used to measure the force of the air on the object.

  5. Question 5 of 5
    5. Question

    5. Which of the following statements correctly explains the purpose behind launching the ‘Red Line’ campaign by the Government of India?

    Correct

    Answer: C
    Explanation
    ● Ministry of Health and Family Welfare (MoHFW) launched Red Line awareness campaign in 2016 on Antimicrobials Resistance, urging people not to use medicines marked with a red vertical line, including antibiotics, without a doctor’s prescription.

    ● Indian Council of Medical Research (ICMR) has released treatment guidelines for antimicrobial use in common syndromes which contain guidelines for use of antibiotics for viral bronchitis and low-grade fever.
    ● Antibiotics are included in Schedule H and H1 of the Drugs Rules, 1945. These drugs have specific caution labeling requirements and are sold only under the prescription of a Registered Medical Practitioner.
    About Antimicrobial Resistance (AMR)
    AMR occurs when bacteria, viruses, fungi and parasites change over time and no longer respond to medicines making infections harder to treat and increasing the risk of disease spread, severe illness and death.
    Antimicrobials – including antibiotics, antivirals, antifungals and antiparasitics – are medicines used to prevent and treat infections in humans, animals, and plants. Microorganisms that develop antimicrobial resistance are sometimes referred to as “superbugs”.

    Incorrect

    Answer: C
    Explanation
    ● Ministry of Health and Family Welfare (MoHFW) launched Red Line awareness campaign in 2016 on Antimicrobials Resistance, urging people not to use medicines marked with a red vertical line, including antibiotics, without a doctor’s prescription.

    ● Indian Council of Medical Research (ICMR) has released treatment guidelines for antimicrobial use in common syndromes which contain guidelines for use of antibiotics for viral bronchitis and low-grade fever.
    ● Antibiotics are included in Schedule H and H1 of the Drugs Rules, 1945. These drugs have specific caution labeling requirements and are sold only under the prescription of a Registered Medical Practitioner.
    About Antimicrobial Resistance (AMR)
    AMR occurs when bacteria, viruses, fungi and parasites change over time and no longer respond to medicines making infections harder to treat and increasing the risk of disease spread, severe illness and death.
    Antimicrobials – including antibiotics, antivirals, antifungals and antiparasitics – are medicines used to prevent and treat infections in humans, animals, and plants. Microorganisms that develop antimicrobial resistance are sometimes referred to as “superbugs”.

window.wpAdvQuizInitList = window.wpAdvQuizInitList || []; window.wpAdvQuizInitList.push({ id: '#wpAdvQuiz_589', init: { quizId: 589, mode: 0, globalPoints: 10, timelimit: 0, resultsGrade: [0], bo: 0, qpp: 0, catPoints: [10], formPos: 0, lbn: "Finish quiz", json: {"2739":{"type":"single","id":2739,"catId":0,"points":2,"correct":[0,0,1,0]},"2740":{"type":"single","id":2740,"catId":0,"points":2,"correct":[1,0,0,0]},"2741":{"type":"single","id":2741,"catId":0,"points":2,"correct":[0,0,1,0]},"2742":{"type":"single","id":2742,"catId":0,"points":2,"correct":[1,0,0,0]},"2743":{"type":"single","id":2743,"catId":0,"points":2,"correct":[0,0,1,0]}} } });




Day-549 | Daily MCQs | UPSC Prelims | CURRENT DEVELOPMENTS

Day-549

Time limit: 0

Quiz-summary

0 of 5 questions completed

Questions:

  1. 1
  2. 2
  3. 3
  4. 4
  5. 5

Information

DAILY MCQ

You have already completed the quiz before. Hence you can not start it again.

Quiz is loading...

You must sign in or sign up to start the quiz.

You have to finish following quiz, to start this quiz:

Results

0 of 5 questions answered correctly

Your time:

Time has elapsed

You have reached 0 of 0 points, (0)

Categories

  1. Not categorized 0%
  1. 1
  2. 2
  3. 3
  4. 4
  5. 5
  1. Answered
  2. Review
  1. Question 1 of 5
    1. Question

    1. Consider the following statements:
    Statement I: Habitat rights are given to PVTGs under the Scheduled Tribes and Other Traditional Forest Dwellers (Recognition of Forest Rights) Act, 2006.
    Statement II: All the PVTGs in India have been given Habitat rights to safeguard and promote their traditional livelihood.
    Which one of the following is correct in respect of the above statements?

    Correct

    Answer: C
    Context: In October, 2023, the Chhattisgarh Government has granted habitat rights to its Baiga PVTG (Particularly Vulnerable Tribal Group) after the Kamar PVTG received habitat rights in the state.
    Explanation:
    Statement 1 is correct: Habitat rights are given to PVTGs under section 3(1) (e) [rights including community tenures of habitat and habitation for primitive tribal groups and pre-agricultural communities] of The Scheduled Tribes and Other Traditional Forest Dwellers (Recognition of Forest Rights) Act, 2006 also known as the Forest Rights Act (FRA).
    Statement 2 is incorrect: Out of 75 PVTG in India, only three have habitat rights. The Bharia PVTG in Madhya Pradesh was the first, followed by the Kamar tribe and now the Baiga tribe in Chhattisgarh.
    Additional information:
    ● Habitat rights recognition provides the community concerned rights over their customary territory of habitation.
    ● It includes socio-cultural practices, economic and livelihood means, intellectual knowledge of biodiversity and ecology, traditional knowledge of use of natural resources, as well as protection and conservation of their natural and cultural heritage.
    ● According to Section 2(h) of FRA, “Habitat includes the area comprising the customary habitat and such other habitats in reserved forests and protected forests of primitive tribal groups and pre-agricultural communities and other forest dwelling Scheduled Tribes.”
    ● Habitat rights safeguard and promote traditional livelihood and ecological knowledge passed down through generations.
    ● They also help converge different government schemes and initiatives from various departments to empower PVTG communities to develop their habitats.

    Incorrect

    Answer: C
    Context: In October, 2023, the Chhattisgarh Government has granted habitat rights to its Baiga PVTG (Particularly Vulnerable Tribal Group) after the Kamar PVTG received habitat rights in the state.
    Explanation:
    Statement 1 is correct: Habitat rights are given to PVTGs under section 3(1) (e) [rights including community tenures of habitat and habitation for primitive tribal groups and pre-agricultural communities] of The Scheduled Tribes and Other Traditional Forest Dwellers (Recognition of Forest Rights) Act, 2006 also known as the Forest Rights Act (FRA).
    Statement 2 is incorrect: Out of 75 PVTG in India, only three have habitat rights. The Bharia PVTG in Madhya Pradesh was the first, followed by the Kamar tribe and now the Baiga tribe in Chhattisgarh.
    Additional information:
    ● Habitat rights recognition provides the community concerned rights over their customary territory of habitation.
    ● It includes socio-cultural practices, economic and livelihood means, intellectual knowledge of biodiversity and ecology, traditional knowledge of use of natural resources, as well as protection and conservation of their natural and cultural heritage.
    ● According to Section 2(h) of FRA, “Habitat includes the area comprising the customary habitat and such other habitats in reserved forests and protected forests of primitive tribal groups and pre-agricultural communities and other forest dwelling Scheduled Tribes.”
    ● Habitat rights safeguard and promote traditional livelihood and ecological knowledge passed down through generations.
    ● They also help converge different government schemes and initiatives from various departments to empower PVTG communities to develop their habitats.

  2. Question 2 of 5
    2. Question

    2. Consider the following statements regarding ‘Reverse Book Building’:
    1. It involves removing of the securities of a listed company from a stock exchange through a fixed price mechanism.
    2. It is a process used for efficient price discovery.
    Which of the statements given above is/are correct?

    Correct

    Answer: B
    Context: The Securities and Exchange Board of India is exploring the possibility of permitting the delisting of companies through a fixed price mechanism instead of the existing reverse book-building procedure.
    Explanation:
    Statement 1 is incorrect: In this method, shareholders of the company place offers to sell their securities back to the promoters or large shareholders who hold significant influence over the company’s policies. There is no fixed price mechanism in this process.
    Statement 2 is correct: Reverse book building is a process used for efficient price discovery. During the period for which the reverse book-building is open, offers are collected from shareholders at various prices, which are above or equal to the floor price. The buyback price is determined after the offer closing price.
    Additional information:
    ● Delisting means removing the securities of a listed company from a stock exchange. Once delisted, the securities of that company can no longer be traded on the stock exchange.
    ● Delisting can be either voluntary or compulsory.
    1. In voluntary delisting, a company decides on its own to remove its securities from a stock exchange.
    2. In compulsory delisting, they are removed as a penal measure for the company not making submissions or complying with requirements set out in the listing agreement within the prescribed timeframes.
    How does Reverse Book-Building Work?
    ● Price Collection Phase: During the reverse book-building process, the company announces the intention to delist, specifying the floor price—the minimum price at which the company will buy back shares from the public shareholders. The company sets a specific period during which shareholders can submit their offers to sell their shares at various prices above or equal to the floor price.
    ● Gathering Offers: Shareholders, including retail and institutional investors, participate in the process by submitting their offers. They indicate the number of shares they are willing to sell and the respective price at which they want to sell them.
    ● Price Determination: At the end of the offer submission period, the company analyzes all the offers received and determines the final buyback price based on the demand and supply of shares at different price points. The objective is to arrive at a price that encourages a sufficient number of shareholders to participate and ensures a fair exit opportunity.

    Incorrect

    Answer: B
    Context: The Securities and Exchange Board of India is exploring the possibility of permitting the delisting of companies through a fixed price mechanism instead of the existing reverse book-building procedure.
    Explanation:
    Statement 1 is incorrect: In this method, shareholders of the company place offers to sell their securities back to the promoters or large shareholders who hold significant influence over the company’s policies. There is no fixed price mechanism in this process.
    Statement 2 is correct: Reverse book building is a process used for efficient price discovery. During the period for which the reverse book-building is open, offers are collected from shareholders at various prices, which are above or equal to the floor price. The buyback price is determined after the offer closing price.
    Additional information:
    ● Delisting means removing the securities of a listed company from a stock exchange. Once delisted, the securities of that company can no longer be traded on the stock exchange.
    ● Delisting can be either voluntary or compulsory.
    1. In voluntary delisting, a company decides on its own to remove its securities from a stock exchange.
    2. In compulsory delisting, they are removed as a penal measure for the company not making submissions or complying with requirements set out in the listing agreement within the prescribed timeframes.
    How does Reverse Book-Building Work?
    ● Price Collection Phase: During the reverse book-building process, the company announces the intention to delist, specifying the floor price—the minimum price at which the company will buy back shares from the public shareholders. The company sets a specific period during which shareholders can submit their offers to sell their shares at various prices above or equal to the floor price.
    ● Gathering Offers: Shareholders, including retail and institutional investors, participate in the process by submitting their offers. They indicate the number of shares they are willing to sell and the respective price at which they want to sell them.
    ● Price Determination: At the end of the offer submission period, the company analyzes all the offers received and determines the final buyback price based on the demand and supply of shares at different price points. The objective is to arrive at a price that encourages a sufficient number of shareholders to participate and ensures a fair exit opportunity.

  3. Question 3 of 5
    3. Question

    3. With reference to the ‘GIFT NIFTY’, consider the following statements:
    1. It is the first cross-border initiative connecting capital markets of India and the USA.
    2. Any trading member Indian or foreign setting up its office through subsidiary model can start trading in the GIFT Nifty products by taking membership of NSE IX.
    3. The GIFT NIFTY contracts are denominated in US dollars, which reduces the currency risk and conversion costs for international investors.
    How many of the statements given above are correct?

    Correct

    Answer: B
    Context: The launch of GIFT NIFTY, formerly known as SGX NIFTY, marked a new milestone in the collaboration between India and Singapore. It began trading at the Gujarat International Finance Tec-City (GIFT City) in July, 2023 attracting more than 30,000 trades in one session.
    Explanation:
    Statement 1 is incorrect: It is the first cross-border initiative in connecting India and Singapore’s capital markets. It is a new product that allows investors to trade futures and options contracts on the NIFTY 50 Index, which is the benchmark index of the National Stock Exchange of India (NSE), on the Singapore Exchange (SGX).
    Statement 2 is correct: Any trading member Indian or foreign setting up its office through subsidiary/branch model can start trading in the GIFT Nifty products by taking member ship of NSE IX. They can be either registered or non-registered.
    Statement 3 is correct: GIFT NIFTY contracts are denominated in US dollars, which reduces the currency risk and conversion costs for international investors.
    Additional information:
    ● The GIFT NIFTY stands for Gujarat International Finance Tec-City (GIFT) NIFTY, as the contracts are cleared and settled at the International Financial Services Centre (IFSC) in GIFT City, Gujarat, India.
    ● The IFSC is a special economic zone that offers various incentives and benefits to participants, such as lower taxes, relaxed regulations, and access to global markets.
    ● It is supported by the regulatory authorities of both countries, namely the Securities and Exchange Board of India (SEBI) and the Monetary Authority of Singapore (MAS).
    ● The GIFT NIFTY contracts are available for trading from 9:00 am to 11:55 pm Singapore time (6:30 am to 9:25 pm India time), which covers both the Indian and Singapore market hours, as well as some of the European and US market hours.
    ● The GIFT NIFTY contracts also have lower transaction fees and margin requirements than the existing NIFTY 50 contracts traded on the NSE.

    Incorrect

    Answer: B
    Context: The launch of GIFT NIFTY, formerly known as SGX NIFTY, marked a new milestone in the collaboration between India and Singapore. It began trading at the Gujarat International Finance Tec-City (GIFT City) in July, 2023 attracting more than 30,000 trades in one session.
    Explanation:
    Statement 1 is incorrect: It is the first cross-border initiative in connecting India and Singapore’s capital markets. It is a new product that allows investors to trade futures and options contracts on the NIFTY 50 Index, which is the benchmark index of the National Stock Exchange of India (NSE), on the Singapore Exchange (SGX).
    Statement 2 is correct: Any trading member Indian or foreign setting up its office through subsidiary/branch model can start trading in the GIFT Nifty products by taking member ship of NSE IX. They can be either registered or non-registered.
    Statement 3 is correct: GIFT NIFTY contracts are denominated in US dollars, which reduces the currency risk and conversion costs for international investors.
    Additional information:
    ● The GIFT NIFTY stands for Gujarat International Finance Tec-City (GIFT) NIFTY, as the contracts are cleared and settled at the International Financial Services Centre (IFSC) in GIFT City, Gujarat, India.
    ● The IFSC is a special economic zone that offers various incentives and benefits to participants, such as lower taxes, relaxed regulations, and access to global markets.
    ● It is supported by the regulatory authorities of both countries, namely the Securities and Exchange Board of India (SEBI) and the Monetary Authority of Singapore (MAS).
    ● The GIFT NIFTY contracts are available for trading from 9:00 am to 11:55 pm Singapore time (6:30 am to 9:25 pm India time), which covers both the Indian and Singapore market hours, as well as some of the European and US market hours.
    ● The GIFT NIFTY contracts also have lower transaction fees and margin requirements than the existing NIFTY 50 contracts traded on the NSE.

  4. Question 4 of 5
    4. Question

    4. Which of the following best describes the meaning of the term ‘’dotted land’’?

    Correct

    Answer: A
    Context: In May, 2023, the Andhra Pradesh government has started removing “dotted lands” in the state from the prohibited list, restoring full rights of selling or pledging these lands to the farmers who own them. Over 2 lakh acres of these British-era dotted lands have been identified for permanent denotification.
    Explanation:
    ● Dotted lands are disputed lands for which there are no clear ownership documents. Typically, one or more individuals as well as the government’s Revenue Department lay claim over the land.
    ● These lands came to be known as “dotted lands’’ during, land ownership surveys and resettlement of land records the British era.
    ● In British era local revenue officials who were tasked with identifying government-owned and privately-owned lands put dots in the ownership column if more than one person claimed ownership.
    How did these ownership disputes arise?
    ● This could happen if landowners did not leave clear wills passing on land to their heirs or children, and if a dispute arose because more than one heir lay claim over the land.
    ● Also, land could be deemed by the government to belong to the state but was under occupation by private parties.
    ● Some of the land records in question are more than 100 years old, and had been locked up in the prohibited list in and registers.
    ● During subsequent surveys, government officials left the ownership column blank indicating their disputed status as per Section 22A of the Registration Act.

    Incorrect

    Answer: A
    Context: In May, 2023, the Andhra Pradesh government has started removing “dotted lands” in the state from the prohibited list, restoring full rights of selling or pledging these lands to the farmers who own them. Over 2 lakh acres of these British-era dotted lands have been identified for permanent denotification.
    Explanation:
    ● Dotted lands are disputed lands for which there are no clear ownership documents. Typically, one or more individuals as well as the government’s Revenue Department lay claim over the land.
    ● These lands came to be known as “dotted lands’’ during, land ownership surveys and resettlement of land records the British era.
    ● In British era local revenue officials who were tasked with identifying government-owned and privately-owned lands put dots in the ownership column if more than one person claimed ownership.
    How did these ownership disputes arise?
    ● This could happen if landowners did not leave clear wills passing on land to their heirs or children, and if a dispute arose because more than one heir lay claim over the land.
    ● Also, land could be deemed by the government to belong to the state but was under occupation by private parties.
    ● Some of the land records in question are more than 100 years old, and had been locked up in the prohibited list in and registers.
    ● During subsequent surveys, government officials left the ownership column blank indicating their disputed status as per Section 22A of the Registration Act.

  5. Question 5 of 5
    5. Question

    5. Consider the following statements regarding Gandhi Peace Prize:
    1. It is only given to individuals who have worked selflessly for peace, non-violence and amelioration of human sufferings.
    2. The awardees do not get any cash reward, but a certificate signed by the President apart from a medallion which they can wear at public or government function.
    Which of the statements given above is/are correct?

    Correct

    Answer: D
    Context: In June 2023, the Gandhi prize was awarded to Gita Press for its ‘outstanding contribution towards social, economic and political transformation through non-violent and other Gandhian methods’.
    Explanation:
    Statement 1 is incorrect: This award is given to individuals, associations, institutions or organizations who have worked selflessly for peace, non-violence and amelioration of human sufferings.
    Statement 2 is incorrect: The award carries an amount of Rs. 1 crore, a citation, a plaque and an exquisite traditional handicraft/handloom item. The award is conferred by the President of India at a function in Rashtrapati Bhavan.
    Additional information:
    ● Gandhi Peace Prize is an annual award instituted by Government of India in 1995, on the occasion of 125th Birth Anniversary of Mahatma Gandhi as a tribute to the ideals espoused by Mahatma Gandhi.
    ● The award is open to all persons regardless of nationality, race, language, caste, creed or gender.
    ● The Award may be divided between two persons / institutions who are considered by the Jury to be equally deserving of recognition in a given year.
    ● Work by a person since deceased cannot be the subject of an Award. If, however, his death occurred subsequent to a proposal having been submitted to the Jury(headed by the Prime Minister) in the manner stipulated in the Code of Procedure of the Prize, then a Posthumous Award may be made.
    ● Established in 1923, Gita Press is one of the world’s largest publishers, having published 41.7 crore books in 14 languages, including 16.21 crore Shrimad Bhagvad Gita.
    ● The institution has never relied on advertisement in its publications, for revenue generation. Gita Press, along with its affiliated organizations, strives for the betterment of life and the wellbeing of all.

    Incorrect

    Answer: D
    Context: In June 2023, the Gandhi prize was awarded to Gita Press for its ‘outstanding contribution towards social, economic and political transformation through non-violent and other Gandhian methods’.
    Explanation:
    Statement 1 is incorrect: This award is given to individuals, associations, institutions or organizations who have worked selflessly for peace, non-violence and amelioration of human sufferings.
    Statement 2 is incorrect: The award carries an amount of Rs. 1 crore, a citation, a plaque and an exquisite traditional handicraft/handloom item. The award is conferred by the President of India at a function in Rashtrapati Bhavan.
    Additional information:
    ● Gandhi Peace Prize is an annual award instituted by Government of India in 1995, on the occasion of 125th Birth Anniversary of Mahatma Gandhi as a tribute to the ideals espoused by Mahatma Gandhi.
    ● The award is open to all persons regardless of nationality, race, language, caste, creed or gender.
    ● The Award may be divided between two persons / institutions who are considered by the Jury to be equally deserving of recognition in a given year.
    ● Work by a person since deceased cannot be the subject of an Award. If, however, his death occurred subsequent to a proposal having been submitted to the Jury(headed by the Prime Minister) in the manner stipulated in the Code of Procedure of the Prize, then a Posthumous Award may be made.
    ● Established in 1923, Gita Press is one of the world’s largest publishers, having published 41.7 crore books in 14 languages, including 16.21 crore Shrimad Bhagvad Gita.
    ● The institution has never relied on advertisement in its publications, for revenue generation. Gita Press, along with its affiliated organizations, strives for the betterment of life and the wellbeing of all.

window.wpAdvQuizInitList = window.wpAdvQuizInitList || []; window.wpAdvQuizInitList.push({ id: '#wpAdvQuiz_588', init: { quizId: 588, mode: 0, globalPoints: 10, timelimit: 0, resultsGrade: [0], bo: 0, qpp: 0, catPoints: [10], formPos: 0, lbn: "Finish quiz", json: {"2734":{"type":"single","id":2734,"catId":0,"points":2,"correct":[0,0,1,0]},"2735":{"type":"single","id":2735,"catId":0,"points":2,"correct":[0,1,0,0]},"2736":{"type":"single","id":2736,"catId":0,"points":2,"correct":[0,1,0,0]},"2737":{"type":"single","id":2737,"catId":0,"points":2,"correct":[1,0,0,0]},"2738":{"type":"single","id":2738,"catId":0,"points":2,"correct":[0,0,0,1]}} } });




Day-548 | Daily MCQs | UPSC Prelims | ECONOMY

Day-548

Time limit: 0

Quiz-summary

0 of 5 questions completed

Questions:

  1. 1
  2. 2
  3. 3
  4. 4
  5. 5

Information

DAILY MCQ

You have already completed the quiz before. Hence you can not start it again.

Quiz is loading...

You must sign in or sign up to start the quiz.

You have to finish following quiz, to start this quiz:

Results

0 of 5 questions answered correctly

Your time:

Time has elapsed

You have reached 0 of 0 points, (0)

Categories

  1. Not categorized 0%
  1. 1
  2. 2
  3. 3
  4. 4
  5. 5
  1. Answered
  2. Review
  1. Question 1 of 5
    1. Question

    1. Consider the following:
    1. Access to a larger and more liquid market
    2. Savings on the costs and time involved in the Initial Public Offering (IPO) process
    3. Avoidance of the dilution of ownership and control
    4. Exposure to the currency fluctuations and market volatility of the foreign exchange
    How many of the above are the possible benefits of direct listing of shares on a foreign stock exchange?

    Correct

    Answer: C
    Explanation:
    ● Direct listing is a process by which a company can list its shares on a foreign stock exchange without issuing new shares or raising capital from investors.
    ● Direct listing is different from the traditional Initial Public Offering (IPO), where a company sells a portion of its shares to the public and raises funds from investors.
    ● Direct listing is also different from the Depository Receipt (DR) route, where a company issues its shares to a custodian bank, which then issues DRs to foreign investors.
    Not a possible benefit of direct foreign listing:
    ● Exposure to the currency fluctuations and market volatility of the foreign exchange is one of the effects of direct foreign list. However, it can affect their share price and returns. So, it is a challenge not a benefit.
    Benefits of Direct Foreign Listing:
    ● Access to a larger and more liquid market, which can increase the demand and valuation of their shares.
    ● Savings on the costs and time involved in the IPO or DR process, such as underwriting fees, listing fees, legal fees, etc.
    ● Avoidance of the dilution of ownership and control that comes with issuing new shares or DRs.
    ● Ability to reach out to a wider and more sophisticated investor base, which can enhance their reputation and credibility.
    ● Startups and unicorns may benefit from this avenue of raising funds and increasing their global profile.
    ● Exposure to the best practices and regulations of the foreign jurisdiction can improve their governance and transparency.
    Additional information:
    How do Indian Companies Currently List on Foreign Exchanges?
    ● Currently, Indian companies list on foreign bourses using depository receipts, including American Depository Receipts (ADRs) and Global Depository Receipts (GDRs).
    ● To list on foreign stock exchanges, Indian companies entrust their shares to an Indian custodian, who then issues depository receipts (DRs) to foreign investors.
    ● Between 2008 and 2018, 109 companies raised over Rs 51,000 crore through ADRs/GDRs.
    ● However, after 2018, no Indian companies pursued overseas listings through this route.
    ● ADR refers to a negotiable certificate issued by a U.S. depositary bank representing a specified number of shares, usually one share of a foreign company’s stock.
    ● GDRs is a certificate issued by a depository bank that represents shares in a foreign company and deposits them in an account. GDRs are mostly traded on the European markets.
    Challenges Involved in Direct Foreign Listing:
    ● Compliance with the laws and rules of the foreign jurisdiction, which may be different from or more stringent than those in India.
    ● Challenges in direct foreign listings include valuation issues, as global investors may not offer the same valuations as in India, potentially impacting the company’s market perception and pricing.
    ● Potential conflicts or disputes with the existing shareholders, regulators, or tax authorities in India or abroad.
    ● Clarity is needed on which classes of public companies can use this route, the classes of securities that can be listed, the foreign jurisdictions and permitted stock exchanges for listing, and the exemptions offered to such companies in terms of procedural compliances.

    Incorrect

    Answer: C
    Explanation:
    ● Direct listing is a process by which a company can list its shares on a foreign stock exchange without issuing new shares or raising capital from investors.
    ● Direct listing is different from the traditional Initial Public Offering (IPO), where a company sells a portion of its shares to the public and raises funds from investors.
    ● Direct listing is also different from the Depository Receipt (DR) route, where a company issues its shares to a custodian bank, which then issues DRs to foreign investors.
    Not a possible benefit of direct foreign listing:
    ● Exposure to the currency fluctuations and market volatility of the foreign exchange is one of the effects of direct foreign list. However, it can affect their share price and returns. So, it is a challenge not a benefit.
    Benefits of Direct Foreign Listing:
    ● Access to a larger and more liquid market, which can increase the demand and valuation of their shares.
    ● Savings on the costs and time involved in the IPO or DR process, such as underwriting fees, listing fees, legal fees, etc.
    ● Avoidance of the dilution of ownership and control that comes with issuing new shares or DRs.
    ● Ability to reach out to a wider and more sophisticated investor base, which can enhance their reputation and credibility.
    ● Startups and unicorns may benefit from this avenue of raising funds and increasing their global profile.
    ● Exposure to the best practices and regulations of the foreign jurisdiction can improve their governance and transparency.
    Additional information:
    How do Indian Companies Currently List on Foreign Exchanges?
    ● Currently, Indian companies list on foreign bourses using depository receipts, including American Depository Receipts (ADRs) and Global Depository Receipts (GDRs).
    ● To list on foreign stock exchanges, Indian companies entrust their shares to an Indian custodian, who then issues depository receipts (DRs) to foreign investors.
    ● Between 2008 and 2018, 109 companies raised over Rs 51,000 crore through ADRs/GDRs.
    ● However, after 2018, no Indian companies pursued overseas listings through this route.
    ● ADR refers to a negotiable certificate issued by a U.S. depositary bank representing a specified number of shares, usually one share of a foreign company’s stock.
    ● GDRs is a certificate issued by a depository bank that represents shares in a foreign company and deposits them in an account. GDRs are mostly traded on the European markets.
    Challenges Involved in Direct Foreign Listing:
    ● Compliance with the laws and rules of the foreign jurisdiction, which may be different from or more stringent than those in India.
    ● Challenges in direct foreign listings include valuation issues, as global investors may not offer the same valuations as in India, potentially impacting the company’s market perception and pricing.
    ● Potential conflicts or disputes with the existing shareholders, regulators, or tax authorities in India or abroad.
    ● Clarity is needed on which classes of public companies can use this route, the classes of securities that can be listed, the foreign jurisdictions and permitted stock exchanges for listing, and the exemptions offered to such companies in terms of procedural compliances.

  2. Question 2 of 5
    2. Question

    2. Consider the following statements about the Index of Industrial Production (IIP):
    1. It measures the short-term changes in the volume of production of a basket of industrial products.
    2. The Central Statistical Organization (CSO) computes and publishes the IIP index on a quarterly basis.
    3. The current base year for the IIP series in India is 2011-12.
    How many of the statements given above are correct?

    Correct

    Answer: B
    Explanation:
    Statement 1 is correct: The IIP measures the short-term changes in the volume of production of a basket of industrial products.
    Statement 2 is incorrect: The Central Statistical Organization (CSO) computes and publishes the IIP index on a monthly basis.
    Statement 3 is correct: The current base year for the IIP series in India is 2011-12.
    Additional information:
    ● IIP is an important composite indicator in India that measures the changes in the volume of production of a basket of industrial products. IIP measures the growth of manufacturing, mining, and electricity sectors.
    ● The aim of IIP is to capture the direction and the trend of industrial production in the country, not the absolute value of industrial production.
    ● This index gives the growth rates of different industry groups of the economy over a specified time period.
    ● The eight core industries of India represent about 40% of the weight of items that are included in the IIP. The Eight Core Sectors/Industries are: Electricity; Steel; Refinery products; Crude oil; Coal; Cement; Natural gas; Fertilizers.
    ● The industry groups that it measures are classified under the following:
    1. Broad sectors like manufacturing, mining, and electricity.
    2. Use-based sectors like capital goods, basic goods, intermediate goods, infrastructure goods, consumer durables, and consumer non-durables.

    Incorrect

    Answer: B
    Explanation:
    Statement 1 is correct: The IIP measures the short-term changes in the volume of production of a basket of industrial products.
    Statement 2 is incorrect: The Central Statistical Organization (CSO) computes and publishes the IIP index on a monthly basis.
    Statement 3 is correct: The current base year for the IIP series in India is 2011-12.
    Additional information:
    ● IIP is an important composite indicator in India that measures the changes in the volume of production of a basket of industrial products. IIP measures the growth of manufacturing, mining, and electricity sectors.
    ● The aim of IIP is to capture the direction and the trend of industrial production in the country, not the absolute value of industrial production.
    ● This index gives the growth rates of different industry groups of the economy over a specified time period.
    ● The eight core industries of India represent about 40% of the weight of items that are included in the IIP. The Eight Core Sectors/Industries are: Electricity; Steel; Refinery products; Crude oil; Coal; Cement; Natural gas; Fertilizers.
    ● The industry groups that it measures are classified under the following:
    1. Broad sectors like manufacturing, mining, and electricity.
    2. Use-based sectors like capital goods, basic goods, intermediate goods, infrastructure goods, consumer durables, and consumer non-durables.

  3. Question 3 of 5
    3. Question

    3. With reference to the Angel tax in India, consider the following statements:
    1. Angel tax is an income tax, applicable only on unlisted companies.
    2. Angel tax can be imposed only on investments made by a resident investor.
    3. All government recognised startups are exempt from Angel tax.
    How many of the statements given above are correct?

    Correct

    Answer: A
    Explanation:
    ● Statement 1 is correct: Angel Tax is a term basically used to refer to the income tax payable on the capital raised by unlisted companies via the issue of shares through off-market transactions.
    ● Statement 2 is incorrect: Before budget 2023-24, angel tax was imposed only on investments made by a resident investor. i.e., it was not applicable in case the investments are made by any non-resident or venture capital funds. With Budget 2023-24, the government has decided to include foreign investors in the ambit. That means when a start-up raises funding from a foreign investor, that too will now be counted as income and be taxable. However, these foreign investors will not need to pay any angel tax while investing in a government-recognised startup in India similar to the provision for domestic investors.
    ● Statement 3 is incorrect: All government recognised startups are not exempt from the Angel Tax. It is only those start-ups which meet certain criteria are exempted from this tax. Moreover, the Central Board of Direct Taxes (CBDT) has clarified that government recognised startups will not be scrutinized for the changes brought about by the Budget 2023-24.
    Additional information:
    ● Angel tax is levied on the capital raised via the issue of shares by unlisted companies from an Indian investor/ foreign investor from certain countries if the share price of issued shares is seen in excess of the fair market value of the company.
    ● Rule related to Angel Tax is described in Section 56(2)(viib) of the Income Tax Act, 1961.
    ● This clause was inserted into the act in 2012 to prevent laundering of black money, round-tripping via investments with a large premium into unlisted companies.
    ● Allaying the concerns of the startup community, the govt had also exempted investments made by the domestic investors in companies approved by an inter-ministerial panel from Angel Tax.
    ● Government recognised startups, upon meeting certain criteria are exempted from this tax.
    ● Currently, angel tax is levied at the rate of 30.6 per cent.

    Incorrect

    Answer: A
    Explanation:
    ● Statement 1 is correct: Angel Tax is a term basically used to refer to the income tax payable on the capital raised by unlisted companies via the issue of shares through off-market transactions.
    ● Statement 2 is incorrect: Before budget 2023-24, angel tax was imposed only on investments made by a resident investor. i.e., it was not applicable in case the investments are made by any non-resident or venture capital funds. With Budget 2023-24, the government has decided to include foreign investors in the ambit. That means when a start-up raises funding from a foreign investor, that too will now be counted as income and be taxable. However, these foreign investors will not need to pay any angel tax while investing in a government-recognised startup in India similar to the provision for domestic investors.
    ● Statement 3 is incorrect: All government recognised startups are not exempt from the Angel Tax. It is only those start-ups which meet certain criteria are exempted from this tax. Moreover, the Central Board of Direct Taxes (CBDT) has clarified that government recognised startups will not be scrutinized for the changes brought about by the Budget 2023-24.
    Additional information:
    ● Angel tax is levied on the capital raised via the issue of shares by unlisted companies from an Indian investor/ foreign investor from certain countries if the share price of issued shares is seen in excess of the fair market value of the company.
    ● Rule related to Angel Tax is described in Section 56(2)(viib) of the Income Tax Act, 1961.
    ● This clause was inserted into the act in 2012 to prevent laundering of black money, round-tripping via investments with a large premium into unlisted companies.
    ● Allaying the concerns of the startup community, the govt had also exempted investments made by the domestic investors in companies approved by an inter-ministerial panel from Angel Tax.
    ● Government recognised startups, upon meeting certain criteria are exempted from this tax.
    ● Currently, angel tax is levied at the rate of 30.6 per cent.

  4. Question 4 of 5
    4. Question

    4. Which one of the following statements about the Sukanya Samriddhi Scheme is incorrect?

    Correct

    Answer: A
    Explanation:
    Statement 1 is incorrect: Parents or legal guardians can open deposits on behalf of a girl child (including adopted girl child) for up to 2 daughters aged below 10.
    Statement 2 is correct: Only one account can be opened in the name of a girl child.
    Statement 3 is correct: Maximum period up to which deposits can be made 15 years from the date of opening of the account.
    Statement 4 is correct: Interest that accrues against this account which gets compounded annually is also exempt from tax under Section 10 of the Income Tax Act.
    Additional information:
    ● Sukanya Samriddhi Yojana (SSY) Scheme was Introduced in 2016, the Sukanya Samriddhi Yojana Account (SSA) is a central government scheme aimed to cater to a girl child.
    ● The Sukanya Samriddhi Yojana is a government savings scheme created with the intention to benefit girl child under the initiative called “Beti Bachao – Beti Padhao”.
    ● This scheme carries a higher interest rate along with several tax benefits.
    ● The principal amount deposited, interest earned during the entire tenure and maturity benefits are tax-exempt. The principal amount is deductible under section 80C up to Rs 1.5 lakh.
    ● The proceeds received upon maturity/withdrawal are also exempt from income tax.
    ● The account matures after 21 years of opening the account or in the event of the marriage of the girl child after she gains the age of 18 years.
    ● A premature withdrawal of up to 50% of the investment is allowed after the child gains the age of 18 years even if she is not getting married.
    ● Three girl children, in case of twin girls as second birth or the first birth itself results in three girl children.
    ● Minimum of Rs 250 of initial deposit with multiple of Rs 150 thereafter with annual ceiling of Rs.150000 in a financial year.
    ● The account shall mature on completion of 21 years from the date of opening of account or on the marriage of Account holder whichever is earlier.

    Incorrect

    Answer: A
    Explanation:
    Statement 1 is incorrect: Parents or legal guardians can open deposits on behalf of a girl child (including adopted girl child) for up to 2 daughters aged below 10.
    Statement 2 is correct: Only one account can be opened in the name of a girl child.
    Statement 3 is correct: Maximum period up to which deposits can be made 15 years from the date of opening of the account.
    Statement 4 is correct: Interest that accrues against this account which gets compounded annually is also exempt from tax under Section 10 of the Income Tax Act.
    Additional information:
    ● Sukanya Samriddhi Yojana (SSY) Scheme was Introduced in 2016, the Sukanya Samriddhi Yojana Account (SSA) is a central government scheme aimed to cater to a girl child.
    ● The Sukanya Samriddhi Yojana is a government savings scheme created with the intention to benefit girl child under the initiative called “Beti Bachao – Beti Padhao”.
    ● This scheme carries a higher interest rate along with several tax benefits.
    ● The principal amount deposited, interest earned during the entire tenure and maturity benefits are tax-exempt. The principal amount is deductible under section 80C up to Rs 1.5 lakh.
    ● The proceeds received upon maturity/withdrawal are also exempt from income tax.
    ● The account matures after 21 years of opening the account or in the event of the marriage of the girl child after she gains the age of 18 years.
    ● A premature withdrawal of up to 50% of the investment is allowed after the child gains the age of 18 years even if she is not getting married.
    ● Three girl children, in case of twin girls as second birth or the first birth itself results in three girl children.
    ● Minimum of Rs 250 of initial deposit with multiple of Rs 150 thereafter with annual ceiling of Rs.150000 in a financial year.
    ● The account shall mature on completion of 21 years from the date of opening of account or on the marriage of Account holder whichever is earlier.

  5. Question 5 of 5
    5. Question

    5. Consider the following statements:
    1. When the RBI buys government securities, it can increase borrowing costs in the market.
    2. When the RBI sells government securities, it can depreciate the rupees.
    Which of the statements given above is/are correct?

    Correct

    Answer: D
    Explanation:
    Statement 1 is incorrect: Open Market Operations (OMOs) have a significant impact on interest rates. When the RBI buys government securities, it injects liquidity in the market. Banks have excess funds to lend to borrowers. This reduces the cost of borrowing in the market.
    Statement 2 is incorrect: OMOs can also influence the exchange rate of the Indian rupee. When the RBI sells government securities, it receives rupees from buyers, reducing the supply of rupees in the market and putting upward pressure on the exchange rate, making the rupee stronger.
    Additional information:
    ● Open market operations (OMOs) are one of the tools that the Reserve Bank of India (RBI) uses to regulate the money supply and liquidity conditions in the economy.
    ● OMOs are the sale and purchase of government securities (G-Secs) and treasury bills (T-Bills) by the RBI in the open market. When RBI wants to inject liquidity into the system, it buys G-Secs and T-Bills from the market, thereby increasing the money supply.
    ● Conversely, when RBI wants to absorb excess liquidity from the system, it sells G-Secs and T-Bills to the market, thereby reducing the money supply.
    ● OMO plays a crucial role in ensuring consistent liquidity availability throughout the year, thus maintaining desired interest rate levels.
    ● Regular OMO activities are conducted to strike a balance between controlling inflation and ensuring that banks maintain sufficient funds for lending.
    ● Factors contributing to this tightening include reduced government expenditure, heightened currency demand during the festive season, and fluctuations in foreign institutional investments and tax payments.

    Incorrect

    Answer: D
    Explanation:
    Statement 1 is incorrect: Open Market Operations (OMOs) have a significant impact on interest rates. When the RBI buys government securities, it injects liquidity in the market. Banks have excess funds to lend to borrowers. This reduces the cost of borrowing in the market.
    Statement 2 is incorrect: OMOs can also influence the exchange rate of the Indian rupee. When the RBI sells government securities, it receives rupees from buyers, reducing the supply of rupees in the market and putting upward pressure on the exchange rate, making the rupee stronger.
    Additional information:
    ● Open market operations (OMOs) are one of the tools that the Reserve Bank of India (RBI) uses to regulate the money supply and liquidity conditions in the economy.
    ● OMOs are the sale and purchase of government securities (G-Secs) and treasury bills (T-Bills) by the RBI in the open market. When RBI wants to inject liquidity into the system, it buys G-Secs and T-Bills from the market, thereby increasing the money supply.
    ● Conversely, when RBI wants to absorb excess liquidity from the system, it sells G-Secs and T-Bills to the market, thereby reducing the money supply.
    ● OMO plays a crucial role in ensuring consistent liquidity availability throughout the year, thus maintaining desired interest rate levels.
    ● Regular OMO activities are conducted to strike a balance between controlling inflation and ensuring that banks maintain sufficient funds for lending.
    ● Factors contributing to this tightening include reduced government expenditure, heightened currency demand during the festive season, and fluctuations in foreign institutional investments and tax payments.

window.wpAdvQuizInitList = window.wpAdvQuizInitList || []; window.wpAdvQuizInitList.push({ id: '#wpAdvQuiz_587', init: { quizId: 587, mode: 0, globalPoints: 10, timelimit: 0, resultsGrade: [0], bo: 0, qpp: 0, catPoints: [10], formPos: 0, lbn: "Finish quiz", json: {"2729":{"type":"single","id":2729,"catId":0,"points":2,"correct":[0,0,1,0]},"2730":{"type":"single","id":2730,"catId":0,"points":2,"correct":[0,1,0,0]},"2731":{"type":"single","id":2731,"catId":0,"points":2,"correct":[1,0,0,0]},"2732":{"type":"single","id":2732,"catId":0,"points":2,"correct":[1,0,0,0]},"2733":{"type":"single","id":2733,"catId":0,"points":2,"correct":[0,0,0,1]}} } });




Day-547 | Daily MCQs | UPSC Prelims | ENVIRONMENT

Day-547

Time limit: 0

Quiz-summary

0 of 5 questions completed

Questions:

  1. 1
  2. 2
  3. 3
  4. 4
  5. 5

Information

DAILY MCQ

You have already completed the quiz before. Hence you can not start it again.

Quiz is loading...

You must sign in or sign up to start the quiz.

You have to finish following quiz, to start this quiz:

Results

0 of 5 questions answered correctly

Your time:

Time has elapsed

You have reached 0 of 0 points, (0)

Categories

  1. Not categorized 0%
  1. 1
  2. 2
  3. 3
  4. 4
  5. 5
  1. Answered
  2. Review
  1. Question 1 of 5
    1. Question

    1. Consider the following statements about ‘Sea Butterfly’, recently seen in the news:
    1. It is a unique pelagic fish species.
    2. It is a carnivore that preys on the other small fish species.
    3. The acidic ocean waters are harmful for its survival.
    How many of the above statements are correct?

    Correct

    Answer: A
    Explanation:
    Statement 1 is incorrect: Sea butterfly is a type of planktonic marine snail belonging to the family Thecosomata, and its scientific name is Limacina helicina. They fall under pelagic sea snails.
    ● They are basically transparent. However, they have an orange red coloured head, a tail, and a gut to easily notice them in the ocean.
    ● Instead of regular wings, they have a set of developed parapodia or lobes. These act as their wings. Using the developed parapodia or lobes, they swim through the ocean water and heavy currents under the sea.

    Statement 2 is incorrect: Sea butterfly is a herbivore whose diet comprises of algae, zooplankton, and chalinid sponges. Sea butterflies have a special gland to capture the food or the tiny plankton. The gland has a mucous web like a net that helps them to take the plankton as food.
    ● The mucous membrane they use to capture the plankton is 5 cm, which is bigger than the body itself.
    ● The predators posing threats to sea butterfly include other bigger fish, birds, jellyfish, sea turtles, and whales.
    Statement 3 is correct: Ocean acidification is more dangerous for shelled, smaller sea butterflies which can impact entire ocean food web. As the sea absorbs an increasing quantity of carbon dioxide (CO2) that turns the water more acidic, the thin outer casing, or the ‘homes’ of these small sea butterflies, dissolve. This leaves them exposed, making it difficult for these delicate species to survive.
    ● The ocean is the most acidic in winter because cooler water absorbs more CO2. This means, the winter months are the most dangerous for the shelled sea butterflies.
    ● Their reduced population also impacts larger pteropods and other oceanic creatures that feed on them. The entire underwater food chain of the seas surrounding Antarctica may suffer as a ripple effect.
    Additional Information:
    ● Distribution: Sea butterflies are abundant worldwide in all open seas and oceans. They are widely available from the western coast of America to the North Atlantic Ocean.
    ● Behaviour Pattern: Social creatures and they often form groups called swarms.
    ● Conservation Status: Least Concern (LC).
    ● Reproduction: Sea butterflies are hermaphrodites. That means sea butterflies, irrespective of male and female; they possess both reproductive organs. During the early stage, sea butterflies remain as males and develop a male reproductive organ. Even when they mate, the sperms are contained until the male reproductive organ vanishes and turn out like females to lay eggs.

    Incorrect

    Answer: A
    Explanation:
    Statement 1 is incorrect: Sea butterfly is a type of planktonic marine snail belonging to the family Thecosomata, and its scientific name is Limacina helicina. They fall under pelagic sea snails.
    ● They are basically transparent. However, they have an orange red coloured head, a tail, and a gut to easily notice them in the ocean.
    ● Instead of regular wings, they have a set of developed parapodia or lobes. These act as their wings. Using the developed parapodia or lobes, they swim through the ocean water and heavy currents under the sea.

    Statement 2 is incorrect: Sea butterfly is a herbivore whose diet comprises of algae, zooplankton, and chalinid sponges. Sea butterflies have a special gland to capture the food or the tiny plankton. The gland has a mucous web like a net that helps them to take the plankton as food.
    ● The mucous membrane they use to capture the plankton is 5 cm, which is bigger than the body itself.
    ● The predators posing threats to sea butterfly include other bigger fish, birds, jellyfish, sea turtles, and whales.
    Statement 3 is correct: Ocean acidification is more dangerous for shelled, smaller sea butterflies which can impact entire ocean food web. As the sea absorbs an increasing quantity of carbon dioxide (CO2) that turns the water more acidic, the thin outer casing, or the ‘homes’ of these small sea butterflies, dissolve. This leaves them exposed, making it difficult for these delicate species to survive.
    ● The ocean is the most acidic in winter because cooler water absorbs more CO2. This means, the winter months are the most dangerous for the shelled sea butterflies.
    ● Their reduced population also impacts larger pteropods and other oceanic creatures that feed on them. The entire underwater food chain of the seas surrounding Antarctica may suffer as a ripple effect.
    Additional Information:
    ● Distribution: Sea butterflies are abundant worldwide in all open seas and oceans. They are widely available from the western coast of America to the North Atlantic Ocean.
    ● Behaviour Pattern: Social creatures and they often form groups called swarms.
    ● Conservation Status: Least Concern (LC).
    ● Reproduction: Sea butterflies are hermaphrodites. That means sea butterflies, irrespective of male and female; they possess both reproductive organs. During the early stage, sea butterflies remain as males and develop a male reproductive organ. Even when they mate, the sperms are contained until the male reproductive organ vanishes and turn out like females to lay eggs.

  2. Question 2 of 5
    2. Question

    2. Consider the following statements in the context of short-lived halogens:
    1. Short-lived halogens with oceanic sources have a lifespan of less than three months in the atmosphere.
    2. They tend to increase global warming by depleting ozone.
    3. They increase the lifespan of methane in the atmosphere.
    How many of the statements given above are correct?

    Correct

    Answer: A
    Explanation:
    Recently, a study was published in the journal Nature which elaborately discussed the role of oceans in release of short-lived halogens.
    Statements 1 and 2 are incorrect- Short-lived halogens from oceans reduces warming by depleting ozone. Its cooling effect was found to be -0.24 ± 0.02 Watts per square metre (W m−2).
    Short-lived halogens have a lifetime of less than six months in the atmosphere and are naturally produced by the oceans. Examples of such halogens include such as chlorine, bromine and iodine. Currently, these halogens contribute 8-10 percent of cooling (and not global warming). This cooling could increase to 18-31 per cent by 2100, the study published in Nature projected.
    Human activities have amplified their release into the atmosphere. Human activities cause pollutants such as ozone to deposit on the ocean, which then convert the soluble short-lived halogens into insoluble ones, forcing them out of the sea water and into the atmosphere.
    Statement 3 is correct- However, their effect on methane is opposite. Short-lived halogens increase methane’s lifetime in the atmosphere by destroying hydroxyl radicals (OH). OH is a sink as it is known to break down this greenhouse gas.
    Additional information:
    ● These short-lived halogens increased global methane burden by 14 per cent and 9 per cent for pre-industrial and present-day conditions, respectively. This leads to a warming effect of 0.09 ± 0.01 W m−2 of warming.
    ● Similarly, these halogens increase the levels of water vapour, a greenhouse gas, in the atmosphere, causing a warming effect of 0.011 ± 0.001 W m−2.
    ● These short-lived halogens reduce the formation of cooling aerosols, which are minute particles suspended in the atmosphere that reflect sunlight. It causes a small warming of 0.03 ± 0.01 W m−2.
    ● Though these halogens drive an increase in warming by influencing methane, water vapour and aerosols, they compensate this by destroying ozone, which exerts a cooling effect. Overall, the net cooling effect was found to be −0.13 ± 0.03 W m−2.

    Incorrect

    Answer: A
    Explanation:
    Recently, a study was published in the journal Nature which elaborately discussed the role of oceans in release of short-lived halogens.
    Statements 1 and 2 are incorrect- Short-lived halogens from oceans reduces warming by depleting ozone. Its cooling effect was found to be -0.24 ± 0.02 Watts per square metre (W m−2).
    Short-lived halogens have a lifetime of less than six months in the atmosphere and are naturally produced by the oceans. Examples of such halogens include such as chlorine, bromine and iodine. Currently, these halogens contribute 8-10 percent of cooling (and not global warming). This cooling could increase to 18-31 per cent by 2100, the study published in Nature projected.
    Human activities have amplified their release into the atmosphere. Human activities cause pollutants such as ozone to deposit on the ocean, which then convert the soluble short-lived halogens into insoluble ones, forcing them out of the sea water and into the atmosphere.
    Statement 3 is correct- However, their effect on methane is opposite. Short-lived halogens increase methane’s lifetime in the atmosphere by destroying hydroxyl radicals (OH). OH is a sink as it is known to break down this greenhouse gas.
    Additional information:
    ● These short-lived halogens increased global methane burden by 14 per cent and 9 per cent for pre-industrial and present-day conditions, respectively. This leads to a warming effect of 0.09 ± 0.01 W m−2 of warming.
    ● Similarly, these halogens increase the levels of water vapour, a greenhouse gas, in the atmosphere, causing a warming effect of 0.011 ± 0.001 W m−2.
    ● These short-lived halogens reduce the formation of cooling aerosols, which are minute particles suspended in the atmosphere that reflect sunlight. It causes a small warming of 0.03 ± 0.01 W m−2.
    ● Though these halogens drive an increase in warming by influencing methane, water vapour and aerosols, they compensate this by destroying ozone, which exerts a cooling effect. Overall, the net cooling effect was found to be −0.13 ± 0.03 W m−2.

  3. Question 3 of 5
    3. Question

    3. Consider the following statements:
    Statement I: In India, light pollution is regulated through the Air (Prevention and Control of Pollution) Act, 1981.
    Statement II: Light pollution causes hormonal imbalances in the marine animals.
    Which one of the following is correct in respect of the above statements?

    Correct

    Answer: D
    Explanation:
    Statement 1 is incorrect- In India, the National Lighting Code (NLC) published by the Bureau of Indian Standard (BIS) regulates the light pollution. No separate legislative measure on the lines of the Air (Prevention and Control of Pollution) Act, 1981 has been taken, as of now. The umbrella legislation for regulation of the environmental stresses such as pollution, degradation of environmental quality etc. is the Environment (Protection) Act, 1986.
    ● The National Lighting Code (NLC) has been formulated for the purpose of setting out in a convenient form the requirements for responsible social, commercial and engineering conduct for the designers, manufacturers and suppliers of lighting.
    ● The intent of this code is to encourage good lighting practices and systems which would minimize light pollution, glare, light trespass and conserve energy while maintaining safety, security, utility and productivity.
    ● This National Lighting Code is applicable to the lighting systems in large varieties of interior and exterior installations including special areas like hospitals, utilities, sports complex, metro railway etc. under the control of qualified persons.
    ● The code, however, does not specify the additional requirements to be considered while designing lightening system for certain areas such as marine, railway rolling stock, theatre, television and photography etc.
    ● The code also does not cover the requirements and methods of test applicable to light sources and luminaries for which separate Indian Standards exists.
    Statement 2 is correct- Marine organisms that evolved over millions of years to adapt to natural light now face an ever-increasing flood of light from anthropogenic sources along the coasts. Artificial light can easily wash out the glow of moonlight and starlight, which are important cues for marine organisms. This disrupts their hormonal cycles, inter-species behaviour and reproduction, noted the findings of the study published in the journal Aquatic Conservation: Marine and Freshwater Ecosystems.
    Compared to prior technologies, LEDs often emit more short-wavelength light and can pierce deep into the water.
    Additional information:
    Impacts Of Light Pollution in a Nutshell:
    ● Light pollution interrupts wildlife patterns and harms biodiversity: Migratory birds rely on natural light to guide them, and interruptions can lead to collisions and incorrect navigations on their long-distance journeys.
    ● Risks to human health: Artificial light can wreak havoc on natural body rhythms in both humans and animals. Nocturnal light interrupts sleep and confuses the circadian rhythm—the internal, twenty-four-hour clock that guides day and night activities and affects physiological processes in nearly all living organisms. One of these processes is the production of the hormone melatonin, which is released when it is dark and is inhibited when there is light present. An increased amount of light at night lowers melatonin production, which results in sleep deprivation, fatigue, headaches, stress, anxiety, and other health problems.
    ● Impact on astronomy: Artificial light in the sky causes ‘sky glow’ which tends to reduce visibility of stars and interferes with the astronomical studies.

    Incorrect

    Answer: D
    Explanation:
    Statement 1 is incorrect- In India, the National Lighting Code (NLC) published by the Bureau of Indian Standard (BIS) regulates the light pollution. No separate legislative measure on the lines of the Air (Prevention and Control of Pollution) Act, 1981 has been taken, as of now. The umbrella legislation for regulation of the environmental stresses such as pollution, degradation of environmental quality etc. is the Environment (Protection) Act, 1986.
    ● The National Lighting Code (NLC) has been formulated for the purpose of setting out in a convenient form the requirements for responsible social, commercial and engineering conduct for the designers, manufacturers and suppliers of lighting.
    ● The intent of this code is to encourage good lighting practices and systems which would minimize light pollution, glare, light trespass and conserve energy while maintaining safety, security, utility and productivity.
    ● This National Lighting Code is applicable to the lighting systems in large varieties of interior and exterior installations including special areas like hospitals, utilities, sports complex, metro railway etc. under the control of qualified persons.
    ● The code, however, does not specify the additional requirements to be considered while designing lightening system for certain areas such as marine, railway rolling stock, theatre, television and photography etc.
    ● The code also does not cover the requirements and methods of test applicable to light sources and luminaries for which separate Indian Standards exists.
    Statement 2 is correct- Marine organisms that evolved over millions of years to adapt to natural light now face an ever-increasing flood of light from anthropogenic sources along the coasts. Artificial light can easily wash out the glow of moonlight and starlight, which are important cues for marine organisms. This disrupts their hormonal cycles, inter-species behaviour and reproduction, noted the findings of the study published in the journal Aquatic Conservation: Marine and Freshwater Ecosystems.
    Compared to prior technologies, LEDs often emit more short-wavelength light and can pierce deep into the water.
    Additional information:
    Impacts Of Light Pollution in a Nutshell:
    ● Light pollution interrupts wildlife patterns and harms biodiversity: Migratory birds rely on natural light to guide them, and interruptions can lead to collisions and incorrect navigations on their long-distance journeys.
    ● Risks to human health: Artificial light can wreak havoc on natural body rhythms in both humans and animals. Nocturnal light interrupts sleep and confuses the circadian rhythm—the internal, twenty-four-hour clock that guides day and night activities and affects physiological processes in nearly all living organisms. One of these processes is the production of the hormone melatonin, which is released when it is dark and is inhibited when there is light present. An increased amount of light at night lowers melatonin production, which results in sleep deprivation, fatigue, headaches, stress, anxiety, and other health problems.
    ● Impact on astronomy: Artificial light in the sky causes ‘sky glow’ which tends to reduce visibility of stars and interferes with the astronomical studies.

  4. Question 4 of 5
    4. Question

    4. Which one of the following species has the habit of washing their face and limbs with their urine as a defence mechanism against the sting of the toxic insects which they eat?

    Correct

    Answer: C
    Explanation- Option C is the correct answer.
    Slender Lorris:

    ● Habitat and Distribution: Commonly found in the tropical scrub and deciduous forests as well as the dense hedgerow plantations bordering farmlands of Southern India and Sri Lanka, the Slender Loris is a small, nocturnal primate. It prefers to inhabit thick, thorny bushes and bamboo clumps where it can evade predators and also find insects, which is the main diet.
    ● Physical traits: These animals are about 25 cm long and have long, thin arms. They weigh around 275 grams. They have a small, vestigial tail. Their most prominent feature is the pair of two large, closely set, brown eyes.
    ● Behaviour: Being arboreal, they spend most of their life on the trees. Though their movements are slow, they can climb up fast to the tree top when threatened. They either hunt on their own or in pairs. They are known to be very social at dusk and dawn, interacting with others of their own.
    ● Reproduction: Their mating season is twice a year, from April to May as well as October to November. Gestation is 166-169 days. The females give birth to normally one and rarely two infants at one time. The mother carries the infants constantly during the first few weeks after birth. They live between 12-15 years.
    ● Diet: (Omnivores): Apart from insects they are also known to eat leaves, flowers, slugs and sometimes eggs of birds.
    ● Uniqueness: Among the strange habits they have is the urine washing of their face and limbs, which is thought to soothe or defend against the sting of the toxic insects they prefer to eat.
    ● Threats: These animals face a threat from poachers due to the misplaced belief that these animals have magical and medicinal powers. This hunting, along with destruction of their habitat, is their major threat.
    ● Conservation status: IUCN has listed them as ‘Endangered’, whereas they are listed under the Schedule I of the Wildlife (Protection) Act of India, 1972, according them the highest level of legal protection.
    About other species given in the options:
    Himalayan Musk Deer
    ● Distribution:The Himalayas from central Kashmir through Himachal Pradesh up to Sikkim.
    ● Habitat: Subalpine oak and rhododendron forests ( 3000-4300 m) medium to high elevation coniferous forest in the Western Himalayas below the tree line ( 2500-4400 m) in Sikkim, thick bamboo forests below alpine zone ( 2600-300 m.
    ● Unique Behaviour: Communication is chiefly by olfaction. The deer scent is marked by defecation and secretion of caudal, musk and inter digital glands.
    ● Solitary, except when female is with young.
    ● IUCN status: Endangered (EN); Schedule 1 of Wildlife (Protection) Act, 1972.

    Nilgiri Tahr
    ● The Nilgiri Tahr is the only mountain ungulate in southern India amongst the 12 species present in India.
    ● It is also the state animal of Tamil Nadu.
    ● The Nilgiri Tahr is a sure-footed ungulate that inhabits the open montane grassland habitats at elevations from 1200 to 2600 m of the South Western Ghats.
    ● The Nilgiri Tahr, which used to be found along the entire stretch of Western Ghats, is presently found only in small fragmented pockets.
    ● The Eravikulam National Park in Anamalai hills, Kerala, is home to the largest population of the Nilgiri tahr, with more than 700 individuals.
    ● It faces several threats such as habitat loss due to rampant deforestation, competition with domestic livestock, hydroelectric projects in Nilgiri tahr habitat, and monoculture plantations and occasional hunting for its meat and skin.
    ● This endemic species of the Western Ghats is listed as ‘Endangered’ in the IUCN Red List of Threatened Species and is protected under Schedule I of the Wildlife (Protection) Act of India, 1972.

    Himalayan serow
    ● Official state animal of the Indian state of Mizoram.
    ● Native to the Himalayas where they inhabit hilly forests above an elevation of 300 m (980 ft) but descend to 100 m (330 ft) in winter.
    ● Himalayan serows are territorial animals that live alone or in small groups.
    ● They are active during the day (not nocturnal), and have acute vision, smell, and hearing, and are wary of humans.
    ● They move with ease through the difficult terrain of mountain and hill slopes and also swim well.
    ● Lifestyle: viviparous and terrestrial
    ● Himalayan serows are herbivores (folivores) and browse various types of vegetation.
    ● After the gestation period of 6 to 7 months, females give birth to a single kid; it is fully developed at birth and is able to travel with its mother shortly after birth.
    ● IUCN status: Vulnerable (VU)

    Incorrect

    Answer: C
    Explanation- Option C is the correct answer.
    Slender Lorris:

    ● Habitat and Distribution: Commonly found in the tropical scrub and deciduous forests as well as the dense hedgerow plantations bordering farmlands of Southern India and Sri Lanka, the Slender Loris is a small, nocturnal primate. It prefers to inhabit thick, thorny bushes and bamboo clumps where it can evade predators and also find insects, which is the main diet.
    ● Physical traits: These animals are about 25 cm long and have long, thin arms. They weigh around 275 grams. They have a small, vestigial tail. Their most prominent feature is the pair of two large, closely set, brown eyes.
    ● Behaviour: Being arboreal, they spend most of their life on the trees. Though their movements are slow, they can climb up fast to the tree top when threatened. They either hunt on their own or in pairs. They are known to be very social at dusk and dawn, interacting with others of their own.
    ● Reproduction: Their mating season is twice a year, from April to May as well as October to November. Gestation is 166-169 days. The females give birth to normally one and rarely two infants at one time. The mother carries the infants constantly during the first few weeks after birth. They live between 12-15 years.
    ● Diet: (Omnivores): Apart from insects they are also known to eat leaves, flowers, slugs and sometimes eggs of birds.
    ● Uniqueness: Among the strange habits they have is the urine washing of their face and limbs, which is thought to soothe or defend against the sting of the toxic insects they prefer to eat.
    ● Threats: These animals face a threat from poachers due to the misplaced belief that these animals have magical and medicinal powers. This hunting, along with destruction of their habitat, is their major threat.
    ● Conservation status: IUCN has listed them as ‘Endangered’, whereas they are listed under the Schedule I of the Wildlife (Protection) Act of India, 1972, according them the highest level of legal protection.
    About other species given in the options:
    Himalayan Musk Deer
    ● Distribution:The Himalayas from central Kashmir through Himachal Pradesh up to Sikkim.
    ● Habitat: Subalpine oak and rhododendron forests ( 3000-4300 m) medium to high elevation coniferous forest in the Western Himalayas below the tree line ( 2500-4400 m) in Sikkim, thick bamboo forests below alpine zone ( 2600-300 m.
    ● Unique Behaviour: Communication is chiefly by olfaction. The deer scent is marked by defecation and secretion of caudal, musk and inter digital glands.
    ● Solitary, except when female is with young.
    ● IUCN status: Endangered (EN); Schedule 1 of Wildlife (Protection) Act, 1972.

    Nilgiri Tahr
    ● The Nilgiri Tahr is the only mountain ungulate in southern India amongst the 12 species present in India.
    ● It is also the state animal of Tamil Nadu.
    ● The Nilgiri Tahr is a sure-footed ungulate that inhabits the open montane grassland habitats at elevations from 1200 to 2600 m of the South Western Ghats.
    ● The Nilgiri Tahr, which used to be found along the entire stretch of Western Ghats, is presently found only in small fragmented pockets.
    ● The Eravikulam National Park in Anamalai hills, Kerala, is home to the largest population of the Nilgiri tahr, with more than 700 individuals.
    ● It faces several threats such as habitat loss due to rampant deforestation, competition with domestic livestock, hydroelectric projects in Nilgiri tahr habitat, and monoculture plantations and occasional hunting for its meat and skin.
    ● This endemic species of the Western Ghats is listed as ‘Endangered’ in the IUCN Red List of Threatened Species and is protected under Schedule I of the Wildlife (Protection) Act of India, 1972.

    Himalayan serow
    ● Official state animal of the Indian state of Mizoram.
    ● Native to the Himalayas where they inhabit hilly forests above an elevation of 300 m (980 ft) but descend to 100 m (330 ft) in winter.
    ● Himalayan serows are territorial animals that live alone or in small groups.
    ● They are active during the day (not nocturnal), and have acute vision, smell, and hearing, and are wary of humans.
    ● They move with ease through the difficult terrain of mountain and hill slopes and also swim well.
    ● Lifestyle: viviparous and terrestrial
    ● Himalayan serows are herbivores (folivores) and browse various types of vegetation.
    ● After the gestation period of 6 to 7 months, females give birth to a single kid; it is fully developed at birth and is able to travel with its mother shortly after birth.
    ● IUCN status: Vulnerable (VU)

  5. Question 5 of 5
    5. Question

    5. Consider the following statements about jellyfish:
    1. In spite of having no central brain, they can learn from the past experiences like humans.
    2. They have a single organ for eating and excreting.
    3. They do not have scales, gills, or fins like fish.
    4. Some of them have the capacity for bioluminescence.
    How many of the above statements are correct?

    Correct

    Answer: D
    Explanation: Pulsing along on our ocean currents, jellyfish can be found in waters both cold and warm, deep and shallow and along coastlines, too. Jellyfish have been around for millions of years, even before dinosaurs lived on the Earth.
    Statement 1 is correct: Even without a central brain, jellyfish can learn from past experiences like humans, mice, and flies. The researchers trained Caribbean box jellyfish (Tripedalia cystophora) to learn to spot and dodge obstacles.
    The study published in ‘Current Biology’ has challenged previous notions that advanced learning requires a centralised brain and sheds light on the evolutionary roots of learning and memory. No bigger than a fingernail, these seemingly simple jellies have a complex visual system with 24 eyes embedded in their bell-like body. Scientists demonstrated that the jellies could acquire the ability to avoid obstacles through associative learning, a process through which organisms form mental connections between sensory stimulations and behaviours.
    Statement 2 is correct: The jellyfish’s mouth is found in the centre of its body. From this small opening it both eats and discards waste. And it serves another purpose, too – by squirting a jet of water from its mouth, the jellyfish can propel forward. Therefore, the jellyfish is known as a simple or “primitive” animal.
    Statement 3 is correct: They aren’t actually fish. They are invertebrates from the phylum Cnidaria and are so varied as a taxonomic group that many scientists have taken to simply referring to them as “gelatinous zooplankton.” Jellyfish do not have scales, gills, or fins like fish. Instead, they swim by opening and closing their “bells.”
    Statement 4 is correct: Some jellyfish are clear, but others are vibrant colours of pink, yellow, blue and purple. They can be bioluminescent, too, which means they produce their own light!
    Additional information:
    ● The jellyfish itself provides a tasty meal for other ocean creatures, particularly sea turtles.
    ● Unlike most marine creatures, jellyfish are thriving in our oceans despite marine heat waves, ocean acidification, overfishing, and various other human influences. While corals, oysters, and any marine organisms that build shells are considered the biggest losers of increasingly acidic oceans, jellyfish don’t seem to be as susceptible to climate change.
    ● When threatened, a species of jellyfish namely ‘Turritopsis nutricula’ is capable of undergoing cellular transdifferentiation, a process whereby the organism’s cells essentially become new again.

    Incorrect

    Answer: D
    Explanation: Pulsing along on our ocean currents, jellyfish can be found in waters both cold and warm, deep and shallow and along coastlines, too. Jellyfish have been around for millions of years, even before dinosaurs lived on the Earth.
    Statement 1 is correct: Even without a central brain, jellyfish can learn from past experiences like humans, mice, and flies. The researchers trained Caribbean box jellyfish (Tripedalia cystophora) to learn to spot and dodge obstacles.
    The study published in ‘Current Biology’ has challenged previous notions that advanced learning requires a centralised brain and sheds light on the evolutionary roots of learning and memory. No bigger than a fingernail, these seemingly simple jellies have a complex visual system with 24 eyes embedded in their bell-like body. Scientists demonstrated that the jellies could acquire the ability to avoid obstacles through associative learning, a process through which organisms form mental connections between sensory stimulations and behaviours.
    Statement 2 is correct: The jellyfish’s mouth is found in the centre of its body. From this small opening it both eats and discards waste. And it serves another purpose, too – by squirting a jet of water from its mouth, the jellyfish can propel forward. Therefore, the jellyfish is known as a simple or “primitive” animal.
    Statement 3 is correct: They aren’t actually fish. They are invertebrates from the phylum Cnidaria and are so varied as a taxonomic group that many scientists have taken to simply referring to them as “gelatinous zooplankton.” Jellyfish do not have scales, gills, or fins like fish. Instead, they swim by opening and closing their “bells.”
    Statement 4 is correct: Some jellyfish are clear, but others are vibrant colours of pink, yellow, blue and purple. They can be bioluminescent, too, which means they produce their own light!
    Additional information:
    ● The jellyfish itself provides a tasty meal for other ocean creatures, particularly sea turtles.
    ● Unlike most marine creatures, jellyfish are thriving in our oceans despite marine heat waves, ocean acidification, overfishing, and various other human influences. While corals, oysters, and any marine organisms that build shells are considered the biggest losers of increasingly acidic oceans, jellyfish don’t seem to be as susceptible to climate change.
    ● When threatened, a species of jellyfish namely ‘Turritopsis nutricula’ is capable of undergoing cellular transdifferentiation, a process whereby the organism’s cells essentially become new again.

window.wpAdvQuizInitList = window.wpAdvQuizInitList || []; window.wpAdvQuizInitList.push({ id: '#wpAdvQuiz_586', init: { quizId: 586, mode: 0, globalPoints: 10, timelimit: 0, resultsGrade: [0], bo: 0, qpp: 0, catPoints: [10], formPos: 0, lbn: "Finish quiz", json: {"2724":{"type":"single","id":2724,"catId":0,"points":2,"correct":[1,0,0,0]},"2725":{"type":"single","id":2725,"catId":0,"points":2,"correct":[1,0,0,0]},"2726":{"type":"single","id":2726,"catId":0,"points":2,"correct":[0,0,0,1]},"2727":{"type":"single","id":2727,"catId":0,"points":2,"correct":[0,0,1,0]},"2728":{"type":"single","id":2728,"catId":0,"points":2,"correct":[0,0,0,1]}} } });




Day-546 | Daily MCQs | UPSC Prelims | POLITY

Day-546

Time limit: 0

Quiz-summary

0 of 5 questions completed

Questions:

  1. 1
  2. 2
  3. 3
  4. 4
  5. 5

Information

DAILY MCQ

You have already completed the quiz before. Hence you can not start it again.

Quiz is loading...

You must sign in or sign up to start the quiz.

You have to finish following quiz, to start this quiz:

Results

0 of 5 questions answered correctly

Your time:

Time has elapsed

You have reached 0 of 0 points, (0)

Categories

  1. Not categorized 0%
  1. 1
  2. 2
  3. 3
  4. 4
  5. 5
  1. Answered
  2. Review
  1. Question 1 of 5
    1. Question

    1. Consider the following statements:
    Statement-I: MPs cannot be drawn to courts and subjected to penal law for their speeches made in the Parliament.
    Statement-II: The MPs are protected by the Constitution from any proceedings in a court for their speeches made in the Parliament.
    Which one of the following is correct in respect of the above statements?

    Correct

    Answer: A
    Explanation:
    Statement-I is correct:
    Article 105 of the Constitution states:
    1) Subject to the provisions of this Constitution and to the rules and standing orders regulating the procedure of Parliament, there shall be freedom of speech in Parliament.
    2) No Member of Parliament shall be liable to any proceedings in any court in respect of anything said or any vote given by him in Parliament or any committee thereof, and no person shall be so liable in respect of the publication by or under the authority of either House of Parliament of any report, paper, votes or proceedings. Hence Statement-II is also correct.
    Article 105(1) and Article 105 (2) of the Constitution of India makes it clear that MPs have freedom of speech and they are protected by the Constitution from any proceedings in a court for what they say or how they vote. However, their speeches are subjected to Rules of Procedure and Conduct of Business of both Houses of Parliament, and the Chair can decide to enforce those rules on them for the orderly conduct of the proceedings of the Parliament. A set of such rules deals with the expunction of words of the speeches they deliver inside the Houses of the Parliament.
    Both the Houses of Parliament have specific rules in their respective Rules of Procedure and Conduct of Business.
    Rule 261 of the Rajya Sabha rules and Rule 380 of the Lok Sabha rules provide that if the presiding officer is of the opinion that a word or words has or have been used in a debate which is or are defamatory or indecent or unparliamentary or undignified, they may, in their discretion, order that such word or words be expunged from the proceedings.
    It is clear that the Chair (Speaker in the Lok Sabha and Chairman in Rajya Sabha) exercises discretion in taking the decision to expunge certain words from the proceedings of the Lok Sabha or Rajya Sabha, as the case may be, and that decision is final.
    Both Statement-I and Statement-II are correct and Statement-II is the correct explanation for Statement-I

    Incorrect

    Answer: A
    Explanation:
    Statement-I is correct:
    Article 105 of the Constitution states:
    1) Subject to the provisions of this Constitution and to the rules and standing orders regulating the procedure of Parliament, there shall be freedom of speech in Parliament.
    2) No Member of Parliament shall be liable to any proceedings in any court in respect of anything said or any vote given by him in Parliament or any committee thereof, and no person shall be so liable in respect of the publication by or under the authority of either House of Parliament of any report, paper, votes or proceedings. Hence Statement-II is also correct.
    Article 105(1) and Article 105 (2) of the Constitution of India makes it clear that MPs have freedom of speech and they are protected by the Constitution from any proceedings in a court for what they say or how they vote. However, their speeches are subjected to Rules of Procedure and Conduct of Business of both Houses of Parliament, and the Chair can decide to enforce those rules on them for the orderly conduct of the proceedings of the Parliament. A set of such rules deals with the expunction of words of the speeches they deliver inside the Houses of the Parliament.
    Both the Houses of Parliament have specific rules in their respective Rules of Procedure and Conduct of Business.
    Rule 261 of the Rajya Sabha rules and Rule 380 of the Lok Sabha rules provide that if the presiding officer is of the opinion that a word or words has or have been used in a debate which is or are defamatory or indecent or unparliamentary or undignified, they may, in their discretion, order that such word or words be expunged from the proceedings.
    It is clear that the Chair (Speaker in the Lok Sabha and Chairman in Rajya Sabha) exercises discretion in taking the decision to expunge certain words from the proceedings of the Lok Sabha or Rajya Sabha, as the case may be, and that decision is final.
    Both Statement-I and Statement-II are correct and Statement-II is the correct explanation for Statement-I

  2. Question 2 of 5
    2. Question

    2. Consider the following statements:
    1. Citizenship is under the exclusive jurisdiction of Parliament.
    2. The first National Register of Citizens was prepared after the Census of 1951.
    3. The National Register of Citizens has not been updated for the entire country since 1951.
    How many of the above given statements are correct?

    Correct

    Answer: C
    Explanation:
    Statement 1 is correct: Citizenship is listed in the Union List under the Constitution and thus is under the exclusive jurisdiction of Parliament. (Entry 17 of the Union List in Schedule 7)
    Statement 2 is correct: National Register of Citizens, 1951 is a register prepared after the conduct of the Census of 1951 in respect of each village, showing the houses or holdings in a serial order and indicating against each house or holding the number and names of persons staying therein.
    Statement 3 is correct: The NRC was published only once in 1951 and since then, it has not been updated for the entire country.
    However, the Assam NRC of 1951 was updated to detect illegal immigrants and the entire process, carried out between 2014 and 2019, was monitored by the Supreme Court. The final NRC list, published in August 2019, found that 1.9 million of the 33 million applicants in the state were not eligible for Indian citizenship.
    Having said that, it is also important to note that after the report was submitted in December 2022, the Comptroller and Auditor General (CAG) of India detected large-scale anomalies in the process of updating the National Register of Citizens (NRC) for Assam. The CAG report practically sounded the death knell for the NRC exercise as it officially declares that “the intended objective of preparing a valid, error-free NRC has not been met”.

    Incorrect

    Answer: C
    Explanation:
    Statement 1 is correct: Citizenship is listed in the Union List under the Constitution and thus is under the exclusive jurisdiction of Parliament. (Entry 17 of the Union List in Schedule 7)
    Statement 2 is correct: National Register of Citizens, 1951 is a register prepared after the conduct of the Census of 1951 in respect of each village, showing the houses or holdings in a serial order and indicating against each house or holding the number and names of persons staying therein.
    Statement 3 is correct: The NRC was published only once in 1951 and since then, it has not been updated for the entire country.
    However, the Assam NRC of 1951 was updated to detect illegal immigrants and the entire process, carried out between 2014 and 2019, was monitored by the Supreme Court. The final NRC list, published in August 2019, found that 1.9 million of the 33 million applicants in the state were not eligible for Indian citizenship.
    Having said that, it is also important to note that after the report was submitted in December 2022, the Comptroller and Auditor General (CAG) of India detected large-scale anomalies in the process of updating the National Register of Citizens (NRC) for Assam. The CAG report practically sounded the death knell for the NRC exercise as it officially declares that “the intended objective of preparing a valid, error-free NRC has not been met”.

  3. Question 3 of 5
    3. Question

    3. Consider the following organizations:
    1. NITI Aayog
    2. Zonal Council
    3. Inter-State Council
    How many of the above are Constitutional bodies meant for securing the cooperation between the States in India?

    Correct

    Answer: A
    Explanations:
    NITI Aayog: NITI Aayog has the twin mandate to oversee the adoption and monitoring of the SDGs in the country and promote competitive and cooperative federalism among States and UTs.
    The task at hand for NITI Aayog is not just to periodically collect data on SDGs but to proactively realise the goals and targets. The Ministry of Statistics and Programme Implementation (MoSPI) has already undertaken a parallel exercise of interaction with the Ministries to evolve indicators reflecting the SDG goals and targets.
    Key Objectives:
    ● To foster cooperative federalism through structured support initiatives and mechanisms with the States on a continuous basis, recognizing that strong States make a strong nation.
    ● To develop mechanisms to formulate credible plans at the village level and aggregate these progressively at higher levels of government.
    ● To ensure, on areas that are specifically referred to it, that the interests of national security are incorporated in economic strategy and policy.
    ● To pay special attention to the sections of our society that may be at risk of not benefitting adequately from economic progress.
    ● To provide advice and encourage partnerships between key stakeholders and national and international like-minded Think Tanks, as well as educational and policy research institutions.
    ● To create a knowledge, innovation and entrepreneurial support system through a collaborative community of national and international experts, practitioners and other partners.
    ● To offer a platform for resolution of inter-sectoral and inter-departmental issues in order to accelerate the implementation of the development agenda.
    ● To maintain a state-of-the-art Resource Centre, be a repository of research on good governance and best practices in sustainable and equitable development as well as help their dissemination to stake-holders.
    Zonal Councils: Zonal Councils were formed in 1957 to promote regional cooperation among states within five designated zones. They serve as platforms for exchanging information, sharing experiences, and coordinating efforts on issues of regional significance, such as infrastructure development, water resource management, and disaster management.
    The creation and functioning of Zonal Councils are not directly mandated by the Constitution of India. They were established by the States Reorganization Act, 1956, an Act of Parliament, rather than the Constitution itself.
    Zonal Councils are intended to:
    1. Foster cooperation and coordination among states within a particular zone.
    2. Discuss and make recommendations on matters of common concern to the states in the zone.
    3. Promote uniformity of legislation and policy among states within the zone.
    4. Provide a forum for exchanging information and sharing experiences among states in the zone.
    Inter-State Council (ISC): Inter-State Councils are created to promote the cooperative federalism in the country. It acts as a mechanism for negotiation, mediation, and conciliation, aiming to amicably settle inter-state conflicts and promote harmonious coexistence.
    The establishment of ISC is consistent with the principles of federalism and cooperation enshrined in the Constitution. The Constitution provides a framework for inter-state relations, including the establishment of mechanisms for resolving inter-state disputes and promoting cooperation on matters of common interest.
    Article 263 of the Indian Constitution empowers the President to establish an Inter-State Council (ISC) if they deem it necessary for the public interest. The article states that the ISC’s duties and organization will be defined by the President’s order.
    Article 263 provides a constitutional basis for the establishment of the ISC, although it doesn’t explicitly mandate its creation. This flexibility allows the government to establish the ISC when it considers it necessary for promoting inter-state cooperation and addressing common issues.
    The ISC serves an advisory body that aims to foster cooperation and coordination between the states of India. It provides a platform for dialogue, consensus-building, and joint decision-making on matters of common interest to the states.

    Incorrect

    Answer: A
    Explanations:
    NITI Aayog: NITI Aayog has the twin mandate to oversee the adoption and monitoring of the SDGs in the country and promote competitive and cooperative federalism among States and UTs.
    The task at hand for NITI Aayog is not just to periodically collect data on SDGs but to proactively realise the goals and targets. The Ministry of Statistics and Programme Implementation (MoSPI) has already undertaken a parallel exercise of interaction with the Ministries to evolve indicators reflecting the SDG goals and targets.
    Key Objectives:
    ● To foster cooperative federalism through structured support initiatives and mechanisms with the States on a continuous basis, recognizing that strong States make a strong nation.
    ● To develop mechanisms to formulate credible plans at the village level and aggregate these progressively at higher levels of government.
    ● To ensure, on areas that are specifically referred to it, that the interests of national security are incorporated in economic strategy and policy.
    ● To pay special attention to the sections of our society that may be at risk of not benefitting adequately from economic progress.
    ● To provide advice and encourage partnerships between key stakeholders and national and international like-minded Think Tanks, as well as educational and policy research institutions.
    ● To create a knowledge, innovation and entrepreneurial support system through a collaborative community of national and international experts, practitioners and other partners.
    ● To offer a platform for resolution of inter-sectoral and inter-departmental issues in order to accelerate the implementation of the development agenda.
    ● To maintain a state-of-the-art Resource Centre, be a repository of research on good governance and best practices in sustainable and equitable development as well as help their dissemination to stake-holders.
    Zonal Councils: Zonal Councils were formed in 1957 to promote regional cooperation among states within five designated zones. They serve as platforms for exchanging information, sharing experiences, and coordinating efforts on issues of regional significance, such as infrastructure development, water resource management, and disaster management.
    The creation and functioning of Zonal Councils are not directly mandated by the Constitution of India. They were established by the States Reorganization Act, 1956, an Act of Parliament, rather than the Constitution itself.
    Zonal Councils are intended to:
    1. Foster cooperation and coordination among states within a particular zone.
    2. Discuss and make recommendations on matters of common concern to the states in the zone.
    3. Promote uniformity of legislation and policy among states within the zone.
    4. Provide a forum for exchanging information and sharing experiences among states in the zone.
    Inter-State Council (ISC): Inter-State Councils are created to promote the cooperative federalism in the country. It acts as a mechanism for negotiation, mediation, and conciliation, aiming to amicably settle inter-state conflicts and promote harmonious coexistence.
    The establishment of ISC is consistent with the principles of federalism and cooperation enshrined in the Constitution. The Constitution provides a framework for inter-state relations, including the establishment of mechanisms for resolving inter-state disputes and promoting cooperation on matters of common interest.
    Article 263 of the Indian Constitution empowers the President to establish an Inter-State Council (ISC) if they deem it necessary for the public interest. The article states that the ISC’s duties and organization will be defined by the President’s order.
    Article 263 provides a constitutional basis for the establishment of the ISC, although it doesn’t explicitly mandate its creation. This flexibility allows the government to establish the ISC when it considers it necessary for promoting inter-state cooperation and addressing common issues.
    The ISC serves an advisory body that aims to foster cooperation and coordination between the states of India. It provides a platform for dialogue, consensus-building, and joint decision-making on matters of common interest to the states.

  4. Question 4 of 5
    4. Question

    4. Consider the following statements regarding the relationship between the Parliament and the President of India:
    1. The President can use Suspensive veto on ordinary bills passed by the Parliament but is obligatory to give assent when the Bill is passed again by the Parliament.
    2. The President can appoint the Prime Minister and other ministers, but they are collectively responsible to the Parliament.
    3. The power of the President to dissolve the Parliament is a crucial aspect of the Indian Constitution’s system of checks and balances.
    How many of the statements given above are correct?

    Correct

    Answer: A
    Explanation:
    • Statement 1 is correct: The President has the power to return a bill passed by Parliament for reconsideration of the Parliament (Suspensive Veto). If Parliament passes the bill again with or without the amendments and again presented to the President, it is obligatory for the President to give assent to the Bill. This means that the Suspensive veto of the President is overridden by the re-passage of the Bill.
    • Statement 2 is incorrect: The President appoints the Prime Minister as the leader of the majority party in Parliament. The Prime Minister and other ministers are collectively responsible to the Lok Sabha (Article 75 of the Constitution of India).
    • Statement 3 is incorrect: While the President does have the power to dissolve the Lok Sabha (the lower house of Parliament), this power is not without limitations. The President can only dissolve the Lok Sabha on the advice of the Prime Minister, who is the leader of the majority party in Parliament. The President cannot dissolve the Rajya Sabha (the upper house of Parliament) at all.

    Incorrect

    Answer: A
    Explanation:
    • Statement 1 is correct: The President has the power to return a bill passed by Parliament for reconsideration of the Parliament (Suspensive Veto). If Parliament passes the bill again with or without the amendments and again presented to the President, it is obligatory for the President to give assent to the Bill. This means that the Suspensive veto of the President is overridden by the re-passage of the Bill.
    • Statement 2 is incorrect: The President appoints the Prime Minister as the leader of the majority party in Parliament. The Prime Minister and other ministers are collectively responsible to the Lok Sabha (Article 75 of the Constitution of India).
    • Statement 3 is incorrect: While the President does have the power to dissolve the Lok Sabha (the lower house of Parliament), this power is not without limitations. The President can only dissolve the Lok Sabha on the advice of the Prime Minister, who is the leader of the majority party in Parliament. The President cannot dissolve the Rajya Sabha (the upper house of Parliament) at all.

  5. Question 5 of 5
    5. Question

    5. Consider the following statements regarding the Preamble of the Constitution of India:
    1. The Preamble serves as a guiding light for the interpretation of other constitutional provisions.
    2. The Preamble can be invoked to challenge laws or actions of the government that violate its fundamental principles.
    3. The Preamble embodies the essence of fundamental rights and freedoms guaranteed to Indian citizens.
    How many of the above given statements are correct?

    Correct

    Answer: B
    Explanation:
    Statement 1 is correct: While not directly enforceable in courts, the Preamble’s principles provide a framework for interpreting the Constitution’s other provisions. When there is ambiguity or uncertainty in the interpretation of a particular provision, courts consider the Preamble’s overarching principles to determine the intended meaning and application of that provision.
    Statement 2 is incorrect: The Preamble to the Constitution of India, though not directly enforceable in courts, plays a crucial role in shaping the interpretation of other constitutional provisions and influencing the government’s actions. While it doesn’t directly provide specific legal remedies, its principles serve as a guiding light for the judiciary and the government, ensuring that their decisions and actions align with the fundamental values enshrined in the Constitution.
    Though the Preamble holds immense moral and constitutional authority, it is not considered a legally enforceable part of the Constitution in the same way as the justiciable provisions. Its principles guide the interpretation of other constitutional provisions, but they cannot be directly invoked to challenge laws or actions of the government.
    Statement 3 is correct: While the Preamble to the Constitution of India does not explicitly enumerate the specific fundamental rights, it embodies the essence of these rights and serves as a foundation for their legal protection. The Preamble’s commitment to democracy, socialism, secularism, and justice forms the bedrock of India’s legal framework, ensuring that fundamental rights are enshrined and protected.
    The Preamble’s Role in Establishing Fundamental Rights
    • Commitment to Democracy: The Preamble’s declaration of India as a “democratic republic” establishes the fundamental right of citizens to participate in the political process, including the right to vote, hold office, and express dissent.
    • Socialism and Economic Rights: The Preamble’s emphasis on “socialism” underscores the importance of social justice and economic equality. This translates into fundamental rights such as the right to work, education, and healthcare.
    • Secularism and Freedom of Religion: The Preamble’s affirmation of India as a “secular state” guarantees freedom of religion for all citizens. This fundamental right allows individuals to practice their faith without fear of discrimination or coercion.
    • Justice and Equality: The Preamble’s aspiration to secure “justice, social, economic and political; liberty of thought, expression, belief, faith and worship” enshrines the fundamental rights to equality, liberty, and justice. These rights protect individuals from discrimination, arbitrary detention, and unfair treatment.
    Indirect Influence of the Preamble to the Constitution of India on Legal Protection of Fundamental Rights:
    While the Preamble does not directly provide specific legal remedies for the violation of fundamental rights, its principles serve as a guiding framework for the judiciary. When interpreting and applying constitutional provisions related to fundamental rights, courts consider the Preamble’s overarching principles to ensure that the rights are upheld and protected.

    Incorrect

    Answer: B
    Explanation:
    Statement 1 is correct: While not directly enforceable in courts, the Preamble’s principles provide a framework for interpreting the Constitution’s other provisions. When there is ambiguity or uncertainty in the interpretation of a particular provision, courts consider the Preamble’s overarching principles to determine the intended meaning and application of that provision.
    Statement 2 is incorrect: The Preamble to the Constitution of India, though not directly enforceable in courts, plays a crucial role in shaping the interpretation of other constitutional provisions and influencing the government’s actions. While it doesn’t directly provide specific legal remedies, its principles serve as a guiding light for the judiciary and the government, ensuring that their decisions and actions align with the fundamental values enshrined in the Constitution.
    Though the Preamble holds immense moral and constitutional authority, it is not considered a legally enforceable part of the Constitution in the same way as the justiciable provisions. Its principles guide the interpretation of other constitutional provisions, but they cannot be directly invoked to challenge laws or actions of the government.
    Statement 3 is correct: While the Preamble to the Constitution of India does not explicitly enumerate the specific fundamental rights, it embodies the essence of these rights and serves as a foundation for their legal protection. The Preamble’s commitment to democracy, socialism, secularism, and justice forms the bedrock of India’s legal framework, ensuring that fundamental rights are enshrined and protected.
    The Preamble’s Role in Establishing Fundamental Rights
    • Commitment to Democracy: The Preamble’s declaration of India as a “democratic republic” establishes the fundamental right of citizens to participate in the political process, including the right to vote, hold office, and express dissent.
    • Socialism and Economic Rights: The Preamble’s emphasis on “socialism” underscores the importance of social justice and economic equality. This translates into fundamental rights such as the right to work, education, and healthcare.
    • Secularism and Freedom of Religion: The Preamble’s affirmation of India as a “secular state” guarantees freedom of religion for all citizens. This fundamental right allows individuals to practice their faith without fear of discrimination or coercion.
    • Justice and Equality: The Preamble’s aspiration to secure “justice, social, economic and political; liberty of thought, expression, belief, faith and worship” enshrines the fundamental rights to equality, liberty, and justice. These rights protect individuals from discrimination, arbitrary detention, and unfair treatment.
    Indirect Influence of the Preamble to the Constitution of India on Legal Protection of Fundamental Rights:
    While the Preamble does not directly provide specific legal remedies for the violation of fundamental rights, its principles serve as a guiding framework for the judiciary. When interpreting and applying constitutional provisions related to fundamental rights, courts consider the Preamble’s overarching principles to ensure that the rights are upheld and protected.

window.wpAdvQuizInitList = window.wpAdvQuizInitList || []; window.wpAdvQuizInitList.push({ id: '#wpAdvQuiz_585', init: { quizId: 585, mode: 0, globalPoints: 10, timelimit: 0, resultsGrade: [0], bo: 0, qpp: 0, catPoints: [10], formPos: 0, lbn: "Finish quiz", json: {"2719":{"type":"single","id":2719,"catId":0,"points":2,"correct":[1,0,0,0]},"2720":{"type":"single","id":2720,"catId":0,"points":2,"correct":[0,0,1,0]},"2721":{"type":"single","id":2721,"catId":0,"points":2,"correct":[1,0,0,0]},"2722":{"type":"single","id":2722,"catId":0,"points":2,"correct":[1,0,0,0]},"2723":{"type":"single","id":2723,"catId":0,"points":2,"correct":[0,1,0,0]}} } });




Day-545 | Daily MCQs | UPSC Prelims | HISTORY

Day-545

Time limit: 0

Quiz-summary

0 of 5 questions completed

Questions:

  1. 1
  2. 2
  3. 3
  4. 4
  5. 5

Information

DAILY MCQ

You have already completed the quiz before. Hence you can not start it again.

Quiz is loading...

You must sign in or sign up to start the quiz.

You have to finish following quiz, to start this quiz:

Results

0 of 5 questions answered correctly

Your time:

Time has elapsed

You have reached 0 of 0 points, (0)

Categories

  1. Not categorized 0%
  1. 1
  2. 2
  3. 3
  4. 4
  5. 5
  1. Answered
  2. Review
  1. Question 1 of 5
    1. Question

    1. Consider the following statements in the context of early history of bhakti movement of South India:
    1. The movement offered sharp criticism to Buddhists and Jains.
    2. It sought to change the prevailing social order by offering a powerful critique against it.
    3. Pulaiyar and Panars were prominent groups who played active roles in the movement.
    How many statements given above are correctly matched?

    Correct

    Answer: B
    Explanation:
    Statement 1 is correct: The seventh to ninth centuries saw the emergence of new religious movements, led by the Nayanars and the Alvars which came to be known as the bhakti saints. The South Indian bhakti saints used to criticize the Jains and Buddhists who enjoyed a privileged status at the courts of South Indian monarchs during that period.
    Statement 2 is incorrect: The South Indian bhakti movement also had drawbacks. It never consciously opposed Brahmanism or the varna and jati systems at the social level. It was integrated with the caste system and the lower castes continued to suffer from myriad social disabilities. There was no elimination of Brahmanical rituals such as worship of idols, recitation of the Vedic mantras and pilgrimages to sacred places in spite of the overriding stress on bhakti as the superior mode of worship. The Jains and Buddhists were its principal targets, not the Brahmins. As a result, the bhakti movement of the south in the long run strengthened the hierarchical system rather than weakening it. So, the bhakti movement critiqued the oppression of the social order but never the social order itself.
    Statement 3 is correct: The importance of the bhakti movement lies in its providing a powerful ideology and the integration of various social groups into its fold. It included saint-poets who came from all jati including those considered “untouchable” like the Pulaiyar and Panars.

    Incorrect

    Answer: B
    Explanation:
    Statement 1 is correct: The seventh to ninth centuries saw the emergence of new religious movements, led by the Nayanars and the Alvars which came to be known as the bhakti saints. The South Indian bhakti saints used to criticize the Jains and Buddhists who enjoyed a privileged status at the courts of South Indian monarchs during that period.
    Statement 2 is incorrect: The South Indian bhakti movement also had drawbacks. It never consciously opposed Brahmanism or the varna and jati systems at the social level. It was integrated with the caste system and the lower castes continued to suffer from myriad social disabilities. There was no elimination of Brahmanical rituals such as worship of idols, recitation of the Vedic mantras and pilgrimages to sacred places in spite of the overriding stress on bhakti as the superior mode of worship. The Jains and Buddhists were its principal targets, not the Brahmins. As a result, the bhakti movement of the south in the long run strengthened the hierarchical system rather than weakening it. So, the bhakti movement critiqued the oppression of the social order but never the social order itself.
    Statement 3 is correct: The importance of the bhakti movement lies in its providing a powerful ideology and the integration of various social groups into its fold. It included saint-poets who came from all jati including those considered “untouchable” like the Pulaiyar and Panars.

  2. Question 2 of 5
    2. Question

    2. Consider the following pairs:
    Harappan sites – Evidences of
    1. Surkotada – Millets
    2. Dholavira – Catfish bones
    3. Nageshwar – Bangles
    4. Lothal – Ivory scale
    How many pairs given above are correctly matched?

    Correct

    Answer: C
    Explanation:
    The correct matches are as follows:
    ● Surkotada- Millet
    ● Harappa- Catfish bones (incorrectly matched). Dholavira does not give evidence of catfish bones.
    ● Nageshwar- Bangles
    ● Lothal- Ivory scales
    Additional Information:
    Given the area covered by the civilization, naturally there were regional variations in the plants grown by farmers. Wheat has been found at Mohenjodaro and Harappa; barley at Mohenjodaro, Harappa, and Kalibangan; and sesamum at Harappa. Harappa has also given evidence of watermelon seeds, peas, and dates. Rice occurs at Harappa, Kalibangan, Lothal, and Rangpur. Millets have been identified at Harappa, Surkotada, and Shortughai.
    Chanhudaro and Balakot were important centres of shell work. Bangles were often made from conch shells. An intensive surface survey and excavations at Nageshwar (in Jamnagar district) have shown that this site was exclusively devoted to shell-working and specialized in making bangles. Evidence of shell working also comes from Kuntasi, Dholavira, Rangpur, Lothal, Nagwada, and Bagasra.
    The Harappan crafts display an impressive level of standardization. Standardization extended to units of weights and measures. Cubical weights made of chert, chalcedony, black stone, etc. have been found at all excavated sites, and their accuracy all over the Harappan culture zone is remarkable. The system is binary in the smaller weights (1:2:8:16:32:64) and decimal in the higher weights (with a ratio of 160, 200, 320, and 640). The largest weight found at Mohenjodaro weighs 10.865 g. A shell scale was found at Mohenjodaro and an ivory scale at Lothal.

    Incorrect

    Answer: C
    Explanation:
    The correct matches are as follows:
    ● Surkotada- Millet
    ● Harappa- Catfish bones (incorrectly matched). Dholavira does not give evidence of catfish bones.
    ● Nageshwar- Bangles
    ● Lothal- Ivory scales
    Additional Information:
    Given the area covered by the civilization, naturally there were regional variations in the plants grown by farmers. Wheat has been found at Mohenjodaro and Harappa; barley at Mohenjodaro, Harappa, and Kalibangan; and sesamum at Harappa. Harappa has also given evidence of watermelon seeds, peas, and dates. Rice occurs at Harappa, Kalibangan, Lothal, and Rangpur. Millets have been identified at Harappa, Surkotada, and Shortughai.
    Chanhudaro and Balakot were important centres of shell work. Bangles were often made from conch shells. An intensive surface survey and excavations at Nageshwar (in Jamnagar district) have shown that this site was exclusively devoted to shell-working and specialized in making bangles. Evidence of shell working also comes from Kuntasi, Dholavira, Rangpur, Lothal, Nagwada, and Bagasra.
    The Harappan crafts display an impressive level of standardization. Standardization extended to units of weights and measures. Cubical weights made of chert, chalcedony, black stone, etc. have been found at all excavated sites, and their accuracy all over the Harappan culture zone is remarkable. The system is binary in the smaller weights (1:2:8:16:32:64) and decimal in the higher weights (with a ratio of 160, 200, 320, and 640). The largest weight found at Mohenjodaro weighs 10.865 g. A shell scale was found at Mohenjodaro and an ivory scale at Lothal.

  3. Question 3 of 5
    3. Question

    3. With reference to the history of colonial India, the term ‘amlah’ refers to:

    Correct

    Answer: A
    Explanation:
    The question has been framed in the context of land revenue settlements introduced by the British administration, i.e. Permanent settlement.
    ● Under this settlement, the revenue collection task was given to Zamindars. ‘Amlah’ were the officials of the Zamindars who would go to the villages and collect the revenue on behalf of Zamindars.
    ● Village headmen were known as Jotedars and Mandals.
    ● Sharecroppers were known as Adhiyars and Bargadars.
    ● Territorial units were known as ‘Taluq’.

    Incorrect

    Answer: A
    Explanation:
    The question has been framed in the context of land revenue settlements introduced by the British administration, i.e. Permanent settlement.
    ● Under this settlement, the revenue collection task was given to Zamindars. ‘Amlah’ were the officials of the Zamindars who would go to the villages and collect the revenue on behalf of Zamindars.
    ● Village headmen were known as Jotedars and Mandals.
    ● Sharecroppers were known as Adhiyars and Bargadars.
    ● Territorial units were known as ‘Taluq’.

  4. Question 4 of 5
    4. Question

    4. Which of the following terms would refer to an urban settlement having a small fixed market?

    Correct

    Answer: D
    Explanation:
    The advent of colonial rule and the gradual erosion of Mughal power led to the decline of old towns and new towns and urban settlement came into existence.
    All the above mentioned terms refer to various types of settlements.
    ● Qashbah refers to a small town in the countryside, often the seat of local notables.
    ● Pet is a Tamil word, meaning ‘settlement’.
    ● Puram is a term used for a village.
    ● Ganj refers to an urban settlement having a small fixed market.

    Incorrect

    Answer: D
    Explanation:
    The advent of colonial rule and the gradual erosion of Mughal power led to the decline of old towns and new towns and urban settlement came into existence.
    All the above mentioned terms refer to various types of settlements.
    ● Qashbah refers to a small town in the countryside, often the seat of local notables.
    ● Pet is a Tamil word, meaning ‘settlement’.
    ● Puram is a term used for a village.
    ● Ganj refers to an urban settlement having a small fixed market.

  5. Question 5 of 5
    5. Question

    5. Which of the following uprisings/movements represented the last remnants of the Revolt of 1857?

    Correct

    Answer: B
    Explanation:
    The Revolt of 1857 sent a clear message to overthrow British rule. After the Revolt, the period between 1857 to 1870, witnessed two anti-British movements the declared aim of which was the armed overthrow of the British government.
    These were the movement of the Wahabis, a Muslim sect, whose adherents had participated in the Revolt of 1857 and that of the Marathas who, undaunted by the defeat of 1857, carried on subsequently, conspirational activity for the same aim of removing British rule from the country.
    Also, all the above mentioned movements/rebellions except the Wahabi movement occurred prior to the Revolt of 1857.
    ● The Rangpur rebellion of 1783 in the northern districts of Bengal against the revenue contractors and company officials.
    ● The revolt of Pazhassi Raja (Malabar) in 1796–1805, followed by the insurrection of Velu Thampi, the prime minister of the Travancore state.
    ● The Sanyasi rebellion, which rocked northern Bengal and adjacent areas of Bihar between 1763 and 1800.

    Incorrect

    Answer: B
    Explanation:
    The Revolt of 1857 sent a clear message to overthrow British rule. After the Revolt, the period between 1857 to 1870, witnessed two anti-British movements the declared aim of which was the armed overthrow of the British government.
    These were the movement of the Wahabis, a Muslim sect, whose adherents had participated in the Revolt of 1857 and that of the Marathas who, undaunted by the defeat of 1857, carried on subsequently, conspirational activity for the same aim of removing British rule from the country.
    Also, all the above mentioned movements/rebellions except the Wahabi movement occurred prior to the Revolt of 1857.
    ● The Rangpur rebellion of 1783 in the northern districts of Bengal against the revenue contractors and company officials.
    ● The revolt of Pazhassi Raja (Malabar) in 1796–1805, followed by the insurrection of Velu Thampi, the prime minister of the Travancore state.
    ● The Sanyasi rebellion, which rocked northern Bengal and adjacent areas of Bihar between 1763 and 1800.

window.wpAdvQuizInitList = window.wpAdvQuizInitList || []; window.wpAdvQuizInitList.push({ id: '#wpAdvQuiz_584', init: { quizId: 584, mode: 0, globalPoints: 10, timelimit: 0, resultsGrade: [0], bo: 0, qpp: 0, catPoints: [10], formPos: 0, lbn: "Finish quiz", json: {"2714":{"type":"single","id":2714,"catId":0,"points":2,"correct":[0,1,0,0]},"2715":{"type":"single","id":2715,"catId":0,"points":2,"correct":[0,0,1,0]},"2716":{"type":"single","id":2716,"catId":0,"points":2,"correct":[1,0,0,0]},"2717":{"type":"single","id":2717,"catId":0,"points":2,"correct":[0,0,0,1]},"2718":{"type":"single","id":2718,"catId":0,"points":2,"correct":[0,1,0,0]}} } });




Day-544 | Daily MCQs | UPSC Prelims | GEOGRAPHY

Day-544

Time limit: 0

Quiz-summary

0 of 5 questions completed

Questions:

  1. 1
  2. 2
  3. 3
  4. 4
  5. 5

Information

DAILY MCQ

You have already completed the quiz before. Hence you can not start it again.

Quiz is loading...

You must sign in or sign up to start the quiz.

You have to finish following quiz, to start this quiz:

Results

0 of 5 questions answered correctly

Your time:

Time has elapsed

You have reached 0 of 0 points, (0)

Categories

  1. Not categorized 0%
  1. 1
  2. 2
  3. 3
  4. 4
  5. 5
  1. Answered
  2. Review
  1. Question 1 of 5
    1. Question

    1. Consider the following statements:
    Statement I: A tropical cyclone is more likely to be developed at the equator in comparison to the tropical regions.
    Statement II: Coriolis force is negligible at the equator.
    Which one of the following is correct in respect of the above statements?

    Correct

    Answer: D
    Explanation:
    Statement 1 is incorrect:
    ● Tropical cyclones cannot originate at the equator because of the lack of a rotating trigger known as the Coriolis force.
    Statement 2 is correct: The rotation of the earth about its axis affects the direction of the wind. This force is called the Coriolis force after the French physicist who described it in 1844. It deflects the wind to the right direction in the northern hemisphere and to the left in the southern hemisphere. The deflection is more when the wind velocity is high. The Coriolis force is directly proportional to the angle of latitude. It is maximum at the poles and is absent at the equator.

    Additional Information
    About Tropical Cyclones-
    ● A Tropical cyclone is a rapid inward air circulation around a low-pressure area.
    ● Tropical cyclones originate and intensify over warm tropical oceans. The conditions favourable for the formation and intensification of tropical storms are: (i) Large sea surface with temperature higher than 27° C; (ii) Presence of the Coriolis force; (iii) Small variations in the vertical wind speed; (iv) A pre-existing weak low-pressure area or low-level-cyclonic circulation; (v) Upper divergence above the sea level system.
    ● The energy that intensifies the storm, comes from the condensation process in the towering cumulonimbus clouds, surrounding the centre of the storm.
    ● The place where a tropical cyclone crosses the coast is called the landfall of the cyclone. The cyclones, which cross 20-degree N latitude generally, recurve and they are more destructive.
    ● A mature tropical cyclone is characterised by the strong spirally circulating wind around the centre, called the eye. The diameter of the circulating system can vary between 150 and 250 km.

    Some examples:
    ● Cyclone Tej (named by India) made its origin in Arabian Sea while Cyclone Hamoon (named by Iran) made its origin in Bay of Bengal.

    Incorrect

    Answer: D
    Explanation:
    Statement 1 is incorrect:
    ● Tropical cyclones cannot originate at the equator because of the lack of a rotating trigger known as the Coriolis force.
    Statement 2 is correct: The rotation of the earth about its axis affects the direction of the wind. This force is called the Coriolis force after the French physicist who described it in 1844. It deflects the wind to the right direction in the northern hemisphere and to the left in the southern hemisphere. The deflection is more when the wind velocity is high. The Coriolis force is directly proportional to the angle of latitude. It is maximum at the poles and is absent at the equator.

    Additional Information
    About Tropical Cyclones-
    ● A Tropical cyclone is a rapid inward air circulation around a low-pressure area.
    ● Tropical cyclones originate and intensify over warm tropical oceans. The conditions favourable for the formation and intensification of tropical storms are: (i) Large sea surface with temperature higher than 27° C; (ii) Presence of the Coriolis force; (iii) Small variations in the vertical wind speed; (iv) A pre-existing weak low-pressure area or low-level-cyclonic circulation; (v) Upper divergence above the sea level system.
    ● The energy that intensifies the storm, comes from the condensation process in the towering cumulonimbus clouds, surrounding the centre of the storm.
    ● The place where a tropical cyclone crosses the coast is called the landfall of the cyclone. The cyclones, which cross 20-degree N latitude generally, recurve and they are more destructive.
    ● A mature tropical cyclone is characterised by the strong spirally circulating wind around the centre, called the eye. The diameter of the circulating system can vary between 150 and 250 km.

    Some examples:
    ● Cyclone Tej (named by India) made its origin in Arabian Sea while Cyclone Hamoon (named by Iran) made its origin in Bay of Bengal.

  2. Question 2 of 5
    2. Question

    2. With reference to limestone caves, consider the following pairs:
    Caves – States
    1. Borra Caves – Meghalaya
    2. Liat Prah Caves – Andhra Pradesh
    3. Khangkhui Cave – Manipur
    How many of the above pairs are correctly matched?

    Correct

    Answer: A
    Explanation:
    Pair 1 is matched incorrectly:
    ● The Borra Caves are located on the East Coast of India, in the Ananthagiri hills of the Araku Valley of the Alluri Sitharama Raju district in Andhra Pradesh.
    ● It is one of India’s longest and the deepest caves extending up to 80 m.
    ● These caves are considered to be among a rare geological formation and these were formed by the water activity by the Gosthani River.
    ● Borra Caves were discovered by William King George in 1807.
    ● There is a naturally formed Shivling inside the caves, which is worshipped by the tribals who live in the nearby forest.

    Pair 2 is matched incorrectly:
    ● Krem Liat Prah is the longest natural cave in South Asia.
    ● Prah (Krem is the Khasi word for “cave”) is one of approximately 150 known caves in the Shnongrim Ridge of the East Jaintia Hills district in the state of Meghalaya, northeast India.

    Pair 3 is matched correctly:
    Khangkhui Cave:
    ● Khangkhui is a Tangkhul Naga village in the North-eastern part of Ukhrul district of Manipur, India.
    ● It is well known for its ancient limestone cave locally called Khangkhui Mangsor.
    ● This pre-historic cave is one of the tourist attractions of Ukhrul district.
    ● The excavation carried out at the Khangkhui Cave yielded evidence of habitation of Stone Age communities.

    Incorrect

    Answer: A
    Explanation:
    Pair 1 is matched incorrectly:
    ● The Borra Caves are located on the East Coast of India, in the Ananthagiri hills of the Araku Valley of the Alluri Sitharama Raju district in Andhra Pradesh.
    ● It is one of India’s longest and the deepest caves extending up to 80 m.
    ● These caves are considered to be among a rare geological formation and these were formed by the water activity by the Gosthani River.
    ● Borra Caves were discovered by William King George in 1807.
    ● There is a naturally formed Shivling inside the caves, which is worshipped by the tribals who live in the nearby forest.

    Pair 2 is matched incorrectly:
    ● Krem Liat Prah is the longest natural cave in South Asia.
    ● Prah (Krem is the Khasi word for “cave”) is one of approximately 150 known caves in the Shnongrim Ridge of the East Jaintia Hills district in the state of Meghalaya, northeast India.

    Pair 3 is matched correctly:
    Khangkhui Cave:
    ● Khangkhui is a Tangkhul Naga village in the North-eastern part of Ukhrul district of Manipur, India.
    ● It is well known for its ancient limestone cave locally called Khangkhui Mangsor.
    ● This pre-historic cave is one of the tourist attractions of Ukhrul district.
    ● The excavation carried out at the Khangkhui Cave yielded evidence of habitation of Stone Age communities.

  3. Question 3 of 5
    3. Question

    3. Consider the following statements:
    Statement-I: Cherrapunji in Meghalaya plateau displays a bare rocky surface devoid of any permanent vegetation cover.
    Statement-II: This area receives maximum rainfall from the Northeast monsoon.
    Which one of the following is correct in respect of the above statements?

    Correct

    Answer: C
    Explanation:
    Statement I is correct: Cherrapunji displays a bare rocky surface devoid of any permanent vegetation cover.
    Statement II is incorrect: The area of Meghalaya plateau receives maximum rainfall from the south west monsoon. As a result, the Meghalaya plateau has a highly eroded surface.
    About Meghalaya plateau:
    ● It is a part of the Northeastern plateau which is an extension of the main peninsular plateau.
    ● It is believed that due to the force exerted by the northeastward movement of the Indian plate at the time of the Himalayan origin, a huge fault was created between the Rajmahal hills and the Meghalaya plateau.
    ● Later, this depression got filled up by the deposition activity of the numerous rivers.
    ● Today, the Meghalaya and Karbi Anglong plateau stand detached from the main Peninsular Block.
    ● The Meghalaya plateau is further sub-divided into three:
    ⮚ The Garo Hills
    ⮚ The Khasi Hills
    ⮚ The Jaintia Hills
    ● They are named after the tribal groups inhabiting this region.
    ● An extension of this is also seen in the Karbi Anglong hills of Assam.
    ● Similar to the Chotanagpur plateau, the Meghalaya plateau is also rich in mineral resources like coal, iron ore, sillimanite, limestone and uranium.
    ● This area receives maximum rainfall from the south west monsoon.

    Incorrect

    Answer: C
    Explanation:
    Statement I is correct: Cherrapunji displays a bare rocky surface devoid of any permanent vegetation cover.
    Statement II is incorrect: The area of Meghalaya plateau receives maximum rainfall from the south west monsoon. As a result, the Meghalaya plateau has a highly eroded surface.
    About Meghalaya plateau:
    ● It is a part of the Northeastern plateau which is an extension of the main peninsular plateau.
    ● It is believed that due to the force exerted by the northeastward movement of the Indian plate at the time of the Himalayan origin, a huge fault was created between the Rajmahal hills and the Meghalaya plateau.
    ● Later, this depression got filled up by the deposition activity of the numerous rivers.
    ● Today, the Meghalaya and Karbi Anglong plateau stand detached from the main Peninsular Block.
    ● The Meghalaya plateau is further sub-divided into three:
    ⮚ The Garo Hills
    ⮚ The Khasi Hills
    ⮚ The Jaintia Hills
    ● They are named after the tribal groups inhabiting this region.
    ● An extension of this is also seen in the Karbi Anglong hills of Assam.
    ● Similar to the Chotanagpur plateau, the Meghalaya plateau is also rich in mineral resources like coal, iron ore, sillimanite, limestone and uranium.
    ● This area receives maximum rainfall from the south west monsoon.

  4. Question 4 of 5
    4. Question

    4. Consider the following statements:
    1. The Labrador Current is a cold current in the North Atlantic Ocean that flows from the Arctic Ocean.
    2. The mixing of the Humboldt Current and the Labrador Current produces heavy fog and has created one of the richest fishing grounds in the world.
    Which of the statements given above is/are correct?

    Correct

    Answer: A
    Explanation:
    Statement 1 is correct:
    ● The Labrador Current is a cold current in the North Atlantic Ocean which flows from the Arctic Ocean south along the coast of Labrador and passes around Newfoundland, continuing south along the east coast of Canada near Nova Scotia.

    Statement 2 is incorrect:
    ● Near Nova Scotia, the Labrador Current meets the warm northward moving Gulf Stream.
    ● The mixing of these two currents produces heavy fog and has also created one of the richest fishing grounds in the world. Example- in Grand banks near Newfoundland, Canada.
    Additional Information-
    Warm and Cold currents of the world:

    Most famous fishing banks in the world due to confluence of warm and cold currents:

    Incorrect

    Answer: A
    Explanation:
    Statement 1 is correct:
    ● The Labrador Current is a cold current in the North Atlantic Ocean which flows from the Arctic Ocean south along the coast of Labrador and passes around Newfoundland, continuing south along the east coast of Canada near Nova Scotia.

    Statement 2 is incorrect:
    ● Near Nova Scotia, the Labrador Current meets the warm northward moving Gulf Stream.
    ● The mixing of these two currents produces heavy fog and has also created one of the richest fishing grounds in the world. Example- in Grand banks near Newfoundland, Canada.
    Additional Information-
    Warm and Cold currents of the world:

    Most famous fishing banks in the world due to confluence of warm and cold currents:

  5. Question 5 of 5
    5. Question

    5. Consider the following statements:
    1. Ken River passes through Panna Tiger Reserve.
    2. Rajaji Tiger Reserve is bisected by Ramganga River.
    3. Mahanadi flows through Satkosia Tiger Reserve.
    4. Sharda River runs through Valmiki Tiger Reserve.
    How many of the statements given above are correct?

    Correct

    Answer: B
    Explanation:
    Statement 1 is correct:
    Ken River passes through Panna Tiger Reserve. As shown in figure given below:
    About Panna Tiger Reserve:
    ● Situated in the Vindhyan mountain range in the northern part of Madhya Pradesh,
    ● Panna Tiger Reserve is spread over the Panna and Chhatarpur districts.
    ● The terrain here consists of extensive plateaus and gorges.
    ● This reserve contains the last remaining tiger habitat of North Madhya Pradesh.
    ● Flowing from the south to the north through the reserve is the River Ken.
    ● These forests along with Ken Gharial Sanctuary form a significant part of the catchment area of this river.
    ● This river is one of the sixteen perennial rivers of Madhya Pradesh.
    ● It is the lifeline of this reserve and is the least polluted of Yamuna’s tributaries.
    ● The path of the meandering Ken offers some spectacular scenery.
    ● The reserve is also dotted with two thousand year-old rock paintings.

    Statement 2 is incorrect: Rajaji Tiger Reserve is bisected by River Ganga, not by River Ramganga. River Ramganga passes through Jim Corbett Tiger Reserve.
    About Rajaji tiger reserve:
    ● Rajaji Tiger Reserve is situated along the hills and foothills of Shiwalik ranges and represents the Shiwalik eco-system.
    ● The area is covered with diverse forest types ranging from semi-evergreen to deciduous and from mixed broad-leaved to Terai grassland and has been classified as Indus-Ganges Monsoon Forest type.

    Statement 3 is correct: Mahanadi River flows through Satkosia Tiger reserve of Odisha.
    About Satkosia Tiger Reserve:
    ● It is a tiger reserve located on the border of Angul and Nayagarh district of Odisha, covering an area of 988.30 km².
    ● It is located where the Mahanadi River passes through a 22 km long gorge in the Eastern Ghats Mountains.
    ● The tiger reserve is located in the Eastern Highlands moist deciduous forests eco-region.

    Statement 4 is incorrect: River Sharda is a major river flowing through Pilibhit Tiger Reserve, while Narayani River is a major river flowing through Valmiki Tiger Reserve.
    About Pilibhit tiger reserve:
    ● Pilibhit Tiger Reserve is located in Pilibhit district, Lakhimpur Kheri District and Bahraich District of Uttar Pradesh.
    ● The Pilibhit Tiger Reserve is one of the finest examples of the exceedingly diverse and productive Terai ecosystems.
    ● Pilibhit Tiger Reserve was declared in September 2008 based on its special type of ecosystem with vast open spaces and sufficient feed for the elegant predators. It is India’s 45th Tiger Reserve Project.
    ● The northern edge of the reserve lies along the Indo-Nepal border while the southern boundary is marked by the river Sharada and Khakra.

    Incorrect

    Answer: B
    Explanation:
    Statement 1 is correct:
    Ken River passes through Panna Tiger Reserve. As shown in figure given below:
    About Panna Tiger Reserve:
    ● Situated in the Vindhyan mountain range in the northern part of Madhya Pradesh,
    ● Panna Tiger Reserve is spread over the Panna and Chhatarpur districts.
    ● The terrain here consists of extensive plateaus and gorges.
    ● This reserve contains the last remaining tiger habitat of North Madhya Pradesh.
    ● Flowing from the south to the north through the reserve is the River Ken.
    ● These forests along with Ken Gharial Sanctuary form a significant part of the catchment area of this river.
    ● This river is one of the sixteen perennial rivers of Madhya Pradesh.
    ● It is the lifeline of this reserve and is the least polluted of Yamuna’s tributaries.
    ● The path of the meandering Ken offers some spectacular scenery.
    ● The reserve is also dotted with two thousand year-old rock paintings.

    Statement 2 is incorrect: Rajaji Tiger Reserve is bisected by River Ganga, not by River Ramganga. River Ramganga passes through Jim Corbett Tiger Reserve.
    About Rajaji tiger reserve:
    ● Rajaji Tiger Reserve is situated along the hills and foothills of Shiwalik ranges and represents the Shiwalik eco-system.
    ● The area is covered with diverse forest types ranging from semi-evergreen to deciduous and from mixed broad-leaved to Terai grassland and has been classified as Indus-Ganges Monsoon Forest type.

    Statement 3 is correct: Mahanadi River flows through Satkosia Tiger reserve of Odisha.
    About Satkosia Tiger Reserve:
    ● It is a tiger reserve located on the border of Angul and Nayagarh district of Odisha, covering an area of 988.30 km².
    ● It is located where the Mahanadi River passes through a 22 km long gorge in the Eastern Ghats Mountains.
    ● The tiger reserve is located in the Eastern Highlands moist deciduous forests eco-region.

    Statement 4 is incorrect: River Sharda is a major river flowing through Pilibhit Tiger Reserve, while Narayani River is a major river flowing through Valmiki Tiger Reserve.
    About Pilibhit tiger reserve:
    ● Pilibhit Tiger Reserve is located in Pilibhit district, Lakhimpur Kheri District and Bahraich District of Uttar Pradesh.
    ● The Pilibhit Tiger Reserve is one of the finest examples of the exceedingly diverse and productive Terai ecosystems.
    ● Pilibhit Tiger Reserve was declared in September 2008 based on its special type of ecosystem with vast open spaces and sufficient feed for the elegant predators. It is India’s 45th Tiger Reserve Project.
    ● The northern edge of the reserve lies along the Indo-Nepal border while the southern boundary is marked by the river Sharada and Khakra.

window.wpAdvQuizInitList = window.wpAdvQuizInitList || []; window.wpAdvQuizInitList.push({ id: '#wpAdvQuiz_583', init: { quizId: 583, mode: 0, globalPoints: 10, timelimit: 0, resultsGrade: [0], bo: 0, qpp: 0, catPoints: [10], formPos: 0, lbn: "Finish quiz", json: {"2709":{"type":"single","id":2709,"catId":0,"points":2,"correct":[0,0,0,1]},"2710":{"type":"single","id":2710,"catId":0,"points":2,"correct":[1,0,0,0]},"2711":{"type":"single","id":2711,"catId":0,"points":2,"correct":[0,0,1,0]},"2712":{"type":"single","id":2712,"catId":0,"points":2,"correct":[1,0,0,0]},"2713":{"type":"single","id":2713,"catId":0,"points":2,"correct":[0,1,0,0]}} } });




Day-543 | Daily MCQs | UPSC Prelims | CURRENT DEVELOPMENTS

Day-543

Time limit: 0

Quiz-summary

0 of 5 questions completed

Questions:

  1. 1
  2. 2
  3. 3
  4. 4
  5. 5

Information

DAILY MCQ

You have already completed the quiz before. Hence you can not start it again.

Quiz is loading...

You must sign in or sign up to start the quiz.

You have to finish following quiz, to start this quiz:

Results

0 of 5 questions answered correctly

Your time:

Time has elapsed

You have reached 0 of 0 points, (0)

Categories

  1. Not categorized 0%
  1. 1
  2. 2
  3. 3
  4. 4
  5. 5
  1. Answered
  2. Review
  1. Question 1 of 5
    1. Question

    1. With reference to the National Mission on Natural Farming, consider the following statements:
    1. Under the mission, natural farming is promoted through adoption of villages by cluster approach and Participatory Guarantee System (PGS) certification.
    2. It is being implemented by the Ministry of Chemicals and Fertilizers.
    3. Under NMNF, farmers will receive a financial assistance of ₹15,000 per hectare per year for three years for the creation of on-farm input production infrastructure.
    How many of the statements given above are correct?

    Correct

    Answer: A
    Explanation:
    Context: Government of India has launched the National Mission on Natural Farming (NMNF) as a separate and independent scheme to promote chemical-free and climate-smart agriculture.
    • Statement 1 is incorrect: NMNF will cover a 7.5 lakh hectare area by developing 15,000 clusters. The farmers willing to implement natural farming on their field will be registered as cluster members, each cluster shall comprise 50 farmers or more with 50-hectare land. Also, each cluster can fall into one village or spread across 2-3 nearby villages under the same gram panchayat. There is no concept of Participatory Guarantee System (PGS) certification in the mission for now.
    • Statement 2 is incorrect: It is being implemented by the Ministry of Agriculture & Farmers Welfare.
    • Statement 3 is correct: Under NMNF, farmers will receive a financial assistance of ₹15,000 per hectare per year for three years for the creation of on-farm input production infrastructure.
    Additional information:
    ● The National Mission on Natural Farming (NMNF) has been formulated by upscaling the Bhartiya Prakritik Krishi Paddhati (BPKP) to promote natural farming across the country.
    ● NMNF will cover a 7.5 lakh hectare area by developing 15,000 clusters. The farmers willing to implement natural farming on their field will be registered as cluster members, each cluster shall comprise 50 farmers or more with 50-hectare land.
    ● Also, each cluster can fall into one village or spread across 2-3 nearby villages under the same gram panchayat.
    Financial Assistance:
    ● However, the incentives would be provided to farmers only when they commit to natural farming and have actually taken it up.
    ● If a farmer defaults or does not continue with natural farming, subsequent instalments shall not be disbursed.

    Incorrect

    Answer: A
    Explanation:
    Context: Government of India has launched the National Mission on Natural Farming (NMNF) as a separate and independent scheme to promote chemical-free and climate-smart agriculture.
    • Statement 1 is incorrect: NMNF will cover a 7.5 lakh hectare area by developing 15,000 clusters. The farmers willing to implement natural farming on their field will be registered as cluster members, each cluster shall comprise 50 farmers or more with 50-hectare land. Also, each cluster can fall into one village or spread across 2-3 nearby villages under the same gram panchayat. There is no concept of Participatory Guarantee System (PGS) certification in the mission for now.
    • Statement 2 is incorrect: It is being implemented by the Ministry of Agriculture & Farmers Welfare.
    • Statement 3 is correct: Under NMNF, farmers will receive a financial assistance of ₹15,000 per hectare per year for three years for the creation of on-farm input production infrastructure.
    Additional information:
    ● The National Mission on Natural Farming (NMNF) has been formulated by upscaling the Bhartiya Prakritik Krishi Paddhati (BPKP) to promote natural farming across the country.
    ● NMNF will cover a 7.5 lakh hectare area by developing 15,000 clusters. The farmers willing to implement natural farming on their field will be registered as cluster members, each cluster shall comprise 50 farmers or more with 50-hectare land.
    ● Also, each cluster can fall into one village or spread across 2-3 nearby villages under the same gram panchayat.
    Financial Assistance:
    ● However, the incentives would be provided to farmers only when they commit to natural farming and have actually taken it up.
    ● If a farmer defaults or does not continue with natural farming, subsequent instalments shall not be disbursed.

  2. Question 2 of 5
    2. Question

    2. With reference to the Sahitya Akademi Awards, consider the following statements:
    1. It can be conferred only upon an Indian Citizen.
    2. It is the highest literary honour given by the Government of India.
    3. It cannot be awarded posthumously.
    How many of the statements given above are correct?

    Correct

    Answer: A
    Explanation:
    Context: Twenty-four writers across as many Indian languages were honoured with the Sahitya Akademi Award 2022, at the beginning of the 39th ‘Festival of Letters’ in Delhi in March, 2023.
    • Statement 1 is correct: It can be conferred upon only on an Indian Citizen.
    • Statement 2 is incorrect: It is the second-highest literary honour given by the Government of India, after the Jnanpith award.
    • Statement 3 is incorrect: A posthumous publication is eligible for Award only if the author has died within the five year period stipulated for the award or later.
    Additional information:
    ● Sahitya Akademi award established in 1954, is a literary honour that is conferred annually by Sahitya Akademi, India’s National Academy of letters.
    ● Akademi gives 24 awards annually to literary works in the languages it has recognized and an equal number of awards to literary translations from and into the languages of India.
    ● Besides the 22 languages enumerated in the Constitution of India, the Sahitya Akademi has recognised English and Rajasthani as languages in which its programme may be implemented.
    Criteria for Choosing Awardee:
    ● The author must be of Indian Nationality.
    ● Book/work eligible for the award must be an outstanding contribution to the language and literature to which it belongs.
    ● When equal merit for books of two or more are found, certain criteria like total literary contribution and standing of authors shall be taken into consideration for declaring the award.

    Incorrect

    Answer: A
    Explanation:
    Context: Twenty-four writers across as many Indian languages were honoured with the Sahitya Akademi Award 2022, at the beginning of the 39th ‘Festival of Letters’ in Delhi in March, 2023.
    • Statement 1 is correct: It can be conferred upon only on an Indian Citizen.
    • Statement 2 is incorrect: It is the second-highest literary honour given by the Government of India, after the Jnanpith award.
    • Statement 3 is incorrect: A posthumous publication is eligible for Award only if the author has died within the five year period stipulated for the award or later.
    Additional information:
    ● Sahitya Akademi award established in 1954, is a literary honour that is conferred annually by Sahitya Akademi, India’s National Academy of letters.
    ● Akademi gives 24 awards annually to literary works in the languages it has recognized and an equal number of awards to literary translations from and into the languages of India.
    ● Besides the 22 languages enumerated in the Constitution of India, the Sahitya Akademi has recognised English and Rajasthani as languages in which its programme may be implemented.
    Criteria for Choosing Awardee:
    ● The author must be of Indian Nationality.
    ● Book/work eligible for the award must be an outstanding contribution to the language and literature to which it belongs.
    ● When equal merit for books of two or more are found, certain criteria like total literary contribution and standing of authors shall be taken into consideration for declaring the award.

  3. Question 3 of 5
    3. Question

    3. Which of the following best reflects the meaning of the term ‘Visa Shopping’?

    Correct

    Answer: A
    Explanation:
    Context: The practice of ‘visa shopping’ has been gaining popularity in several Indian states, including Punjab, where travel companies often facilitate this setup for travellers.
    ● Visa shopping refers to obtaining visas for countries that individuals may or may not visit during the granted visa period.
    ● This is because such visas can provide a gateway for people to enter their preferred countries, particularly in Europe, even if they have obtained a visa for another European nation.
    ● Additionally, people who have no immediate plans to visit a specific country are also purchasing visas for such nations.
    ● This is to increase the chances of obtaining visas for countries of their choice in the future, given how the application process often takes a long time or has chances of rejection.
    ● Also, visas usually remain valid for a few years after being issued.

    Incorrect

    Answer: A
    Explanation:
    Context: The practice of ‘visa shopping’ has been gaining popularity in several Indian states, including Punjab, where travel companies often facilitate this setup for travellers.
    ● Visa shopping refers to obtaining visas for countries that individuals may or may not visit during the granted visa period.
    ● This is because such visas can provide a gateway for people to enter their preferred countries, particularly in Europe, even if they have obtained a visa for another European nation.
    ● Additionally, people who have no immediate plans to visit a specific country are also purchasing visas for such nations.
    ● This is to increase the chances of obtaining visas for countries of their choice in the future, given how the application process often takes a long time or has chances of rejection.
    ● Also, visas usually remain valid for a few years after being issued.

  4. Question 4 of 5
    4. Question

    4. With reference to the term ‘Dynamic Injuction’, which of the following statements is correct?

    Correct

    Answer: C
    Explanation:
    Context: The Delhi High Court restrained nine websites from illegally broadcasting the ICC Men’s Cricket World Cup 2023 matches by passing a “dynamic injunction”.
    ● A dynamic injunction is an official order given by a law court, usually to stop someone from doing something.
    ● However, in most cases, such injunctions are granted by the court only after the court identifies the work and determines the plaintiff’s (party who initiates a lawsuit) copyright in that work.
    ● To avoid this cumbersome process and grant protection to copyrighted works in a timely manner, courts sometimes rely on the concept of “dynamic” injunctions.
    ● A dynamic injunction is passed to protect copyrighted works even before they are publicly released, distributed, or created.
    ● It ensures that no irreparable loss is caused to its authors and owner, owing to the imminent possibility of such works being uploaded on rogue websites or their newer versions immediately after their creation or release, given the challenges posed by online piracy.

    Incorrect

    Answer: C
    Explanation:
    Context: The Delhi High Court restrained nine websites from illegally broadcasting the ICC Men’s Cricket World Cup 2023 matches by passing a “dynamic injunction”.
    ● A dynamic injunction is an official order given by a law court, usually to stop someone from doing something.
    ● However, in most cases, such injunctions are granted by the court only after the court identifies the work and determines the plaintiff’s (party who initiates a lawsuit) copyright in that work.
    ● To avoid this cumbersome process and grant protection to copyrighted works in a timely manner, courts sometimes rely on the concept of “dynamic” injunctions.
    ● A dynamic injunction is passed to protect copyrighted works even before they are publicly released, distributed, or created.
    ● It ensures that no irreparable loss is caused to its authors and owner, owing to the imminent possibility of such works being uploaded on rogue websites or their newer versions immediately after their creation or release, given the challenges posed by online piracy.

  5. Question 5 of 5
    5. Question

    5. The term ‘Scarborough Shoal’, often seen in the news is:

    Correct

    Answer: D
    Explanation:
    Context: A recent dispute between China and the Philippines over Scarborough Shoal in the South China Sea turns the spotlight on simmering tensions in the strategic waters with ramifications to all countries in the Indo-Pacific region, including India.
    ● The Scarborough Shoal is a dispute reef formation between the Philippines and the People’s Republic of China in South China Sea.
    ● Scarborough Shoal is also known as Bajo de Masinloc, Panatag Shoal, Huangyan Island, and Democracy Reef, are two rocks in an atoll administered by China in the South China Sea.
    ● It is located between Macclesfield Bank to the west and Luzon, its nearest landmass, to the east.
    ● Scarborough Shoal forms a triangle-shaped chain of reefs and rocks with a perimeter of 46 km (29 mi).
    ● It covers an area of 150 km2 (58 sq mi), including an inner lagoon. The atolls’ highest point, South Rock, is 1.8 m (5 ft 11 in) above sea-level at high tide.

    Additional information:
    ● The heart of the South China Sea dispute revolves around territorial claims to land features (islands and reefs) and their associated territorial waters.
    ● The major island and reef formations in the South China Sea are the Spratly Islands, Paracel Islands, Pratas, the Natuna Islands and Scarborough Shoal.

    Incorrect

    Answer: D
    Explanation:
    Context: A recent dispute between China and the Philippines over Scarborough Shoal in the South China Sea turns the spotlight on simmering tensions in the strategic waters with ramifications to all countries in the Indo-Pacific region, including India.
    ● The Scarborough Shoal is a dispute reef formation between the Philippines and the People’s Republic of China in South China Sea.
    ● Scarborough Shoal is also known as Bajo de Masinloc, Panatag Shoal, Huangyan Island, and Democracy Reef, are two rocks in an atoll administered by China in the South China Sea.
    ● It is located between Macclesfield Bank to the west and Luzon, its nearest landmass, to the east.
    ● Scarborough Shoal forms a triangle-shaped chain of reefs and rocks with a perimeter of 46 km (29 mi).
    ● It covers an area of 150 km2 (58 sq mi), including an inner lagoon. The atolls’ highest point, South Rock, is 1.8 m (5 ft 11 in) above sea-level at high tide.

    Additional information:
    ● The heart of the South China Sea dispute revolves around territorial claims to land features (islands and reefs) and their associated territorial waters.
    ● The major island and reef formations in the South China Sea are the Spratly Islands, Paracel Islands, Pratas, the Natuna Islands and Scarborough Shoal.

window.wpAdvQuizInitList = window.wpAdvQuizInitList || []; window.wpAdvQuizInitList.push({ id: '#wpAdvQuiz_582', init: { quizId: 582, mode: 0, globalPoints: 10, timelimit: 0, resultsGrade: [0], bo: 0, qpp: 0, catPoints: [10], formPos: 0, lbn: "Finish quiz", json: {"2704":{"type":"single","id":2704,"catId":0,"points":2,"correct":[1,0,0,0]},"2705":{"type":"single","id":2705,"catId":0,"points":2,"correct":[1,0,0,0]},"2706":{"type":"single","id":2706,"catId":0,"points":2,"correct":[1,0,0,0]},"2707":{"type":"single","id":2707,"catId":0,"points":2,"correct":[0,0,1,0]},"2708":{"type":"single","id":2708,"catId":0,"points":2,"correct":[0,0,0,1]}} } });




Day-542 | Daily MCQs | UPSC Prelims | ENVIRONMENT AND ECOLOGY

Day-542

Time limit: 0

Quiz-summary

0 of 5 questions completed

Questions:

  1. 1
  2. 2
  3. 3
  4. 4
  5. 5

Information

DAILY MCQ

You have already completed the quiz before. Hence you can not start it again.

Quiz is loading...

You must sign in or sign up to start the quiz.

You have to finish following quiz, to start this quiz:

Results

0 of 5 questions answered correctly

Your time:

Time has elapsed

You have reached 0 of 0 points, (0)

Categories

  1. Not categorized 0%
  1. 1
  2. 2
  3. 3
  4. 4
  5. 5
  1. Answered
  2. Review
  1. Question 1 of 5
    1. Question

    1. Consider the following statements in the context of lead pollution:
    1. Lead is commonly found in paints and cigarette smoke.
    2. Exposure to lead can cause neurodegenerative diseases.
    3. Unlike other toxins, lead can be naturally excreted from the human body.
    How many of the above statements are correct?

    Correct

    Answer: B
    Explanation:
    Statement 1 is correct: Lead is commonly found in paints, ceramic cookware, cigarette smoke, PVC plastics, lead acid batteries, etc.
    Cigarette smoke is known to contain several toxic and heavy metals such as cadmium, lead, copper, nickel, chromium, and zinc.
    The sources of lead are as follows:

    Statement 2 is correct: Lead is a metal which harms both the environment and human health. Exposure to lead can cause numerous and severe neurological problems including nerve damage, cognitive problems, loss of IQ and possibly even conditions such as Alzheimer’s disease.
    During pregnancy, if the woman consumes lead in some form, there is no placental barrier either, so the lead is transferred to the foetus. In a newborn, lead poisoning can result in premature birth, low birth weight and slow growth. In children and adults, it can cause anaemia as well as neurological, skeletal and neuromuscular illnesses.
    There is no known safe level of lead in the human body. However, WHO regulations specify a tolerable limit of 5 μg/dL for lead exposure, beyond which lead poisoning occurs.
    Treatment for lead poisoning involves chelation, in which the patient is given medication that binds with lead molecules and facilitates their excretion through urine.
    Statement 3 is incorrect: Lead, unlike many other toxins, does not get diluted when exposure decreases. Nor does the body excrete it naturally. Instead, it accumulates in the bones and leaches into blood.

    Additional information-
    ● Management of lead-acid batteries came under the Batteries (Management and Handling) Rules, 2001.
    ● In 2022, the Union Ministry of Environment, Forest and Climate Change notified the Battery Waste Management Rules, 2022. The new rules aim at reducing share of battery recycling in the informal sector and stress on extended producer responsibility.

    Incorrect

    Answer: B
    Explanation:
    Statement 1 is correct: Lead is commonly found in paints, ceramic cookware, cigarette smoke, PVC plastics, lead acid batteries, etc.
    Cigarette smoke is known to contain several toxic and heavy metals such as cadmium, lead, copper, nickel, chromium, and zinc.
    The sources of lead are as follows:

    Statement 2 is correct: Lead is a metal which harms both the environment and human health. Exposure to lead can cause numerous and severe neurological problems including nerve damage, cognitive problems, loss of IQ and possibly even conditions such as Alzheimer’s disease.
    During pregnancy, if the woman consumes lead in some form, there is no placental barrier either, so the lead is transferred to the foetus. In a newborn, lead poisoning can result in premature birth, low birth weight and slow growth. In children and adults, it can cause anaemia as well as neurological, skeletal and neuromuscular illnesses.
    There is no known safe level of lead in the human body. However, WHO regulations specify a tolerable limit of 5 μg/dL for lead exposure, beyond which lead poisoning occurs.
    Treatment for lead poisoning involves chelation, in which the patient is given medication that binds with lead molecules and facilitates their excretion through urine.
    Statement 3 is incorrect: Lead, unlike many other toxins, does not get diluted when exposure decreases. Nor does the body excrete it naturally. Instead, it accumulates in the bones and leaches into blood.

    Additional information-
    ● Management of lead-acid batteries came under the Batteries (Management and Handling) Rules, 2001.
    ● In 2022, the Union Ministry of Environment, Forest and Climate Change notified the Battery Waste Management Rules, 2022. The new rules aim at reducing share of battery recycling in the informal sector and stress on extended producer responsibility.

  2. Question 2 of 5
    2. Question

    2. Consider the following provisions of the Wildlife (Protection) Act, 1972:
    1. The onus of immunization of livestock within at least ten km of a Wildlife Sanctuary is on the government authorities.
    2. The diversion of area of two (or more) connected tiger reserves for unsustainable projects requires the approval of the National Tiger Conservation Authority as well as the National Board for Wildlife.
    3. Plants and their derivatives lawfully collected from any National Park are considered the property of respective State governments.
    How many of the above statements are correct?

    Correct

    Answer: A
    Explanation:
    Statement 1 is incorrect: As per the provisions of Section 33(A) of the Wildlife (Protection) Act 1972, the onus of immunization of livestock within at least 5 km of Wildlife Sanctuary is on the State government.
    It states that it is the responsibility of the Chief Wildlife Warden to take measures necessary for immunisation against communicable diseases of the live-stock kept in or within five kilometres of a sanctuary. Also, it prohibits the entry of any livestock in a sanctuary without getting it immunised.
    Statement 2 is correct: The Sec. 38(O) of the Wildlife (Protection) Act 1972 states that the diversion of area of two (or more) connected tiger reserves for unsustainable projects requires the approval of both the National Tiger Conservation Authority (NTCA) and the National Board for Wildlife (NBW).
    Sec. 38(W) of the Act also specifies that no alteration in the boundaries of a tiger reserve shall be made except on a recommendation of the Tiger Conservation Authority and the approval of the National Board for Wildlife.
    Statement 3 is incorrect: As per the provisions of Section 17(H) of the Wildlife Protection Act, every specified plant or part or derivative thereof, which has been lawfully collected or acquired from a sanctuary or National Park declared by the Central Government, shall be the property of the Central Government.

    Incorrect

    Answer: A
    Explanation:
    Statement 1 is incorrect: As per the provisions of Section 33(A) of the Wildlife (Protection) Act 1972, the onus of immunization of livestock within at least 5 km of Wildlife Sanctuary is on the State government.
    It states that it is the responsibility of the Chief Wildlife Warden to take measures necessary for immunisation against communicable diseases of the live-stock kept in or within five kilometres of a sanctuary. Also, it prohibits the entry of any livestock in a sanctuary without getting it immunised.
    Statement 2 is correct: The Sec. 38(O) of the Wildlife (Protection) Act 1972 states that the diversion of area of two (or more) connected tiger reserves for unsustainable projects requires the approval of both the National Tiger Conservation Authority (NTCA) and the National Board for Wildlife (NBW).
    Sec. 38(W) of the Act also specifies that no alteration in the boundaries of a tiger reserve shall be made except on a recommendation of the Tiger Conservation Authority and the approval of the National Board for Wildlife.
    Statement 3 is incorrect: As per the provisions of Section 17(H) of the Wildlife Protection Act, every specified plant or part or derivative thereof, which has been lawfully collected or acquired from a sanctuary or National Park declared by the Central Government, shall be the property of the Central Government.

  3. Question 3 of 5
    3. Question

    3. Consider the following statements:
    Statement I: Black rhinos have hooked upper lips, whereas white rhinos have squared lips.
    Statement II: The black rhino is a browser and feeds on leaves, shoots, and branches.
    Which one of the following is correct in respect of the above statements?

    Correct

    Answer: A
    Explanation:
    Black Rhino and White Rhino are found in the African continent.
    Statement 1 is correct: Among black and white rhinos, black rhinos are the smaller of the two African rhino species. Black and white rhinos can be distinguished by the shape of their lips. Black rhinos have hooked upper lips, whereas white rhinos are characterized by a square lip.
    Statement 2 is correct and explains statement 1: Black rhinos are browsers, rather than grazers, meaning they are herbivores who do not feed on low-growing vegetation, and their pointed lip helps them feed on leaves from bushes and trees.
    A white rhinoceros has a very broad, flat, wide lip, which gives this creature the right idea as a grazer and requires a mouth designed for it. The broad muscular lips are ideal for tearing and tearing the grass.
    HABITATS: Semi-Desert Savannah, Woodlands, Forests, Wetlands
    DISTRIBUTION: Black rhinos or Dicerosbicornis are spread across 12 African countries including Kenya, Namibia, Botswana, South Africa, Tanzania, Zimbabwe, and Zambia.

    Additional Information:
    DIFFERENCES BETWEEN WHITE RHINO AND BLACK RHINO:

    PARAMETER – BLACK RHINO – WHITE RHINO
    1. Size of body – Smaller and more compact physical structure.It has shorter face compared to white rhino. However, it prefers keeping its head lifted because it spends most of its time eating trees. – Taller, bigger, and more complex physical structure.It has a long face, small eyes and a neck weakened because it does not need to be raised so large to feed.
    2. Structure of lips – Black rhinos feed on leaves and branches. Accordingly, their lips shape has evolved into a pointed lip (with the shape of a hook) to be able to grab hold of the trees. – Squared lips. White rhinos are grazers, so flat and broad lips serve excellently to the purpose.
    3. Ears – Since black rhinos are less dependent on just one sense, as they have better eye-sight than white rhinos. This results in black rhinos having smaller and rounded ears. – White rhinos’ ears are tubular and very long, acting as little satellites to keep them aware of the surroundings.
    4. Volume of body – A black rhino that weighs between 800 – 1,400 kg. – A white rhino female weighs about 1,700 kg and a male weighs about 2,300 kg.
    5. Length of horn – The black rhinos have shorter front horns than white rhinos, but the second horn is slightly shorter. – A white rhino will usually have a longer front horn and a much shorter second horn.
    6. Habitat and Behaviour – The black rhinos are most commonly spotted in dense brush.Black rhinos have a reputation for being more aggressive and territorial than white rhinos. – White rhinos prefer to live in grasslands and fields. White rhinos are calmer and more placid.
    7. IUCN status – Critically Endangered (CR) – Near Threatened (NT)

    Incorrect

    Answer: A
    Explanation:
    Black Rhino and White Rhino are found in the African continent.
    Statement 1 is correct: Among black and white rhinos, black rhinos are the smaller of the two African rhino species. Black and white rhinos can be distinguished by the shape of their lips. Black rhinos have hooked upper lips, whereas white rhinos are characterized by a square lip.
    Statement 2 is correct and explains statement 1: Black rhinos are browsers, rather than grazers, meaning they are herbivores who do not feed on low-growing vegetation, and their pointed lip helps them feed on leaves from bushes and trees.
    A white rhinoceros has a very broad, flat, wide lip, which gives this creature the right idea as a grazer and requires a mouth designed for it. The broad muscular lips are ideal for tearing and tearing the grass.
    HABITATS: Semi-Desert Savannah, Woodlands, Forests, Wetlands
    DISTRIBUTION: Black rhinos or Dicerosbicornis are spread across 12 African countries including Kenya, Namibia, Botswana, South Africa, Tanzania, Zimbabwe, and Zambia.

    Additional Information:
    DIFFERENCES BETWEEN WHITE RHINO AND BLACK RHINO:

    PARAMETER – BLACK RHINO – WHITE RHINO
    1. Size of body – Smaller and more compact physical structure.It has shorter face compared to white rhino. However, it prefers keeping its head lifted because it spends most of its time eating trees. – Taller, bigger, and more complex physical structure.It has a long face, small eyes and a neck weakened because it does not need to be raised so large to feed.
    2. Structure of lips – Black rhinos feed on leaves and branches. Accordingly, their lips shape has evolved into a pointed lip (with the shape of a hook) to be able to grab hold of the trees. – Squared lips. White rhinos are grazers, so flat and broad lips serve excellently to the purpose.
    3. Ears – Since black rhinos are less dependent on just one sense, as they have better eye-sight than white rhinos. This results in black rhinos having smaller and rounded ears. – White rhinos’ ears are tubular and very long, acting as little satellites to keep them aware of the surroundings.
    4. Volume of body – A black rhino that weighs between 800 – 1,400 kg. – A white rhino female weighs about 1,700 kg and a male weighs about 2,300 kg.
    5. Length of horn – The black rhinos have shorter front horns than white rhinos, but the second horn is slightly shorter. – A white rhino will usually have a longer front horn and a much shorter second horn.
    6. Habitat and Behaviour – The black rhinos are most commonly spotted in dense brush.Black rhinos have a reputation for being more aggressive and territorial than white rhinos. – White rhinos prefer to live in grasslands and fields. White rhinos are calmer and more placid.
    7. IUCN status – Critically Endangered (CR) – Near Threatened (NT)

  4. Question 4 of 5
    4. Question

    4. Consider the following statements about rhododendrons:
    1. Rhododendrons are restricted to the Eastern Himalayas only.
    2. Usually, they are not found in the regions where mosses and lichens are found.
    3. The phenology of rhododendrons can be an important indicator of climate change.
    How many of the above statements are correct?

    Correct

    Answer: A
    Explanation:
    Rhododendron is mostly concentrated in the temperate regions of northern hemisphere especially in Sino-Himalayas (Eastern Himalayas and Western China). A good number are also found in Myanmar, China, Japan, Thailand, Malaysia, Indonesia, Philippines and New Guinea. Most rhododendrons are found in high rainfall, humid temperate regions of highly organic well-drained acidic soils and reach their magnificent development in the high hills and mountains that have dry, cool summer and rainy seasons.
    Statement 1 is incorrect: Rhododendrons are found in abundance in the eastern Himalayas in India but they are also distributed in the Western Himalayas and Western Ghats.

    ● Of the four parallel ranges in Himalayas, rhododendrons are practically absent in the Shiwaliks, a few are found in the lesser Himalayas and majority of them are in the greater Himalayas.
    ● Only one species occurs in Trans-Himalayan region located in extreme north-west of India (including the cold deserts of Jammu and Kashmir and Himachal Pradesh). The greater Himalayas are thus an ideal locality for rhododendrons in India.
    ● Besides this, a good number of species are found in north-eastern India particularly in Naga and Khasi hills.
    ● One subspecies, nilagiricum of Rhododendron arboreum occurs in Western Ghats.
    ● State-wise, Arunachal Pradesh harbours the maximum number of species (67 species) followed by Sikkim (36 species).
    Statement 2 is incorrect: Mosses and lichens are commonly found in association with rhododendron species. The epilithic/epiphytic rhododendrons grow on the moss-wrapped rocks and tree trunks associated with Agapetes, Vaccinium and orchids.
    Eight species are found growing epilithically/epiphytically, which are confined to eastern Himalayas and north-east India. Such species are entirely absent in western Himalayas.
    Statement 3 is correct: Rhododendron, meaning rose tree in Greek, is considered an indicator species for climate change. Generally, the flowering season for rhododendrons starts in March and continues till May. However, recently, flowering was found to begin as early as January for some species. This is an indication that those areas are getting warmer, and the phenology (the timing of biological events such as flowering and fruiting) of rhododendrons can be an important indicator of climate change.
    Of the 45 taxa of rhododendrons documented by the Botanical Survey of India, five are facing a high threat due to anthropological pressures and climate change.

    Incorrect

    Answer: A
    Explanation:
    Rhododendron is mostly concentrated in the temperate regions of northern hemisphere especially in Sino-Himalayas (Eastern Himalayas and Western China). A good number are also found in Myanmar, China, Japan, Thailand, Malaysia, Indonesia, Philippines and New Guinea. Most rhododendrons are found in high rainfall, humid temperate regions of highly organic well-drained acidic soils and reach their magnificent development in the high hills and mountains that have dry, cool summer and rainy seasons.
    Statement 1 is incorrect: Rhododendrons are found in abundance in the eastern Himalayas in India but they are also distributed in the Western Himalayas and Western Ghats.

    ● Of the four parallel ranges in Himalayas, rhododendrons are practically absent in the Shiwaliks, a few are found in the lesser Himalayas and majority of them are in the greater Himalayas.
    ● Only one species occurs in Trans-Himalayan region located in extreme north-west of India (including the cold deserts of Jammu and Kashmir and Himachal Pradesh). The greater Himalayas are thus an ideal locality for rhododendrons in India.
    ● Besides this, a good number of species are found in north-eastern India particularly in Naga and Khasi hills.
    ● One subspecies, nilagiricum of Rhododendron arboreum occurs in Western Ghats.
    ● State-wise, Arunachal Pradesh harbours the maximum number of species (67 species) followed by Sikkim (36 species).
    Statement 2 is incorrect: Mosses and lichens are commonly found in association with rhododendron species. The epilithic/epiphytic rhododendrons grow on the moss-wrapped rocks and tree trunks associated with Agapetes, Vaccinium and orchids.
    Eight species are found growing epilithically/epiphytically, which are confined to eastern Himalayas and north-east India. Such species are entirely absent in western Himalayas.
    Statement 3 is correct: Rhododendron, meaning rose tree in Greek, is considered an indicator species for climate change. Generally, the flowering season for rhododendrons starts in March and continues till May. However, recently, flowering was found to begin as early as January for some species. This is an indication that those areas are getting warmer, and the phenology (the timing of biological events such as flowering and fruiting) of rhododendrons can be an important indicator of climate change.
    Of the 45 taxa of rhododendrons documented by the Botanical Survey of India, five are facing a high threat due to anthropological pressures and climate change.

  5. Question 5 of 5
    5. Question

    5. Consider the following statements:
    Statement I: Antibiosis is a specific type of amensalism.
    Statement II: In antibiosis, one organism produces toxic chemicals that may have antagonistic effect on another organism but has no effect on the former.
    Which one of the following is correct in respect of the above statements?

    Correct

    Answer: A
    Explanation:
    Statement 1 is correct: Antibiosis is a specific type of amensalism. Amensalism defines the association between organisms of two different species in which one is inhibited or destroyed and the other is unaffected. There are two basic modes:
    ● Competition, in which a larger or stronger organism excludes a smaller or weaker one from living space or deprives it of food, and
    ● Antibiosis, in which one organism is unaffected but the other is damaged or killed by a chemical secretion.
    Statement 2 is correct and explains statement 1: Antibiosis can be defined as the antagonism resulting from the toxicity of secondary metabolites produced by one microorganism for other microorganisms.
    The classic demonstration of antibiosis is the destructive effect that the bread mold Penicillium has upon certain bacteria; the secretion, known as penicillin, has become a potent medicine in combating bacterial infections.
    Some higher plants secrete substances that inhibit the growth of—or kill outright—nearby competing plants. An example is the black walnut (Juglans nigra), which secretes juglone, a substance that destroys many herbaceous plants within its root zone.
    Antibiosis may involve an important role in plant disease suppression by certain bacteria and fungi. Microorganism performs an active role in the suppression of pathogens by secreting one or more antibiotics.

    Incorrect

    Answer: A
    Explanation:
    Statement 1 is correct: Antibiosis is a specific type of amensalism. Amensalism defines the association between organisms of two different species in which one is inhibited or destroyed and the other is unaffected. There are two basic modes:
    ● Competition, in which a larger or stronger organism excludes a smaller or weaker one from living space or deprives it of food, and
    ● Antibiosis, in which one organism is unaffected but the other is damaged or killed by a chemical secretion.
    Statement 2 is correct and explains statement 1: Antibiosis can be defined as the antagonism resulting from the toxicity of secondary metabolites produced by one microorganism for other microorganisms.
    The classic demonstration of antibiosis is the destructive effect that the bread mold Penicillium has upon certain bacteria; the secretion, known as penicillin, has become a potent medicine in combating bacterial infections.
    Some higher plants secrete substances that inhibit the growth of—or kill outright—nearby competing plants. An example is the black walnut (Juglans nigra), which secretes juglone, a substance that destroys many herbaceous plants within its root zone.
    Antibiosis may involve an important role in plant disease suppression by certain bacteria and fungi. Microorganism performs an active role in the suppression of pathogens by secreting one or more antibiotics.

window.wpAdvQuizInitList = window.wpAdvQuizInitList || []; window.wpAdvQuizInitList.push({ id: '#wpAdvQuiz_581', init: { quizId: 581, mode: 0, globalPoints: 10, timelimit: 0, resultsGrade: [0], bo: 0, qpp: 0, catPoints: [10], formPos: 0, lbn: "Finish quiz", json: {"2699":{"type":"single","id":2699,"catId":0,"points":2,"correct":[0,1,0,0]},"2700":{"type":"single","id":2700,"catId":0,"points":2,"correct":[1,0,0,0]},"2701":{"type":"single","id":2701,"catId":0,"points":2,"correct":[1,0,0,0]},"2702":{"type":"single","id":2702,"catId":0,"points":2,"correct":[1,0,0,0]},"2703":{"type":"single","id":2703,"catId":0,"points":2,"correct":[1,0,0,0]}} } });




Day-541 | Daily MCQs | UPSC Prelims | CURRENT DEVELOPMENTS

Day-541

Time limit: 0

Quiz-summary

0 of 5 questions completed

Questions:

  1. 1
  2. 2
  3. 3
  4. 4
  5. 5

Information

DAILY MCQ

You have already completed the quiz before. Hence you can not start it again.

Quiz is loading...

You must sign in or sign up to start the quiz.

You have to finish following quiz, to start this quiz:

Results

0 of 5 questions answered correctly

Your time:

Time has elapsed

You have reached 0 of 0 points, (0)

Categories

  1. Not categorized 0%
  1. 1
  2. 2
  3. 3
  4. 4
  5. 5
  1. Answered
  2. Review
  1. Question 1 of 5
    1. Question

    1. With reference to the Gender Social Norms Index, consider the following statements:
    1. It is published annually by the World Economic Forum (WEF).
    2. It benchmarks the people’s attitudes towards women across four key dimensions.
    Which of the statements given above is/are correct?

    Correct

    Answer: B
    Explanation:
    Context: The UNDP’s 2023 Gender Social Norms Index (GSNI) provides insights into the persistence of the biases and their impact on various aspects of women’s lives.
    Statement 1 is incorrect: It is published annually by the United Nations Development Programme.
    Statement 2 is correct: The UNDP tracked people’s attitudes towards women in four dimensions: political, educational, economic and physical integrity. The UNDP reports that nearly 90% of people still hold at least one bias against women.
    Additional information:
    Findings in gender social norms 2023:
    ▪ Political Participation and Representation: Biases in gender social norms contribute to a lack of equality in political participation. Approximately half of the world’s population believes men make better political leaders, while two in five believe men make better business executives.
    ▪ Economic Empowerment: Despite progress in education, gender gaps in economic empowerment persist. The increase in women’s education has not translated into better economic outcomes. In 59 countries where adult women are more educated than men, the average income gap is 39%.

    Incorrect

    Answer: B
    Explanation:
    Context: The UNDP’s 2023 Gender Social Norms Index (GSNI) provides insights into the persistence of the biases and their impact on various aspects of women’s lives.
    Statement 1 is incorrect: It is published annually by the United Nations Development Programme.
    Statement 2 is correct: The UNDP tracked people’s attitudes towards women in four dimensions: political, educational, economic and physical integrity. The UNDP reports that nearly 90% of people still hold at least one bias against women.
    Additional information:
    Findings in gender social norms 2023:
    ▪ Political Participation and Representation: Biases in gender social norms contribute to a lack of equality in political participation. Approximately half of the world’s population believes men make better political leaders, while two in five believe men make better business executives.
    ▪ Economic Empowerment: Despite progress in education, gender gaps in economic empowerment persist. The increase in women’s education has not translated into better economic outcomes. In 59 countries where adult women are more educated than men, the average income gap is 39%.

  2. Question 2 of 5
    2. Question

    2. With reference to the SACRED Portal, consider the following statements:
    1. It is developed by the Ministry of Social Justice and Empowerment.
    2. It provides assisted living devices to eligible senior citizens suffering from any of the age-related disabilities.
    How many of the statements given above is/are correct?

    Correct

    Answer: A
    Explanation:
    Context: In a first of its kind move, the government has come up with Senior Able Citizens for Re-Employment in Dignity(SACRED) Portal, an online employment exchange platform to cater to senior citizens seeking job opportunities.
    Statement 1 is correct: It is a portal developed by the Ministry of Social Justice and Empowerment.
    Statement 2 is incorrect: The portal provides opportunities for senior citizens to find suitable jobs and work options that match their preferences and skills.
    Additional information:
    ▪ SACRED Portal empowers the senior citizens and provide them with opportunities for re-employment.
    ▪ This portal specifically targets citizens aged 60 years and above, focusing on the needs and requirements of this age group.
    ▪ The SACRED Portal employs a virtual matching system that aligns the preferences of senior citizens with private enterprises seeking experienced individuals.
    ▪ Through job opportunities, the SACRED Portal helps senior citizens attain financial security and reduces their dependence on external support.

    Incorrect

    Answer: A
    Explanation:
    Context: In a first of its kind move, the government has come up with Senior Able Citizens for Re-Employment in Dignity(SACRED) Portal, an online employment exchange platform to cater to senior citizens seeking job opportunities.
    Statement 1 is correct: It is a portal developed by the Ministry of Social Justice and Empowerment.
    Statement 2 is incorrect: The portal provides opportunities for senior citizens to find suitable jobs and work options that match their preferences and skills.
    Additional information:
    ▪ SACRED Portal empowers the senior citizens and provide them with opportunities for re-employment.
    ▪ This portal specifically targets citizens aged 60 years and above, focusing on the needs and requirements of this age group.
    ▪ The SACRED Portal employs a virtual matching system that aligns the preferences of senior citizens with private enterprises seeking experienced individuals.
    ▪ Through job opportunities, the SACRED Portal helps senior citizens attain financial security and reduces their dependence on external support.

  3. Question 3 of 5
    3. Question

    3. Consider the following pairs:
    Operations – Related events
    1. Operation Kaveri – Evacuation of citizens during crisis in Afghanistan
    2. Operation Ajay – Evacuation of citizens during crisis in Israel
    3. Operation Rahat – Evacuation of citizens during crisis in Yemen
    How many of the pairs given above are correctly matched?

    Correct

    Answer: B
    Explanation:
    Context: Operation Ajay has been launched to facilitate the evacuation of Indians after a series of attacks on Israeli towns by Hamas militants.
    Operations – Related events
    1) Operation Kaveri – Evacuation of citizens during crisis in Sudan
    2) Operation Ajay – Evacuation of citizens during crisis in Israel
    3) Operation Rahat – Evacuation of citizens during crisis in Yemen
    ▪ Operation Kaveri: Operation Kaveri is a code name for India’s evacuation effort to bring back its citizens stranded in Sudan amid intense fighting between the army and a rival paramilitary force there. The operation involves the deployment of Indian Navy’s INS Sumedha, a stealth offshore patrol vessel, and two Indian Air Force C-130J special operations aircraft on standby in Jeddah.
    ▪ Operation Ajay: Operation Ajay is to bring back Indians from Israel as the conflict escalated with Israel retaliating against Hamas’s attack.
    ▪ Operation Rahat: In 2015, a conflict raged between the Yemeni government and Houthi rebels. Thousands of Indians were stranded and Yemen was not accessible by air due to a no-fly zone announced by Saudi Arabia. Under Operation Raahat, India evacuated nearly 5,600 people from Yemen.
    ▪ Operation Devi Shakti: It was India’s complex mission to evacuate its citizens and Afghan partners from Kabul after its swift takeover by the Taliban.

    Incorrect

    Answer: B
    Explanation:
    Context: Operation Ajay has been launched to facilitate the evacuation of Indians after a series of attacks on Israeli towns by Hamas militants.
    Operations – Related events
    1) Operation Kaveri – Evacuation of citizens during crisis in Sudan
    2) Operation Ajay – Evacuation of citizens during crisis in Israel
    3) Operation Rahat – Evacuation of citizens during crisis in Yemen
    ▪ Operation Kaveri: Operation Kaveri is a code name for India’s evacuation effort to bring back its citizens stranded in Sudan amid intense fighting between the army and a rival paramilitary force there. The operation involves the deployment of Indian Navy’s INS Sumedha, a stealth offshore patrol vessel, and two Indian Air Force C-130J special operations aircraft on standby in Jeddah.
    ▪ Operation Ajay: Operation Ajay is to bring back Indians from Israel as the conflict escalated with Israel retaliating against Hamas’s attack.
    ▪ Operation Rahat: In 2015, a conflict raged between the Yemeni government and Houthi rebels. Thousands of Indians were stranded and Yemen was not accessible by air due to a no-fly zone announced by Saudi Arabia. Under Operation Raahat, India evacuated nearly 5,600 people from Yemen.
    ▪ Operation Devi Shakti: It was India’s complex mission to evacuate its citizens and Afghan partners from Kabul after its swift takeover by the Taliban.

  4. Question 4 of 5
    4. Question

    4. Consider the following statements:
    Statement I: Markets in Crypto Assets (MiCA) regulation is a European Union initiative which aims to bring largely unregulated cryptocurrency markets under government regulation.
    Statement II: MiCA aims to regulate Non Fungible Tokens (NFTs) and digital assets that would qualify as transferable securities.
    Which one of the following is correct in respect of the above statements?

    Correct

    Answer: C
    Explanation:
    Context: Recently, the European Parliament has approved the Markets in Crypto Assets (MiCA) regulation. The regulation will come into force after formal approval by member states.
    Statement 1 is correct: Markets in Crypto Assets (MiCA) regulation is an European Union initiative as the world’s first comprehensive set of rules that aims to bring largely unregulated cryptocurrency markets under government regulation.
    Statement 2 is incorrect: MiCA will not regulate digital assets that would qualify as transferable securities and function like shares or their equivalent and other crypto assets that already qualify as financial instruments under existing regulation. It will also exclude nonfungible tokens (NFTs).
    Additional information:
    Assets Covered under MiCA:
    ● The MiCA legislation will apply to crypto assets. These are broadly defined as “a digital representation of a value or a right that uses cryptography for security. And is in the form of a coin or a token or any other digital medium which may be transferred and stored electronically.
    ● This definition implies that it will apply not only to traditional cryptocurrencies like Bitcoin and Ethereum but also to newer ones like stablecoins.
    ● MiCA will also establish new rules for three types of stablecoins.
    Assets Out of MiCA’s Scope:
    ● MiCA will also not regulate central bank digital currencies issued by the European Central Bank and digital assets issued by national central banks of EU member countries when acting in their capacity as monetary authorities.

    Incorrect

    Answer: C
    Explanation:
    Context: Recently, the European Parliament has approved the Markets in Crypto Assets (MiCA) regulation. The regulation will come into force after formal approval by member states.
    Statement 1 is correct: Markets in Crypto Assets (MiCA) regulation is an European Union initiative as the world’s first comprehensive set of rules that aims to bring largely unregulated cryptocurrency markets under government regulation.
    Statement 2 is incorrect: MiCA will not regulate digital assets that would qualify as transferable securities and function like shares or their equivalent and other crypto assets that already qualify as financial instruments under existing regulation. It will also exclude nonfungible tokens (NFTs).
    Additional information:
    Assets Covered under MiCA:
    ● The MiCA legislation will apply to crypto assets. These are broadly defined as “a digital representation of a value or a right that uses cryptography for security. And is in the form of a coin or a token or any other digital medium which may be transferred and stored electronically.
    ● This definition implies that it will apply not only to traditional cryptocurrencies like Bitcoin and Ethereum but also to newer ones like stablecoins.
    ● MiCA will also establish new rules for three types of stablecoins.
    Assets Out of MiCA’s Scope:
    ● MiCA will also not regulate central bank digital currencies issued by the European Central Bank and digital assets issued by national central banks of EU member countries when acting in their capacity as monetary authorities.

  5. Question 5 of 5
    5. Question

    5. With reference to the Doha Political Declaration, consider the following statements:
    1. It is a 10-year plan to put the world’s most vulnerable countries back on track to achieving the Sustainable Development Goals (SDG).
    2. It consists of three key focus areas of poverty, environment and potential of science and technology.
    Which of the statements given above is/are correct?

    Correct

    Answer: A
    Explanation:
    Context: Doha Political Declaration has been adopted by world leaders to fast-track progress in least developed countries.
    Statement 1 is correct: It is a 10-year plan to put the world’s 46 most vulnerable countries back on track to achieving the Sustainable Development Goals (SDG).
    LDCs are a group of countries identified by the United Nations as having the lowest indicators of socioeconomic development. These countries are characterized by high levels of poverty, low levels of human capital, and limited access to basic services, such as healthcare and education.
    Currently, there are 46 countries on the United Nations’ list of LDCs.
    Africa (33);
    Asia (9);
    Caribbean (1): Haiti;
    Pacific (3): Kiribati, Solomon Islands and Tuvalu.
    Statement 2 is incorrect: It consisted of six key focus areas:
    ▪ eradicating poverty;
    ▪ leveraging the potential of science and technology;
    ▪ addressing climate change;
    ▪ environmental degradation;
    ▪ recovering from COVID-19 pandemic;
    ▪ building resilience against future shocks for risk-informed sustainable development
    Additional information:
    ● 5th United Nations Conference on the Least Developed Countries (LDC5) concluded with adoption of the ‘Doha Political Declaration’ by the world leaders.
    ● The declaration is a key outcome of the second part of LDC5 conference held under the theme “From Potential to Prosperity” in Qatar.
    Need for the Declaration:
    ● The 46 LDCs are being hit the hardest by multiple crises including the Covid-19 Pandemic, Climate Crisis, growing Inequalities, rising Debt Burdens and economic shocks.
    ● They have contributed minimally to Carbon Dioxide Emissions, but disproportionately bear the burden of climate change impacts.
    ● These countries, which include 33 African nations, face the challenge of high debt costs while having inadequate liquidity to provide essential services.

    Incorrect

    Answer: A
    Explanation:
    Context: Doha Political Declaration has been adopted by world leaders to fast-track progress in least developed countries.
    Statement 1 is correct: It is a 10-year plan to put the world’s 46 most vulnerable countries back on track to achieving the Sustainable Development Goals (SDG).
    LDCs are a group of countries identified by the United Nations as having the lowest indicators of socioeconomic development. These countries are characterized by high levels of poverty, low levels of human capital, and limited access to basic services, such as healthcare and education.
    Currently, there are 46 countries on the United Nations’ list of LDCs.
    Africa (33);
    Asia (9);
    Caribbean (1): Haiti;
    Pacific (3): Kiribati, Solomon Islands and Tuvalu.
    Statement 2 is incorrect: It consisted of six key focus areas:
    ▪ eradicating poverty;
    ▪ leveraging the potential of science and technology;
    ▪ addressing climate change;
    ▪ environmental degradation;
    ▪ recovering from COVID-19 pandemic;
    ▪ building resilience against future shocks for risk-informed sustainable development
    Additional information:
    ● 5th United Nations Conference on the Least Developed Countries (LDC5) concluded with adoption of the ‘Doha Political Declaration’ by the world leaders.
    ● The declaration is a key outcome of the second part of LDC5 conference held under the theme “From Potential to Prosperity” in Qatar.
    Need for the Declaration:
    ● The 46 LDCs are being hit the hardest by multiple crises including the Covid-19 Pandemic, Climate Crisis, growing Inequalities, rising Debt Burdens and economic shocks.
    ● They have contributed minimally to Carbon Dioxide Emissions, but disproportionately bear the burden of climate change impacts.
    ● These countries, which include 33 African nations, face the challenge of high debt costs while having inadequate liquidity to provide essential services.

window.wpAdvQuizInitList = window.wpAdvQuizInitList || []; window.wpAdvQuizInitList.push({ id: '#wpAdvQuiz_580', init: { quizId: 580, mode: 0, globalPoints: 10, timelimit: 0, resultsGrade: [0], bo: 0, qpp: 0, catPoints: [10], formPos: 0, lbn: "Finish quiz", json: {"2694":{"type":"single","id":2694,"catId":0,"points":2,"correct":[0,1,0,0]},"2695":{"type":"single","id":2695,"catId":0,"points":2,"correct":[1,0,0,0]},"2696":{"type":"single","id":2696,"catId":0,"points":2,"correct":[0,1,0,0]},"2697":{"type":"single","id":2697,"catId":0,"points":2,"correct":[0,0,1,0]},"2698":{"type":"single","id":2698,"catId":0,"points":2,"correct":[1,0,0,0]}} } });




Day-540 | Daily MCQs | UPSC Prelims | ECONOMY

Day-540

Time limit: 0

Quiz-summary

0 of 5 questions completed

Questions:

  1. 1
  2. 2
  3. 3
  4. 4
  5. 5

Information

DAILY MCQ

You have already completed the quiz before. Hence you can not start it again.

Quiz is loading...

You must sign in or sign up to start the quiz.

You have to finish following quiz, to start this quiz:

Results

0 of 5 questions answered correctly

Your time:

Time has elapsed

You have reached 0 of 0 points, (0)

Categories

  1. Not categorized 0%
  1. 1
  2. 2
  3. 3
  4. 4
  5. 5
  1. Answered
  2. Review
  1. Question 1 of 5
    1. Question

    1. Consider the following:
    1. Jowar
    2. Ragi
    3. Tur
    4. Gram
    5. Jute
    6. Wheat
    How many of the above are Kharif crops that receive MSP?

    Correct

    Answer: A
    Explanation:
    ● Minimum Support Price (MSP) is a form of market intervention by the Government of India to insure agricultural producers against any sharp fall in farm prices. The minimum support prices are announced by the Government of India at the beginning of the sowing season for certain crops on the basis of the recommendations of the Commission for Agricultural Costs and Prices (CACP). MSP is price fixed by the Government of India to protect the producer (farmers) against excessive fall in price during bumper production years.
    ● Minimum Support Prices for all Rabi and Kharif crops are given below:
    ● List of Kharif crops:
    o Paddy
    o Jowar
    o Bajra
    o Maize
    o Ragi
    o Arhar (Tur)
    o Moong
    o Cotton
    o Groundnut in shell
    o Sunflower seed
    o Soybean
    o Sesamum
    o Niger Seed
    ● List of Rabi crops:
    o Wheat
    o Barley
    o Gram
    o Masur (Lentil)
    o Rapeseed & Mustard
    o Safflower
    o Toria

    Incorrect

    Answer: A
    Explanation:
    ● Minimum Support Price (MSP) is a form of market intervention by the Government of India to insure agricultural producers against any sharp fall in farm prices. The minimum support prices are announced by the Government of India at the beginning of the sowing season for certain crops on the basis of the recommendations of the Commission for Agricultural Costs and Prices (CACP). MSP is price fixed by the Government of India to protect the producer (farmers) against excessive fall in price during bumper production years.
    ● Minimum Support Prices for all Rabi and Kharif crops are given below:
    ● List of Kharif crops:
    o Paddy
    o Jowar
    o Bajra
    o Maize
    o Ragi
    o Arhar (Tur)
    o Moong
    o Cotton
    o Groundnut in shell
    o Sunflower seed
    o Soybean
    o Sesamum
    o Niger Seed
    ● List of Rabi crops:
    o Wheat
    o Barley
    o Gram
    o Masur (Lentil)
    o Rapeseed & Mustard
    o Safflower
    o Toria

  2. Question 2 of 5
    2. Question

    2. Consider the following statements:
    Statement I: Marginal standing facility (MSF) rate is always greater than the repo rate.
    Statement II: MSF can be availed by dipping SLR up to a limit at a penal rate of interest.
    Which one of the following is correct in respect of the above statements?

    Correct

    Answer: A
    Explanation:
    Marginal Standing Facility (MSF):
    ● It is a facility introduced in 2011, under which scheduled commercial banks can borrow additional amounts of overnight money (over and above what is available to them through repo rate) from the Reserve Bank by dipping into their SLR portfolio up to a limit (2%) at a penal rate of interest. This provides a safety valve against unanticipated liquidity shocks to the banking system.
    ● When banks take loans from RBI at Repo rate, banks need to keep Government Securities with RBI, but this security is in addition to the requirement under SLR. Banks cannot keep SLR securities to avail loan from RBI at Repo Rate.
    ● But under MSF, banks can borrow from the RBI by dipping into the SLR reserve. This means that the banks can keep 2% of the SLR securities with RBI (i.e., the SLR can go down up to 2% below the normal SLR requirement) and can borrow cash from RBI. This is called Marginal Standing Facility (MSF). But for deepening the SLR below their limit’s banks have to give some penalty. As a result, MSF is greater than the repo rate.
    ● MSF Rate = Repo Rate + 0.25%

    Incorrect

    Answer: A
    Explanation:
    Marginal Standing Facility (MSF):
    ● It is a facility introduced in 2011, under which scheduled commercial banks can borrow additional amounts of overnight money (over and above what is available to them through repo rate) from the Reserve Bank by dipping into their SLR portfolio up to a limit (2%) at a penal rate of interest. This provides a safety valve against unanticipated liquidity shocks to the banking system.
    ● When banks take loans from RBI at Repo rate, banks need to keep Government Securities with RBI, but this security is in addition to the requirement under SLR. Banks cannot keep SLR securities to avail loan from RBI at Repo Rate.
    ● But under MSF, banks can borrow from the RBI by dipping into the SLR reserve. This means that the banks can keep 2% of the SLR securities with RBI (i.e., the SLR can go down up to 2% below the normal SLR requirement) and can borrow cash from RBI. This is called Marginal Standing Facility (MSF). But for deepening the SLR below their limit’s banks have to give some penalty. As a result, MSF is greater than the repo rate.
    ● MSF Rate = Repo Rate + 0.25%

  3. Question 3 of 5
    3. Question

    3. Consider the following statements:
    1. Currency convertibility refers to the freedom to convert the domestic currency into other currencies.
    2. India practices full current account as well as capital account convertibility.
    Which of the statements given above is/are correct?

    Correct

    Answer: A
    Explanation:
    Statement 1 is correct: Currency convertibility refers to the freedom to convert the domestic currency into other internationally accepted currencies and vice versa at the market-determined exchange rate.
    Current Account Convertibility:
    ● RBI allows full conversion of Rupee into foreign currencies and foreign currencies into Rupee (at market price i.e., Nominal Exchange Rate) for any transactions under the current account of balance of payments (BoP). This is called “rupee is fully convertible at current account”. So, suppose someone wants to import commodities worth $10 billion in India then RBI will convert that many Rupees into $10 billion without any restriction for import purposes.
    ● As a part of the economic reforms initiated in 1991 rupee was made fully convertible at current account in 1993.
    Capital Account Convertibility:
    ● RBI does not allow full conversion of Rupee into foreign currencies and foreign currencies into Rupee for transactions falling under the capital account of BoP. There are restrictions/limits imposed by the RBI and government on the value of transactions that anybody can do under a capital account. This is called “rupee is partially convertible at capital account”.
    ● There are restrictions on how much ECB can be raised in a particular year, there are restrictions on how much foreign investors can invest in Government securities, there are restrictions on how much individuals/companies can do capital account kind of transactions etc.
    ● Capital account convertibility leads to free exchange of currency at market rates and an unrestricted mobility of capital. It is beneficial for a country because it increases inflow of foreign investment. But the flip side is that it could destabilize an economy due to massive capital flows in and out of the country.
    ● Rupee will move to full capital account convertibility once the macroeconomic parameters like current account deficit, fiscal deficit, external debt, inflation become stable at low range and there is resilience to absorb shocks related to capital outflows.
    ● Since capital convertibility is risky and makes foreign exchange rate more volatile, it is introduced only sometime after the introduction of convertibility on current account when the exchange rate of currency of a country is relatively stable, deficit in balance of payments is well under control and enough foreign exchange reserves are available with the Central Bank.
    Statement 2 is incorrect: In India, there is full current account convertibility since August 20, 1993. But there is partial capital account convertibility in India.
    Balance of payments: The balance of payments (BOP), also known as the balance of international payments, is a statement of all transactions made between entities in one country and the rest of the world over a defined period, such as a quarter or a year. It summarizes all transactions that a country’s individuals, companies, and government bodies complete with individuals, companies, and government bodies outside the country.

    Incorrect

    Answer: A
    Explanation:
    Statement 1 is correct: Currency convertibility refers to the freedom to convert the domestic currency into other internationally accepted currencies and vice versa at the market-determined exchange rate.
    Current Account Convertibility:
    ● RBI allows full conversion of Rupee into foreign currencies and foreign currencies into Rupee (at market price i.e., Nominal Exchange Rate) for any transactions under the current account of balance of payments (BoP). This is called “rupee is fully convertible at current account”. So, suppose someone wants to import commodities worth $10 billion in India then RBI will convert that many Rupees into $10 billion without any restriction for import purposes.
    ● As a part of the economic reforms initiated in 1991 rupee was made fully convertible at current account in 1993.
    Capital Account Convertibility:
    ● RBI does not allow full conversion of Rupee into foreign currencies and foreign currencies into Rupee for transactions falling under the capital account of BoP. There are restrictions/limits imposed by the RBI and government on the value of transactions that anybody can do under a capital account. This is called “rupee is partially convertible at capital account”.
    ● There are restrictions on how much ECB can be raised in a particular year, there are restrictions on how much foreign investors can invest in Government securities, there are restrictions on how much individuals/companies can do capital account kind of transactions etc.
    ● Capital account convertibility leads to free exchange of currency at market rates and an unrestricted mobility of capital. It is beneficial for a country because it increases inflow of foreign investment. But the flip side is that it could destabilize an economy due to massive capital flows in and out of the country.
    ● Rupee will move to full capital account convertibility once the macroeconomic parameters like current account deficit, fiscal deficit, external debt, inflation become stable at low range and there is resilience to absorb shocks related to capital outflows.
    ● Since capital convertibility is risky and makes foreign exchange rate more volatile, it is introduced only sometime after the introduction of convertibility on current account when the exchange rate of currency of a country is relatively stable, deficit in balance of payments is well under control and enough foreign exchange reserves are available with the Central Bank.
    Statement 2 is incorrect: In India, there is full current account convertibility since August 20, 1993. But there is partial capital account convertibility in India.
    Balance of payments: The balance of payments (BOP), also known as the balance of international payments, is a statement of all transactions made between entities in one country and the rest of the world over a defined period, such as a quarter or a year. It summarizes all transactions that a country’s individuals, companies, and government bodies complete with individuals, companies, and government bodies outside the country.

  4. Question 4 of 5
    4. Question

    4. With reference to the PM Vishwakarma scheme, consider the following statements:
    1. It is a centrally sponsored scheme for the upliftment of the traditional artisans and craftsmen.
    2. It is being implemented jointly by the Ministry of Skill Development and Entrepreneurship and the Ministry of Social Justice and Empowerment.
    3. It will be initially implemented over a five-year period.
    How many of the statements given above are correct?

    Correct

    Answer: A
    Explanation:
    Context: PM inaugurates Vishwakarma Scheme for the upliftment of the marginalised.
    Statement 1 is incorrect: It is a central sector scheme for the upliftment of the traditional artisans and craftsmen.
    Statement 2 is incorrect: The Ministry of Micro, Small and Medium Enterprises (MoMSME) is the Nodal Ministry for the Scheme. The Scheme will be jointly implemented by the MoMSME, the Ministry of Skill Development and Entrepreneurship and the Department of Financial Services, Ministry of Finance, Government of India.
    Statement 3 is correct: It will initially be implemented over a five-year period, up to the fiscal year 2027-28.
    Additional information:
    Objectives of the Scheme:
    ● Enable recognition of artisans and craftspeople as Vishwakarma.
    ● Provide skill upgradation and training opportunities.
    ● Support access to modern tools.
    ● Facilitate easy access to collateral-free credit.
    ● Encourage digital transactions and empowerment.
    ● Create a platform for brand promotion and market linkages.
    Scheme Eligibility:
    ● Artisans or craftspersons engaged in one of the specified traditional trades.
    ● Minimum age of 18 years.
    ● Not availed loans under similar credit-based schemes in the past 5 years, except for MUDRA and SVANidhi beneficiaries who have fully repaid their loans.
    ● Registration and benefits restricted to one member per family.
    ● Government employees and their family members are not eligible.

    Incorrect

    Answer: A
    Explanation:
    Context: PM inaugurates Vishwakarma Scheme for the upliftment of the marginalised.
    Statement 1 is incorrect: It is a central sector scheme for the upliftment of the traditional artisans and craftsmen.
    Statement 2 is incorrect: The Ministry of Micro, Small and Medium Enterprises (MoMSME) is the Nodal Ministry for the Scheme. The Scheme will be jointly implemented by the MoMSME, the Ministry of Skill Development and Entrepreneurship and the Department of Financial Services, Ministry of Finance, Government of India.
    Statement 3 is correct: It will initially be implemented over a five-year period, up to the fiscal year 2027-28.
    Additional information:
    Objectives of the Scheme:
    ● Enable recognition of artisans and craftspeople as Vishwakarma.
    ● Provide skill upgradation and training opportunities.
    ● Support access to modern tools.
    ● Facilitate easy access to collateral-free credit.
    ● Encourage digital transactions and empowerment.
    ● Create a platform for brand promotion and market linkages.
    Scheme Eligibility:
    ● Artisans or craftspersons engaged in one of the specified traditional trades.
    ● Minimum age of 18 years.
    ● Not availed loans under similar credit-based schemes in the past 5 years, except for MUDRA and SVANidhi beneficiaries who have fully repaid their loans.
    ● Registration and benefits restricted to one member per family.
    ● Government employees and their family members are not eligible.

  5. Question 5 of 5
    5. Question

    5. Consider the following financial instruments:
    1. Postal Deposits
    2. Public Provident Fund
    3. Sukanya Samriddhi Scheme
    How many of the above are small savings instruments?

    Correct

    Answer: C
    Explanation:
    Context: Despite successive hikes in the interest rates on several small savings instruments (SSIs), the returns are still significantly lower than expected.
    Small savings instruments help individuals achieve their financial goals over a particular period. They are the major source of household savings in India.
    Collections from all small savings instruments are credited to the National Small Savings Fund (NSSF).
    The small savings instrument basket comprises 12 instruments which can be classified into three categories:
    ▪ Postal Deposits: (comprising savings account, recurring deposits, time deposits of varying maturities and monthly income scheme).
    ▪ Savings Certificates: National Small Savings Certificate (NSC) and Kisan Vikas Patra (KVP).
    ▪ Social Security Schemes: Sukanya Samriddhi Scheme, Public Provident Fund (PPF) and Senior Citizens ‘Savings Scheme (SCSS).
    Rates of Small Saving Instruments:
    ▪ The rates for small saving instruments are announced quarterly.
    ▪ Theoretically, it is based on yields of G-Secs of corresponding maturity but political factors also influence the rate change.
    ▪ The Shyamala Gopinath panel (2010) constituted on the Small Saving (SS) Scheme had suggested a market-linked interest rate system for SS Schemes.
    Formula for Small Savings Rates:
    ▪ It is used to calculate the interest rates for various SSIs in India and is based on the average quarterly yields on G-Secs in the first 3 of the preceding 4 months.
    ▪ The formula is used to decide how much interest to pay to savers who invest in SS schemes.

    Incorrect

    Answer: C
    Explanation:
    Context: Despite successive hikes in the interest rates on several small savings instruments (SSIs), the returns are still significantly lower than expected.
    Small savings instruments help individuals achieve their financial goals over a particular period. They are the major source of household savings in India.
    Collections from all small savings instruments are credited to the National Small Savings Fund (NSSF).
    The small savings instrument basket comprises 12 instruments which can be classified into three categories:
    ▪ Postal Deposits: (comprising savings account, recurring deposits, time deposits of varying maturities and monthly income scheme).
    ▪ Savings Certificates: National Small Savings Certificate (NSC) and Kisan Vikas Patra (KVP).
    ▪ Social Security Schemes: Sukanya Samriddhi Scheme, Public Provident Fund (PPF) and Senior Citizens ‘Savings Scheme (SCSS).
    Rates of Small Saving Instruments:
    ▪ The rates for small saving instruments are announced quarterly.
    ▪ Theoretically, it is based on yields of G-Secs of corresponding maturity but political factors also influence the rate change.
    ▪ The Shyamala Gopinath panel (2010) constituted on the Small Saving (SS) Scheme had suggested a market-linked interest rate system for SS Schemes.
    Formula for Small Savings Rates:
    ▪ It is used to calculate the interest rates for various SSIs in India and is based on the average quarterly yields on G-Secs in the first 3 of the preceding 4 months.
    ▪ The formula is used to decide how much interest to pay to savers who invest in SS schemes.

window.wpAdvQuizInitList = window.wpAdvQuizInitList || []; window.wpAdvQuizInitList.push({ id: '#wpAdvQuiz_579', init: { quizId: 579, mode: 0, globalPoints: 10, timelimit: 0, resultsGrade: [0], bo: 0, qpp: 0, catPoints: [10], formPos: 0, lbn: "Finish quiz", json: {"2689":{"type":"single","id":2689,"catId":0,"points":2,"correct":[1,0,0,0]},"2690":{"type":"single","id":2690,"catId":0,"points":2,"correct":[1,0,0,0]},"2691":{"type":"single","id":2691,"catId":0,"points":2,"correct":[1,0,0,0]},"2692":{"type":"single","id":2692,"catId":0,"points":2,"correct":[1,0,0,0]},"2693":{"type":"single","id":2693,"catId":0,"points":2,"correct":[0,0,1,0]}} } });




Day-539 | Daily MCQs | UPSC Prelims | SCIENCE AND TECHNOLOGY

Day-539

Time limit: 0

Quiz-summary

0 of 5 questions completed

Questions:

  1. 1
  2. 2
  3. 3
  4. 4
  5. 5

Information

DAILY MCQ

You have already completed the quiz before. Hence you can not start it again.

Quiz is loading...

You must sign in or sign up to start the quiz.

You have to finish following quiz, to start this quiz:

Results

0 of 5 questions answered correctly

Your time:

Time has elapsed

You have reached 0 of 0 points, (0)

Categories

  1. Not categorized 0%
  1. 1
  2. 2
  3. 3
  4. 4
  5. 5
  1. Answered
  2. Review
  1. Question 1 of 5
    1. Question

    1. With reference to various generations of DNA sequencing, consider the following pairs:
    Generation – Explanation
    1. 1st generation – Parallel sequencing
    2. 2nd generation – Sequencing of clonal DNA population
    3. 3rd generation – Single DNA molecule sequencing
    How many of the above pairs are correct?

    Correct

    Answer: A
    Explanation:
    DNA sequencing-
    DNA sequencing is a technique used to determine the nucleotide sequence of DNA (deoxyribonucleic acid). The nucleotide sequence is the most fundamental level of knowledge of a gene or genome. It is the blueprint that contains the instructions for building an organism, and no understanding of genetic function or evolution could be complete without obtaining this information.
    ● Pair 1 is incorrect: The 1st generation sequencing involved sequencing of clonal DNA population. Under this, a population of nested, truncated DNA molecules was produced that represented each of the sites of that particular nucleotide in the template DNA. The molecules were separated according to size in a procedure called electrophoresis, and the inferred nucleotide sequence was deduced by a computer. It emerged in the 1970s. It included the Maxam-Gilbert method, discovered by American molecular biologists Allan M. Maxam and Walter Gilbert, and the Sanger method (or dideoxy method), discovered by English biochemist Frederick Sanger.
    ● Pair 2 is incorrect: The 2nd generation sequencing is also known as the Next Generation Sequencing (NGS). The key feature of NGS is parallelisation of a large number of reactions. This is achieved through automation and miniaturisation of the reactions. This reduces cost without compromising on the efficiency of the method.
    ● Pair 3 is correct: The 3rd generation sequencing technologies offer the capability for single molecule real-time sequencing of longer reads, and detection of DNA modification. This method is currently under active development, and it is expected that there will be improvements to the high error rates.

    Incorrect

    Answer: A
    Explanation:
    DNA sequencing-
    DNA sequencing is a technique used to determine the nucleotide sequence of DNA (deoxyribonucleic acid). The nucleotide sequence is the most fundamental level of knowledge of a gene or genome. It is the blueprint that contains the instructions for building an organism, and no understanding of genetic function or evolution could be complete without obtaining this information.
    ● Pair 1 is incorrect: The 1st generation sequencing involved sequencing of clonal DNA population. Under this, a population of nested, truncated DNA molecules was produced that represented each of the sites of that particular nucleotide in the template DNA. The molecules were separated according to size in a procedure called electrophoresis, and the inferred nucleotide sequence was deduced by a computer. It emerged in the 1970s. It included the Maxam-Gilbert method, discovered by American molecular biologists Allan M. Maxam and Walter Gilbert, and the Sanger method (or dideoxy method), discovered by English biochemist Frederick Sanger.
    ● Pair 2 is incorrect: The 2nd generation sequencing is also known as the Next Generation Sequencing (NGS). The key feature of NGS is parallelisation of a large number of reactions. This is achieved through automation and miniaturisation of the reactions. This reduces cost without compromising on the efficiency of the method.
    ● Pair 3 is correct: The 3rd generation sequencing technologies offer the capability for single molecule real-time sequencing of longer reads, and detection of DNA modification. This method is currently under active development, and it is expected that there will be improvements to the high error rates.

  2. Question 2 of 5
    2. Question

    2. With reference to the Nanocomposites, consider the following statements:
    1. They are made up of two or more materials.
    2. All the combining materials must have similar properties for better fusion.
    3. There are no nanocomposite materials found in nature.
    How many of the above statements are correct?

    Correct

    Answer: A
    Explanation:
    A nanocomposite combines two or more materials, of which at least one is a nanomaterial (size 100 nanometer or less), with different physical and chemical properties. Nanocomposite materials are designed to exhibit properties that exceed, sometimes drastically, the capabilities of the sum of their constituent parts.

    ● Statement 1 is correct: The word composite means anything which is made up of two or more materials. nanocomposites are made up of two or more materials.
    ● Statement 2 is incorrect: The basic idea behind creating nanocomposites is to fuse two or more material of different physical and chemical properties so that the resultant material has qualities of all those materials.
    ● Statement 3 is incorrect: Some nanocomposites are found in nature, for example in the structure of the abalone shell and bone.

    Incorrect

    Answer: A
    Explanation:
    A nanocomposite combines two or more materials, of which at least one is a nanomaterial (size 100 nanometer or less), with different physical and chemical properties. Nanocomposite materials are designed to exhibit properties that exceed, sometimes drastically, the capabilities of the sum of their constituent parts.

    ● Statement 1 is correct: The word composite means anything which is made up of two or more materials. nanocomposites are made up of two or more materials.
    ● Statement 2 is incorrect: The basic idea behind creating nanocomposites is to fuse two or more material of different physical and chemical properties so that the resultant material has qualities of all those materials.
    ● Statement 3 is incorrect: Some nanocomposites are found in nature, for example in the structure of the abalone shell and bone.

  3. Question 3 of 5
    3. Question

    3. With respect to Lagrange Points, consider the following statements:
    1. At these points, gravitational force acting on a spacecraft is equal to the magnetic force, which is exerted by the planet.
    2. At these points, the fuel consumption for a spacecraft is reduced.
    3. There are five Lagrange points in the Earth-Sun system.
    How many of the above statements are correct?

    Correct

    Answer: B
    Explanation:
    Lagrange points are positions in space where objects sent there tend to stay at that position.
    ● Statement 1 is incorrect: At Lagrange points, the gravitational pull of two large masses precisely equals the centripetal force required for a small object to move with them.
    ● Statement 2 is correct: These points in space can be used by the spacecraft to reduce fuel consumption needed to remain in position.

    ● Statement 3 is correct: There are 5 such points – L1, L2, L3, L4 and L5 in the Earth-Sun system. In fact, any planet-sun system in a solar system has 5 such points. Of the five Lagrange points, three are unstable and two are stable. The unstable Lagrange points – labelled L1, L2 and L3 – lie along the line connecting the two large masses. The stable Lagrange points – labelled L4 and L5 – form the apex of two equilateral triangles that have the large masses at their vertices. L4 leads the orbit of earth and L5 follows.

    Incorrect

    Answer: B
    Explanation:
    Lagrange points are positions in space where objects sent there tend to stay at that position.
    ● Statement 1 is incorrect: At Lagrange points, the gravitational pull of two large masses precisely equals the centripetal force required for a small object to move with them.
    ● Statement 2 is correct: These points in space can be used by the spacecraft to reduce fuel consumption needed to remain in position.

    ● Statement 3 is correct: There are 5 such points – L1, L2, L3, L4 and L5 in the Earth-Sun system. In fact, any planet-sun system in a solar system has 5 such points. Of the five Lagrange points, three are unstable and two are stable. The unstable Lagrange points – labelled L1, L2 and L3 – lie along the line connecting the two large masses. The stable Lagrange points – labelled L4 and L5 – form the apex of two equilateral triangles that have the large masses at their vertices. L4 leads the orbit of earth and L5 follows.

  4. Question 4 of 5
    4. Question

    4. With reference to Artificial Intelligence (AI), consider the following statements:
    1. Self-driving cars are reactive machines.
    2. Self-awareness level of AI is a theoretical concept.
    3. General AI will be more intelligent than human beings.
    How many of the above statements are correct?

    Correct

    Answer: A
    Explanation:
    Artificial Intelligence can be divided in various types, there are mainly two types of categorization which are based on capabilities and functionality of AI.
    Based on capabilities
    1. Weak AI or Narrow AI: Narrow AI is a type of AI which is able to perform a dedicated task with intelligence. Narrow AI can fail in unpredictable ways if it goes beyond its limits. Some Examples of Narrow AI are playing chess, purchasing suggestions on e-commerce site, self-driving cars, speech recognition, and image recognition.
    2. General AI: General AI is a type of intelligence which could perform any intellectual task with efficiency like a human. These machines could think like humans. Currently, there is no such system exist which could come under general AI and can perform any task as perfect as a human.
    3. Super AI: Super AI is a level of Intelligence of Systems at which machines could surpass human intelligence and can perform any task better than human with cognitive properties. It is an outcome of general AI.
    Based on functionality
    1. Reactive Machines: Purely reactive machines are the most basic types of Artificial Intelligence. These machines only focus on current scenarios and react on it as per possible best action.
    2. Limited Memory: Limited memory machines can store past experiences or some data for a short period of time. Self-driving cars are one of the best examples of Limited Memory systems. These cars can store recent speed of nearby cars, the distance of other cars, speed limit, and other information to navigate the road.
    3. Theory of Mind: Theory of Mind AI should understand the human emotions, people, beliefs, and be able to interact socially like humans. These are still not developed, but researchers are making lots of efforts and improvement for developing such AI machines.
    4. Self-Awareness: Self-awareness AI is the future of Artificial Intelligence. These machines will be super intelligent, and will have their own consciousness, sentiments, and self-awareness. Self-Awareness AI does not exist in reality still, and it is a hypothetical concept.
    ● Statement 1 is incorrect: Self driving cars are Limited memory type. These cars store recent data of nearby activities and react in a swift manner. Purely reactive machines prompt to the scenario placed and react based on best possible actions.
    ● Statement 2 is correct: Self Awareness based AI is still a theoretical concept because these machines is considered to be the ultimate level of Artificial Intelligence.
    ● Statement 3 is incorrect: General AI is at par with human intelligence. They can think and act like humans.

    Incorrect

    Answer: A
    Explanation:
    Artificial Intelligence can be divided in various types, there are mainly two types of categorization which are based on capabilities and functionality of AI.
    Based on capabilities
    1. Weak AI or Narrow AI: Narrow AI is a type of AI which is able to perform a dedicated task with intelligence. Narrow AI can fail in unpredictable ways if it goes beyond its limits. Some Examples of Narrow AI are playing chess, purchasing suggestions on e-commerce site, self-driving cars, speech recognition, and image recognition.
    2. General AI: General AI is a type of intelligence which could perform any intellectual task with efficiency like a human. These machines could think like humans. Currently, there is no such system exist which could come under general AI and can perform any task as perfect as a human.
    3. Super AI: Super AI is a level of Intelligence of Systems at which machines could surpass human intelligence and can perform any task better than human with cognitive properties. It is an outcome of general AI.
    Based on functionality
    1. Reactive Machines: Purely reactive machines are the most basic types of Artificial Intelligence. These machines only focus on current scenarios and react on it as per possible best action.
    2. Limited Memory: Limited memory machines can store past experiences or some data for a short period of time. Self-driving cars are one of the best examples of Limited Memory systems. These cars can store recent speed of nearby cars, the distance of other cars, speed limit, and other information to navigate the road.
    3. Theory of Mind: Theory of Mind AI should understand the human emotions, people, beliefs, and be able to interact socially like humans. These are still not developed, but researchers are making lots of efforts and improvement for developing such AI machines.
    4. Self-Awareness: Self-awareness AI is the future of Artificial Intelligence. These machines will be super intelligent, and will have their own consciousness, sentiments, and self-awareness. Self-Awareness AI does not exist in reality still, and it is a hypothetical concept.
    ● Statement 1 is incorrect: Self driving cars are Limited memory type. These cars store recent data of nearby activities and react in a swift manner. Purely reactive machines prompt to the scenario placed and react based on best possible actions.
    ● Statement 2 is correct: Self Awareness based AI is still a theoretical concept because these machines is considered to be the ultimate level of Artificial Intelligence.
    ● Statement 3 is incorrect: General AI is at par with human intelligence. They can think and act like humans.

  5. Question 5 of 5
    5. Question

    5. Babesiosis disease affects which of the following components of the human blood?

    Correct

    Answer: D
    Explanation
    Babesiosis is caused by microscopic parasites that infect red blood cells. The disease is transmitted mainly by ticks which become infected by feeding on infected cattle, roe deer and rodents, which are the main reservoirs for this parasite. Human babesiosis infection is treated with antibiotics and quinine. There is no vaccine available. Prevention from Babesiosis is limited to avoiding tick bites and promoting personal measures of protection against ticks.

    Incorrect

    Answer: D
    Explanation
    Babesiosis is caused by microscopic parasites that infect red blood cells. The disease is transmitted mainly by ticks which become infected by feeding on infected cattle, roe deer and rodents, which are the main reservoirs for this parasite. Human babesiosis infection is treated with antibiotics and quinine. There is no vaccine available. Prevention from Babesiosis is limited to avoiding tick bites and promoting personal measures of protection against ticks.

window.wpAdvQuizInitList = window.wpAdvQuizInitList || []; window.wpAdvQuizInitList.push({ id: '#wpAdvQuiz_578', init: { quizId: 578, mode: 0, globalPoints: 10, timelimit: 0, resultsGrade: [0], bo: 0, qpp: 0, catPoints: [10], formPos: 0, lbn: "Finish quiz", json: {"2684":{"type":"single","id":2684,"catId":0,"points":2,"correct":[1,0,0,0]},"2685":{"type":"single","id":2685,"catId":0,"points":2,"correct":[1,0,0,0]},"2686":{"type":"single","id":2686,"catId":0,"points":2,"correct":[0,1,0,0]},"2687":{"type":"single","id":2687,"catId":0,"points":2,"correct":[1,0,0,0]},"2688":{"type":"single","id":2688,"catId":0,"points":2,"correct":[0,0,0,1]}} } });




Day-538 | Daily MCQs | UPSC Prelims | ENVIRONMENT AND ECOLOGY

Day-538

Time limit: 0

Quiz-summary

0 of 5 questions completed

Questions:

  1. 1
  2. 2
  3. 3
  4. 4
  5. 5

Information

To attempt the Quiz, simply click on START Button.

You have already completed the quiz before. Hence you can not start it again.

Quiz is loading...

You must sign in or sign up to start the quiz.

You have to finish following quiz, to start this quiz:

Results

0 of 5 questions answered correctly

Your time:

Time has elapsed

You have reached 0 of 0 points, (0)

Categories

  1. Not categorized 0%
  1. 1
  2. 2
  3. 3
  4. 4
  5. 5
  1. Answered
  2. Review
  1. Question 1 of 5
    1. Question

    1. Consider the following pairs:

    Wetlands                                           States
    1. Pallikaranai marshland                             Kerala
    2. Kole wetlands                                      Madhya Pradesh
    3. Nanda lake                                               Maharashtra

    How many of the above pairs are correctly matched?

    Correct

    Answer: D
    Explanation:

    Options 1, 2 and 3 are incorrectly matched:
    1. Pallikaranai marshland Tamil Nadu
    ● The Pallikaranai Marsh is one of the last remaining
    natural wetlands of Chennai city.
    ● Designated as Ramsar site in India.
    ● It is locally known by the generic Tamil name

    ‘kazhuveli’ which means a flood plain or water-
    logged area.

    ● On its eastern periphery, the Marsh is flanked by the
    Buckingham Canal.
    ● Pallikaranai Marsh is a part of the vast Bay of Bengal
    Large Marine Ecosystem. It is one of the few natural coastal aquatic habitats that qualify as a
    wetland in India. The seasonal dynamics in water volume, spread and chemistry, both
    historical and current, have rendered the Pallikaranai Marsh a biodiversity-rich South Indian
    wetland.
    ● The Site is threatened by invasive and non-native species, household sewage, urban
    wastewater and droughts.

    Incorrect

    Answer: D
    Explanation:

    Options 1, 2 and 3 are incorrectly matched:
    1. Pallikaranai marshland Tamil Nadu
    ● The Pallikaranai Marsh is one of the last remaining
    natural wetlands of Chennai city.
    ● Designated as Ramsar site in India.
    ● It is locally known by the generic Tamil name

    ‘kazhuveli’ which means a flood plain or water-
    logged area.

    ● On its eastern periphery, the Marsh is flanked by the
    Buckingham Canal.
    ● Pallikaranai Marsh is a part of the vast Bay of Bengal
    Large Marine Ecosystem. It is one of the few natural coastal aquatic habitats that qualify as a
    wetland in India. The seasonal dynamics in water volume, spread and chemistry, both
    historical and current, have rendered the Pallikaranai Marsh a biodiversity-rich South Indian
    wetland.
    ● The Site is threatened by invasive and non-native species, household sewage, urban
    wastewater and droughts.

  2. Question 2 of 5
    2. Question

    2. In the context of methane emissions, consider the following statements:

    1. Methane emissions from the waste and energy sector together are higher
    than those from agricultural sources.

    2. Methane contributes to the formation of ground-level ozone.
    Which of the statements given above is/are correct?

    Correct

     Answer: B
    Explanation:

    Recently, COP28 President (from UAE) has urged the oil and gas industry to phase out methane
    emissions by 2030. Methane is the second most abundant GHG after CO2. It is 86 times more
    powerful than CO2 over a period of 20 years. Over a period of 100 years, the Global warming potential
    of methane is about 25 times higher than that of CO2.
    It is a short-lived climate pollutant that has a lifespan of about 12-13 years, after which it gets
    decomposed to Carbon Dioxide.
    Statement 1 is incorrect: The methane emissions from waste sector, energy sector and industrial
    process/product use clubbed together amount to 26% of total methane emissions, while the
    agricultural sector alone emits about 74% of methane into the atmosphere.

    Natural Sources of Methane-
    • Wetlands
    • Microorganisms from water-logged soils
    • Decaying organic matter, landfills
    • Hydro dams and reservoirs
    • Melting Arctic Sea and soil (source of methane gas hydrates)
    • Wildfires
    Anthropogenic sources-
    • Burning of fossil fuels
    • Waste
    • Agriculture (paddy fields)
    • Livestock (methanogens in digestive tract of ruminants)
    Statement 2 is correct: Methane is a main precursor of tropospheric ozone which is a powerful GHG
    and air pollutant that leads to formation of photochemical smog on warm sunny days. Oxidation of
    methane contributes to the formation of bad ozone.

    Incorrect

     Answer: B
    Explanation:

    Recently, COP28 President (from UAE) has urged the oil and gas industry to phase out methane
    emissions by 2030. Methane is the second most abundant GHG after CO2. It is 86 times more
    powerful than CO2 over a period of 20 years. Over a period of 100 years, the Global warming potential
    of methane is about 25 times higher than that of CO2.
    It is a short-lived climate pollutant that has a lifespan of about 12-13 years, after which it gets
    decomposed to Carbon Dioxide.
    Statement 1 is incorrect: The methane emissions from waste sector, energy sector and industrial
    process/product use clubbed together amount to 26% of total methane emissions, while the
    agricultural sector alone emits about 74% of methane into the atmosphere.

    Natural Sources of Methane-
    • Wetlands
    • Microorganisms from water-logged soils
    • Decaying organic matter, landfills
    • Hydro dams and reservoirs
    • Melting Arctic Sea and soil (source of methane gas hydrates)
    • Wildfires
    Anthropogenic sources-
    • Burning of fossil fuels
    • Waste
    • Agriculture (paddy fields)
    • Livestock (methanogens in digestive tract of ruminants)
    Statement 2 is correct: Methane is a main precursor of tropospheric ozone which is a powerful GHG
    and air pollutant that leads to formation of photochemical smog on warm sunny days. Oxidation of
    methane contributes to the formation of bad ozone.

  3. Question 3 of 5
    3. Question

    3. Consider the following statements:

    1. The COP16 of UNFCCC led to the establishment of the Adaptation Fund.

    2. The Loss and Damage Fund is the outcome of the COP26 of UNFCCC.

    Which of the statements given above is/are incorrect?

    Correct

    Answer: C
    Explanation:

    Statement 1 is incorrect: The Cancun (Mexico) Conference of UNFCCC (COP16 in 2010) led to
    establishment of Green Climate Fund (not Adaptation Fund). The general concept for GCF was first
    proposed at the Conference of the Parties (COP) to the UNFCCC in Copenhagen, Denmark (COP 15).

    THE PT QUEST 2024 LUKMAAN IAS
    ● It is designed as an operating entity of the Convention’s financial mechanism and is
    headquartered in Incheon, Republic of Korea.
    ● It is governed by a 24 Board member Board, representing countries, and receives guidance
    from the Conference of the Parties to the Convention (COP).
    ● The Fund pays particular attention to the needs of societies that are highly vulnerable to the
    effects of climate change, in particular Least Developed Countries (LDCs), Small Island
    Developing States (SIDS), and African States.
    Adaptation Fund was established in 2001 to finance concrete adaptation projects and programmes
    in developing country Parties to the Kyoto Protocol that are particularly vulnerable to the adverse
    effects of climate change.
    ● The Adaptation Fund serves the Paris Agreement under the CMA with respect to all Paris
    Agreement matters, effective 1 January 2019 and with this, it no longer serves as financing
    mechanism for Kyoto Protocol now. The Parties had decided that once the share of proceeds
    becomes available under Article 6 of the Paris Agreement, the Adaptation Fund shall no longer
    serve the Kyoto Protocol.
    ● The Adaptation Fund is supervised and managed by the Adaptation Fund Board (AFB). The
    AFB is composed of 16 members and 16 alternates and meets at least twice a year.
    Statement 2 is incorrect: Loss and Damage Fund was created at the 27th Conference of Parties
    (COP27, Sharm-el-Sheikh, Egypt) to the United Nations Framework Convention on Climate Change
    (UNFCCC), focusing on particularly vulnerable countries hit hard by floods, droughts and other climate
    disasters. For the first time, countries recognized the need for finance to respond to loss and damage
    associated with the catastrophic effects of climate change and agreed to the establishing of a fund
    and the necessary funding arrangements.
    However, the decisions regarding its funding and operationalisation would be taken at the upcoming
    UNFCCC COP28 (Dubai, UAE).

    Incorrect

    Answer: C
    Explanation:

    Statement 1 is incorrect: The Cancun (Mexico) Conference of UNFCCC (COP16 in 2010) led to
    establishment of Green Climate Fund (not Adaptation Fund). The general concept for GCF was first
    proposed at the Conference of the Parties (COP) to the UNFCCC in Copenhagen, Denmark (COP 15).

    THE PT QUEST 2024 LUKMAAN IAS
    ● It is designed as an operating entity of the Convention’s financial mechanism and is
    headquartered in Incheon, Republic of Korea.
    ● It is governed by a 24 Board member Board, representing countries, and receives guidance
    from the Conference of the Parties to the Convention (COP).
    ● The Fund pays particular attention to the needs of societies that are highly vulnerable to the
    effects of climate change, in particular Least Developed Countries (LDCs), Small Island
    Developing States (SIDS), and African States.
    Adaptation Fund was established in 2001 to finance concrete adaptation projects and programmes
    in developing country Parties to the Kyoto Protocol that are particularly vulnerable to the adverse
    effects of climate change.
    ● The Adaptation Fund serves the Paris Agreement under the CMA with respect to all Paris
    Agreement matters, effective 1 January 2019 and with this, it no longer serves as financing
    mechanism for Kyoto Protocol now. The Parties had decided that once the share of proceeds
    becomes available under Article 6 of the Paris Agreement, the Adaptation Fund shall no longer
    serve the Kyoto Protocol.
    ● The Adaptation Fund is supervised and managed by the Adaptation Fund Board (AFB). The
    AFB is composed of 16 members and 16 alternates and meets at least twice a year.
    Statement 2 is incorrect: Loss and Damage Fund was created at the 27th Conference of Parties
    (COP27, Sharm-el-Sheikh, Egypt) to the United Nations Framework Convention on Climate Change
    (UNFCCC), focusing on particularly vulnerable countries hit hard by floods, droughts and other climate
    disasters. For the first time, countries recognized the need for finance to respond to loss and damage
    associated with the catastrophic effects of climate change and agreed to the establishing of a fund
    and the necessary funding arrangements.
    However, the decisions regarding its funding and operationalisation would be taken at the upcoming
    UNFCCC COP28 (Dubai, UAE).

  4. Question 4 of 5
    4. Question

    4. Consider the following statements about the International Whaling
    Commission (IWC):

    Statement I: The International Whaling Commission (IWC) is responsible for
    setting catch limits for commercial whaling.
    Statement II: It regulates commercial, aboriginal subsistence and special permit
    whaling.

    Which one of the following is correct in respect of the above statements?

    Correct

    Answer: C
    Explanation:

    The International Whaling Commission (IWC) was set up under the International Convention for the
    Regulation of Whaling which was signed in Washington D.C. on 2nd December 1946. The preamble to
    the Convention states that its purpose is to provide for the proper conservation of whale stocks and
    thus make possible the orderly development of the whaling industry.
    • Statement 1 is correct: The IWC is responsible for setting catch limits for commercial whaling
    (with the exceptions of Norway and Iceland, see below). The Commission receives advice on
    sustainability from its Scientific Committee before deciding catch limits, which are then set
    out in a document called the Schedule to the International Convention for the Regulation of
    Whaling.
    • An integral part of the Convention is its legally binding ‘Schedule.’ The Schedule sets out
    specific measures that the IWC has collectively decided are necessary in order to regulate
    whaling and conserve whale stocks.
    • These measures include catch limits (which may be zero as it the case for commercial whaling)
    by species and area, designating specified areas as whale sanctuaries, protection of calves
    and females accompanied by calves, and restrictions on hunting methods.
    • Statement 2 is incorrect: Special permit whaling (for scientific purposes) is not regulated by
    the Commission but by national governments, although none currently undertake this type
    of whaling.
    • The legal framework of the IWC is the International Convention for the Regulation of Whaling
    which recognises three different types of whaling: commercial, aboriginal subsistence and

    THE PT QUEST 2024 LUKMAAN IAS
    special permit (also known as scientific) whaling. It regulates only commercial whaling and
    aboriginal subsistence whaling.
    • Following considerable debate, a commercial whaling moratorium was adopted in 1982 and
    came into full force in 1986. The Commission continues to regulate Aboriginal Subsistence
    Whaling, which is conducted by indigenous communities, often in remote parts of the world.

    Incorrect

    Answer: C
    Explanation:

    The International Whaling Commission (IWC) was set up under the International Convention for the
    Regulation of Whaling which was signed in Washington D.C. on 2nd December 1946. The preamble to
    the Convention states that its purpose is to provide for the proper conservation of whale stocks and
    thus make possible the orderly development of the whaling industry.
    • Statement 1 is correct: The IWC is responsible for setting catch limits for commercial whaling
    (with the exceptions of Norway and Iceland, see below). The Commission receives advice on
    sustainability from its Scientific Committee before deciding catch limits, which are then set
    out in a document called the Schedule to the International Convention for the Regulation of
    Whaling.
    • An integral part of the Convention is its legally binding ‘Schedule.’ The Schedule sets out
    specific measures that the IWC has collectively decided are necessary in order to regulate
    whaling and conserve whale stocks.
    • These measures include catch limits (which may be zero as it the case for commercial whaling)
    by species and area, designating specified areas as whale sanctuaries, protection of calves
    and females accompanied by calves, and restrictions on hunting methods.
    • Statement 2 is incorrect: Special permit whaling (for scientific purposes) is not regulated by
    the Commission but by national governments, although none currently undertake this type
    of whaling.
    • The legal framework of the IWC is the International Convention for the Regulation of Whaling
    which recognises three different types of whaling: commercial, aboriginal subsistence and

    THE PT QUEST 2024 LUKMAAN IAS
    special permit (also known as scientific) whaling. It regulates only commercial whaling and
    aboriginal subsistence whaling.
    • Following considerable debate, a commercial whaling moratorium was adopted in 1982 and
    came into full force in 1986. The Commission continues to regulate Aboriginal Subsistence
    Whaling, which is conducted by indigenous communities, often in remote parts of the world.

  5. Question 5 of 5
    5. Question

    5. Consider the following statements about Fungi:

    1. Some fungi exhibit symbiotic relationships with blue-green algae.
    2. Some fungi are useful in decontaminating pollutants.
    3. Some fungi are capable of photosynthesis.
    4. Some fungi are good decomposers.

    How many of the above statements are correct?

    Correct

    Answer: C
    Explanation:

    Fungi are eukaryotic organisms which include the yeasts, rusts, smuts, mildews, moulds, and
    mushrooms. Historically, fungi were included in the plant kingdom; however, because fungi lack
    chlorophyll and are distinguished by unique structural and physiological features (i.e., components of
    the cell wall and cell membrane), they have been separated from plants.
    • Statement 1 is correct: Lichen represent the symbiotic relationship between blue-green
    algae (cyanobacteria) and fungi. The Lichen are a huge group of composite organisms.
    (Composite organisms are simply organisms that are made up of two or more independent
    organisms.)
    • Lichens are made up of an alga or cyanobacterium partner (called the photobiont), and a
    fungus partner (called the mycobiont). The mycobiont provides the lichen with minerals
    (structure), water, and shelter, while the photobiont provides the lichen with nutrients
    through photosynthesis.
    • Statement 2 is correct: Mycoremediation is a method of bioremediation using fungi to
    decontaminate contaminated areas. The white-rot fungi are much effective in degrading a
    wide range of organic molecules due to their release of extra-cellular lignin-modifying
    enzymes, with a low substrate-specificity, so they can act upon various molecules that are
    broadly similar to lignin.
    • The white-rot fungus ‘Phanerochaete chrysosporium’ is an ideal model for bioremediation by
    fungi, since it is more efficient than other fungi or microorganisms in degrading toxic or
    insoluble materials. It presents simultaneous oxidative and reductive mechanisms which
    permit its use in many different situations, regarding the type of contamination, its degree,
    and the nature of the site itself.
    • Statement 3 is incorrect: There are some fungi such as mushroom which look like plants, but
    they cannot carry out photosynthesis. Since fungi lack chlorophyll, they are not capable of
    making their own food through photosynthesis. Also, they can not ingest food like animals
    do. Instead, they feed by absorption of nutrients from the environment around them. They
    possess hyphae which secrete digestive enzymes. These enzymes help in breaking down the
    substrate, making it easier for the fungus to absorb nutrients which the substrate contains.
    • Statement 4 is correct: Fungi are among the primary decomposers in many environments
    such as forests. Most of the fungi are saprophytes, feeding on dead and decaying material.

    Incorrect

    Answer: C
    Explanation:

    Fungi are eukaryotic organisms which include the yeasts, rusts, smuts, mildews, moulds, and
    mushrooms. Historically, fungi were included in the plant kingdom; however, because fungi lack
    chlorophyll and are distinguished by unique structural and physiological features (i.e., components of
    the cell wall and cell membrane), they have been separated from plants.
    • Statement 1 is correct: Lichen represent the symbiotic relationship between blue-green
    algae (cyanobacteria) and fungi. The Lichen are a huge group of composite organisms.
    (Composite organisms are simply organisms that are made up of two or more independent
    organisms.)
    • Lichens are made up of an alga or cyanobacterium partner (called the photobiont), and a
    fungus partner (called the mycobiont). The mycobiont provides the lichen with minerals
    (structure), water, and shelter, while the photobiont provides the lichen with nutrients
    through photosynthesis.
    • Statement 2 is correct: Mycoremediation is a method of bioremediation using fungi to
    decontaminate contaminated areas. The white-rot fungi are much effective in degrading a
    wide range of organic molecules due to their release of extra-cellular lignin-modifying
    enzymes, with a low substrate-specificity, so they can act upon various molecules that are
    broadly similar to lignin.
    • The white-rot fungus ‘Phanerochaete chrysosporium’ is an ideal model for bioremediation by
    fungi, since it is more efficient than other fungi or microorganisms in degrading toxic or
    insoluble materials. It presents simultaneous oxidative and reductive mechanisms which
    permit its use in many different situations, regarding the type of contamination, its degree,
    and the nature of the site itself.
    • Statement 3 is incorrect: There are some fungi such as mushroom which look like plants, but
    they cannot carry out photosynthesis. Since fungi lack chlorophyll, they are not capable of
    making their own food through photosynthesis. Also, they can not ingest food like animals
    do. Instead, they feed by absorption of nutrients from the environment around them. They
    possess hyphae which secrete digestive enzymes. These enzymes help in breaking down the
    substrate, making it easier for the fungus to absorb nutrients which the substrate contains.
    • Statement 4 is correct: Fungi are among the primary decomposers in many environments
    such as forests. Most of the fungi are saprophytes, feeding on dead and decaying material.

window.wpAdvQuizInitList = window.wpAdvQuizInitList || []; window.wpAdvQuizInitList.push({ id: '#wpAdvQuiz_577', init: { quizId: 577, mode: 0, globalPoints: 10, timelimit: 0, resultsGrade: [0], bo: 0, qpp: 0, catPoints: [10], formPos: 0, lbn: "Finish quiz", json: {"2679":{"type":"single","id":2679,"catId":0,"points":2,"correct":[0,0,0,1]},"2680":{"type":"single","id":2680,"catId":0,"points":2,"correct":[0,0,0,1]},"2681":{"type":"single","id":2681,"catId":0,"points":2,"correct":[0,0,1,0]},"2682":{"type":"single","id":2682,"catId":0,"points":2,"correct":[0,0,1,0]},"2683":{"type":"single","id":2683,"catId":0,"points":2,"correct":[0,0,1,0]}} } });




Day-536 | Daily MCQs | UPSC Prelims | ECONOMY

Day-536

Time limit: 0

Quiz-summary

0 of 5 questions completed

Questions:

  1. 1
  2. 2
  3. 3
  4. 4
  5. 5

Information

To attempt the Quiz, simply click on START Button.

You have already completed the quiz before. Hence you can not start it again.

Quiz is loading...

You must sign in or sign up to start the quiz.

You have to finish following quiz, to start this quiz:

Results

0 of 5 questions answered correctly

Your time:

Time has elapsed

You have reached 0 of 0 points, (0)

Categories

  1. Not categorized 0%
  1. 1
  2. 2
  3. 3
  4. 4
  5. 5
  1. Answered
  2. Review
  1. Question 1 of 5
    1. Question

    1. Consider the following statements regarding ‘Accommodating transactions’
    under Balance of Payments (BoP):

    1. It refers to those transactions which are made independently of the state of
    the BoP.
    2. The official reserve transactions are considered as the accommodating item in
    the BoP.
    3. It compensates for the surplus or deficit brought by autonomous transactions.

    How many of the above statements are correct ?

    Correct

    Answer: B
    Explanation:

    Accommodating transactions:
    ● Statement 1 is incorrect: Accommodating transactions are also termed as the ‘below the line’
    items. It implies those economic transactions carried out with the purpose of meeting the gap
    in autonomous transactions. They are determined by the net consequences of the
    autonomous items, that is, whether the BoP is in surplus or deficit.
    ● Statement 2 is correct: The official reserve transactions are seen as the accommodating item
    in the BoP .
    ● Statement 3 is correct: Accommodating transactions compensate for the surplus or deficit
    brought about by autonomous transactions. It seeks to bring equality between the payments
    and receipts of foreign exchange.
    ● These transactions are carried out for maintaining the balance in the Balance of Payment
    Account. These transactions fall in the capital account only.
    ● These transactions happen in the Government sector only.
    Autonomous transaction:
    ● International economic transactions are called autonomous when transactions are made
    independently of the state of the BoP (for instance due to profit motive).
    ● Autonomous transactions will include imports and exports (current account transactions) and
    also lending or borrowing of loans (cross border) or payment or receipt of interests thereon.

    Incorrect

    Answer: B
    Explanation:

    Accommodating transactions:
    ● Statement 1 is incorrect: Accommodating transactions are also termed as the ‘below the line’
    items. It implies those economic transactions carried out with the purpose of meeting the gap
    in autonomous transactions. They are determined by the net consequences of the
    autonomous items, that is, whether the BoP is in surplus or deficit.
    ● Statement 2 is correct: The official reserve transactions are seen as the accommodating item
    in the BoP .
    ● Statement 3 is correct: Accommodating transactions compensate for the surplus or deficit
    brought about by autonomous transactions. It seeks to bring equality between the payments
    and receipts of foreign exchange.
    ● These transactions are carried out for maintaining the balance in the Balance of Payment
    Account. These transactions fall in the capital account only.
    ● These transactions happen in the Government sector only.
    Autonomous transaction:
    ● International economic transactions are called autonomous when transactions are made
    independently of the state of the BoP (for instance due to profit motive).
    ● Autonomous transactions will include imports and exports (current account transactions) and
    also lending or borrowing of loans (cross border) or payment or receipt of interests thereon.

  2. Question 2 of 5
    2. Question

    Recently Gresham’s Law was in the news. Which of the following statements
    is correct about it?

    Correct

     Answer: C

    Explanation:
    ● Sir Thomas Gresham lived from 1519 to 1579 and wrote about the value and minting of coins
    while working as a financier.
    ● It states that, in a fixed exchange rate system, “bad money drives out good”.
    ● It comes into play when the exchange rate between two currencies is fixed by the government
    at a certain ratio that is different from market exchange rate.
    Or
    ● When the price of a currency in terms of another currency is arbitrarily fixed by lawmakers, it
    leads to a shortage in the supply of the currency that is undervalued, while there is at the
    same time an over-supply of the currency that is overvalued.
    ● The law observes that legally overvalued currency will drive legally undervalued currency out
    of circulation.

    Incorrect

     Answer: C

    Explanation:
    ● Sir Thomas Gresham lived from 1519 to 1579 and wrote about the value and minting of coins
    while working as a financier.
    ● It states that, in a fixed exchange rate system, “bad money drives out good”.
    ● It comes into play when the exchange rate between two currencies is fixed by the government
    at a certain ratio that is different from market exchange rate.
    Or
    ● When the price of a currency in terms of another currency is arbitrarily fixed by lawmakers, it
    leads to a shortage in the supply of the currency that is undervalued, while there is at the
    same time an over-supply of the currency that is overvalued.
    ● The law observes that legally overvalued currency will drive legally undervalued currency out
    of circulation.

  3. Question 3 of 5
    3. Question

    Consider the following sectors:
    1. Drones and Drone Components
    2. White Goods
    3. Toys
    4. Specialty Steel
    5. Cottage industry
    6. Automobiles and Auto Components
    7. Leather and footwear
    How many of the above sectors are covered under the Production Linked
    Incentive Schemes?

    Correct

    3. Answer: A
    Explanation:
    ● Production Linked Incentive Scheme (PLI) was launched in April , 2020. PLI offers a production
    linked incentive to boost domestic manufacturing and attract large investments in various
    sectors.
    ● The scheme shall extend an incentive of 4% to 6% on incremental sales (over base year) of
    goods manufactured in India and covered under target segments, to eligible companies, for a
    period of five (5) years subsequent to the base year as defined.
    ● The Scheme is open for applications for a period of 4 months initially which may be extended.
    Support under the Scheme shall be provided for a period of five (5) years subsequent to the
    base year

    Incorrect

  4. Question 4 of 5
    4. Question

    4. Consider the following pairs:
    Terms Descriptions
    1. Tax Base – It is the total number of people who pay tax in the

    economy.

    2. Tax Buoyancy – It measures the responsiveness of tax revenue to changes

    in tax rate.

    3. Tax Elasticity – It is an indicator to measure efficiency and responsiveness
    of revenue mobilization in response to growth in the GDP

    How many of the above pairs are correctly matched?

    Correct

    4. Answer: D
    Explanation:
    ● Tax Base: Tax base is defined as the total value of the financial streams or assets on which
    tax can be imposed by the government. For example, in case of income tax, the tax base is
    all the income that can be taxed by the government (taxable income). If the minimum
    amount/threshold of the income on which tax is imposed is lowered, this will automatically
    increase (widen) the tax base; if it is raised, the tax base will be narrowed. In case of service
    tax, tax base is the value of services on which service tax is imposed. For example, if CCD sells
    tea/coffee worth Rs. 10 crore and of course it collects service tax/GST on 10 crores, so tax
    base will be Rs. 10 crores. In case of property tax, tax base is the value of property on which
    property tax is imposed. Because the size of the tax base influences the taxable revenues that
    are available to a government, the size and growth of the tax base is crucial to the planning
    efforts of any government.
    ● Tax Buoyancy: Tax buoyancy is an indicator to measure efficiency and responsiveness of
    revenue mobilization in response to growth in the GDP or National Income. It is measured
    as a ratio of growth in Tax Revenue to the growth in nominal GDP (i.e., ratio of percentage
    change in tax revenue to percentage change in nominal GDP). Tax buoyancy greater than one
    is good for the economy.
    ● Tax Elasticity: The tax elasticity measures the responsiveness of tax revenue to changes in tax
    rate and is defined as the ratio of percentage change in tax revenue to percentage change in
    tax rate.

    Incorrect

  5. Question 5 of 5
    5. Question

    5. Consider the following:
    1. Capital gains tax
    2. Stamp duty
    3. Property tax
    4. Securities transaction tax
    5. Luxury tax
    6. Equalization levy
    How many of the above are direct tax?

    Correct

    5. Answer: B
    Explanation:
    ● A direct tax is a tax that a person or organization pays directly to the entity that imposed it.
    Examples include income tax, real property tax, personal property tax, and taxes on assets, all
    of which are paid by an individual taxpayer directly to the government.
    ● List of direct tax:

    THE PT QUEST 2024 LUKMAAN IAS

    o Income Tax
    o Corporate Tax
    o Securities Transaction Tax
    o Capital Gains Tax
    o Gift Tax
    o Wealth Tax
    o Land revenue
    o Property tax
    o Dividend distribution tax
    o Equalization levy
    ● Indirect tax is the tax levied on the consumption of goods and services. It is not directly levied
    on the income of a person. Instead, he/she has to pay the tax along with the price of goods or
    services bought by the seller.
    ● List of indirect tax:
    o Sales Tax
    o Value Added Tax (VAT)
    o Goods & Services Tax (GST)
    o Luxury tax
    o Stamp duty
    o Custom Duty

    Incorrect

window.wpAdvQuizInitList = window.wpAdvQuizInitList || []; window.wpAdvQuizInitList.push({ id: '#wpAdvQuiz_575', init: { quizId: 575, mode: 0, globalPoints: 10, timelimit: 0, resultsGrade: [0], bo: 0, qpp: 0, catPoints: [10], formPos: 0, lbn: "Finish quiz", json: {"2668":{"type":"single","id":2668,"catId":0,"points":2,"correct":[0,1,0,0]},"2669":{"type":"single","id":2669,"catId":0,"points":2,"correct":[0,0,1,0]},"2670":{"type":"single","id":2670,"catId":0,"points":2,"correct":[1,0,0,0]},"2671":{"type":"single","id":2671,"catId":0,"points":2,"correct":[0,0,0,1]},"2672":{"type":"single","id":2672,"catId":0,"points":2,"correct":[0,1,0,0]}} } });




Day-537 | Daily MCQs | UPSC Prelims | HISTORY

Day-537

Time limit: 0

Quiz-summary

0 of 5 questions completed

Questions:

  1. 1
  2. 2
  3. 3
  4. 4
  5. 5

Information

To attempt the Quiz, simply click on START Button.

You have already completed the quiz before. Hence you can not start it again.

Quiz is loading...

You must sign in or sign up to start the quiz.

You have to finish following quiz, to start this quiz:

Results

0 of 5 questions answered correctly

Your time:

Time has elapsed

You have reached 0 of 0 points, (0)

Categories

  1. Not categorized 0%
Your result has been entered into leaderboard
Loading
captcha
  1. 1
  2. 2
  3. 3
  4. 4
  5. 5
  1. Answered
  2. Review
  1. Question 1 of 5
    1. Question
    2 points

    1. With reference to the history of India, the term ‘suyurghals’ refers to:

    Correct

    Answer: B
    Explanation:

    ‘Suyurghals’ is the practice of granting revenue producing lands to scholars, theologians and
    others by the Mughal emperors which is tax free.
    It was an Mongolian tradition. The suyurghals holder was free from any administrative or
    judicial interference from the central government. There was no compulsory obligation to
    provide for military contingents.

    Incorrect

    Answer: B
    Explanation:

    ‘Suyurghals’ is the practice of granting revenue producing lands to scholars, theologians and
    others by the Mughal emperors which is tax free.
    It was an Mongolian tradition. The suyurghals holder was free from any administrative or
    judicial interference from the central government. There was no compulsory obligation to
    provide for military contingents.

  2. Question 2 of 5
    2. Question
    2 points

    2. Consider the following statements in the context of Mughals:
    1. They traced their paternal ancestry from Chingiz Khan and maternal ancestry
    from Timur.
    2. They followed the principle of primoginature in the inheritance of the throne.
    Which of the statements given above is/are correct?

    Correct

    Answer: D
    Explanation:
    Statement 1 is incorrect: The Mughals in India proudly traced their ancestry to both Chingiz
    and Timur. In Babur Nama, emperor Babur talked of Chingiz, an ancestor of his mother and
    Timur, his paternal ancestor.
    Statement 2 is incorrect: The Mughals adopted the Timurid ‘appanage system’ or the
    principle of heritable division of territory. It means the entire empire will be divided amongst
    all the sons. Primoginature is the system in which the eldest son acquires the throne.

    Incorrect

  3. Question 3 of 5
    3. Question
    2 points

    Consider the following pairs:

    Kingdoms                  Dynasties
    1. Bijapur                    Barid Shahi
    2. Berar                      Imad Shahi
    3. Golconda               Nizam Shahi
    4. Bidar                        Qutb Shahi

    How many pairs given above are correctly matched?

    Correct

     Answer: A
    Explanation:
    During the reign of Vijaynagara’s greatest ruler Krishnadeva Raya (1509-29), the power of the
    Bahmanis declined, leading to the emergence of five kingdoms:
    ● the Nizam Shahis of Ahmadnagar;
    ● the Adil Shahis of Bijapur;
    ● the Imad Shahis of Berar,
    ● the Qutb Shahis of Golconda and
    ● the Barid Shahis of Bidar

    Incorrect

  4. Question 4 of 5
    4. Question
    2 points

    4. Consider the following statements about the idea of Dvaitadvaita:
    1. It was given by Ramanuja against the philosophy of maya by Shankara.
    2. The idea emphasises the identicality and the distinctness of the Supreme soul,
    individual soul and the inanimate world.
    Which of the statements given above is/are correct?

    Correct

    Answer: B
    Explanation:
    Statement 1 is incorrect: Nimbarka (12th century), a Telegu Brahman, propounded the
    idea of Dvaitadvaita (bheda-bheda; dualistic-nondualism),against Shankara’s philosophy of
    maya (illusion).
    Statement 2 is correct: Nimbarka emphasizes that the Supreme soul (God) and individual
    soul and the inanimate world are both ‘identical’ (monism) and at the same time were
    ‘distinct’ (dualism). ‘They are identical in the sense that the individual soul and the
    inanimate world are entirely dependent on God and have no independent existence’. Thus
    Nimbarka’s philosophy is both ‘monistic and pluralistic’.

    Incorrect

  5. Question 5 of 5
    5. Question
    2 points

    5. Consider the following pairs:

    Regions                         Known in ancient times

    1. Godavari basin                    Asmaka
    2. Krishna river valley           Kuntala
    3. Konkan                                Aparanta
    How many pairs given above are correctly matched?

    Correct

    Answer: C
    Explanation:

    The land south of the river Narmada was called by the Aryan settlers as Dakshinapatha and
    included the Dandakaranya which was so named from the vast forest that stretched
    southwards from the Tapti to the Godavari. During the early centuries of the common era
    Mahārāshtra consisted of three distinct portions, viz. the first, Vidarbha or Berar; the
    second, Asmaka or the Godavari basin (later known as Seuna Desh); and the third Kuntala,
    that is the valley of the river Krishnā. It also embraced the western coastal region known as
    Aparänta or Konkan, stretching from Daman in the north to Goa or even Karwar in the
    south.

    Incorrect

window.wpAdvQuizInitList = window.wpAdvQuizInitList || []; window.wpAdvQuizInitList.push({ id: '#wpAdvQuiz_576', init: { quizId: 576, mode: 0, globalPoints: 10, timelimit: 0, resultsGrade: [0], bo: 0, qpp: 0, catPoints: [10], formPos: 0, lbn: "Finish quiz", json: {"2674":{"type":"single","id":2674,"catId":0,"points":2,"correct":[0,1,0,0]},"2675":{"type":"single","id":2675,"catId":0,"points":2,"correct":[0,0,0,1]},"2676":{"type":"single","id":2676,"catId":0,"points":2,"correct":[1,0,0,0]},"2677":{"type":"single","id":2677,"catId":0,"points":2,"correct":[0,1,0,0]},"2678":{"type":"single","id":2678,"catId":0,"points":2,"correct":[0,0,1,0]}} } });